You are on page 1of 127

.

1.1. DỰ ĐOÁN SỐ HẠNG TỔNG QUÁT VÀ CHỨNG MINH BẰNG QUY NẠP.

u  11
Bài 1. Cho dãy số  un  xác định bởi :  1 . Xác định số hạng tổng quát của
un 1  10un  1  9n, n  N
dãy đã cho.

Hướng dẫn giải


Ta có:.
u1  11  10  1
u2  10.11  1  9  102  100  2 .
u3  10.102  1  9.2  1003  1000  3

Dự đoán: un  10n  n 1 .

Chứng minh theo quy nạp ta có.

u1  11  101  1 , công thức 1 đúng với n  1 . Giả sử công thức 1 đúng với n  k ta có uk  10k  k .

Ta có: uk  1  10 10k  k   1  9k  10k 1   k  1 .

Công thức 1 đúng với n  k  1 .

Vậy un  10n  n , n  N . .

u1  2
Bài 2. Cho dãy số (un ) biết  . Xác định số hạng tổng quát của dãy.
un  3un 1  1, n  2

Hướng dẫn giải


1 3 1 1
un  3un 1  1  un   3un 1   un   3(un 1  )(1) .
2 2 2 2
1 1 5
Đặt vn  un   v1  u1   .
2 2 2
(1)  vn  3vn 1 , n  2 .

Dãy (vn ) là cấp số nhân với công bội là q  3 .

5 n 1
Nên vn  v1.q n 1  .3 .
2
1 5 n 1 1
Do đó un  vn   3  , n  1, 2,... .
2 2 2
3 n4 
Bài 3. Cho dãy số  un  xác định bởi u1  1; u n 1   un  2  , n 
*
.Tìm công thức số hạng
2 n  3n  2 
tổng quát u n của dãy số theo n .

HƯỚNG DẪN GIẢI


Với mọi n  *
, ta có.
n4 2 3
2un 1  3(un  )  2un 1  3(un   )
(n  1)(n  2) n  2 n 1 .

3 3 3 3 3
 2(un 1  )  3(un  )  un 1   (un  ).
n2 n 1 n2 2 n 1 .

3 3 1
Dãy số (vn ), vn  un  là cấp số nhân có công bội q  và v1   .
n 1 2 2
n 1 n 1
3  1 3 13
vn    .    , n  *
 un     , n  *
.
2  2 n 1 2  2 

Bài 4. Cho hàm số f : Z   Z  thỏa mãn đồng thời các điều kiện:.

(1) f  n  1  f  n  , n  Z  . .

(2) f  f  n    n  2000 , n  Z  . .

a/Chứng minh: f  n  1  f  n  , n  Z  . .

b/Tìm biểu thức f  n  .

HƯỚNG DẪN GIẢI


Câu a.

Vì f  n   Z  nên từ giả thiết (1) ta được: f  n  1  f  n   1 , n  Z  . .

Kết hợp giả thiết (2) ta được n  Z  . .

n  2001   n  1  2000  f  f  n  1   f  f  n    1  n  2001 do đó: f  n  1  f  n   1 , n  Z  . .

Câu b.
f  n   f 1  n –1, n  Z   f  f 1  f 1  f 1 –1 ,.

Suyra: 1  2000  2 f 1 –1  f 1  1001  f  n   n  1000, n  Z  .

Thử lại thỏa các điều kiện, nên f  n   n  1000, n  Z  . .

Bài 5.

a)Xác định ba số hạng đầu của một cấp số cộng, biết tổng của chúng bằng 9 và tổng các bình phương của
chúng là 125.

u1  16

b)Cho dãy số  un  có  15  n.un  1 . Tìm số hạng tổng quát u n .
 n 1
u  14  , n  1
 n 1
Hướng dẫn giải
a)Xác định ba số hạng đầu của một cấp số cộng, biết tổng của chúng bằng 9 và tổng các bình phương của
chúng là 125.
Gọi d là công sai, số hạng thứ 2 là a. Khi đó 3 số hạng đầu của csc là a  d , a, a  d .

a  d  a  a  d  9
Theo giả thiết ta có hệ:  .
 a  d   a   a  d   125
2 2 2

3a  9
 2
3a  2d  125
2

.
a  3

 d  7
Vậy có 2 cấp số thỏa mãn có 3 số hạng đầu là: -4;3;10 hoặc 10;3;-4.

u1  16

b)Cho dãy số  un  có  15  n.un  1 . Tìm số hạng tổng quát u n .
un 1  14  , n  1
 n 1

15  n.un  1
Ta có: un 1  14    un 1  14  n  1  15  n.un  1 .
n 1
  n  1 un 1  15nun  14n  1 (1).

Đặt vn  nun   v1  16  .

(1) trở thành: vn 1  15vn  14n  1  vn 1   n  1  15  vn  n  (2).

Đặt w n  vn  n   w1  15 .

(2) trở thành: wn 1  15wn   w n  là csn có w1  15, q  15  w n  15n .


15n  n
Từ đó ta có: un  .
n
Bài 6. Cho dãy số  un  xác định bởi : u1  1; u2  4; un  2  7un 1  un  2, n  * .

Chứng minh : u n là số chính phương với mọi n nguyên dương.


Hướng dẫn giải
Ta có u1  1; u2  4; u3  25 .

2 3 18 123
Đặt un  vn  thì v1  ; v2  ; v3  .
5 5 5 5
2  2  2
Khi đó un  2  7un 1  un  2, n  *  vn  2   7  vn 1     vn    2, n  *
5  5  5
 vn  2  7vn 1  vn , n  *.

Ta có : vn  2 .vn  vn21  (7vn 1  vn ).vn  vn21  vn 1 (7vn  vn 1 )  vn2  vn 1vn 1  vn2 .

9
Suy ra : vn  2 .vn  vn21  vn 1vn 1  vn2   v3v1  v22  ; n  * .
5
2
 2  2  2 9 2 4  4 4  9
Suy ra :  un  2   .  un     un 1     un  2un   un  2  un     un21  un 1   
 5  5  5 5 5 25  5 25  5
2 4 9
 un  2un   7un 1  2   un21  un 1   un  2un  un21  2un 1  1  (un 1  1) 2 ; n  * .
5 5 5
Từ hệ thức un  2un  (un 1  1) ; n  * và u1 ; u2 là các số chính phương suy ra u n là số chính phương với
2

mọi n nguyên dương.

an n 1  xn n 1
 
Bài 7. Cho dãy số tăng, an  0 n  1, 2,3,.... và   0 . Xét dãy số xác định bởi
n
ai 1  ai
xn   . Chứng minh rằng tồn tại lim xn .
i 1 ai 1ai n 

Hướng dẫn giải

Dễ dàng thấy rằng dãy  xn n 1 tăng ngặt.




Trường hợp 1. Nếu   1 .


ai 1  ai 1 1 1 1 1
     xn   vậy dãy  xn n 1 .


    1
ai 1ai ai ai 1ai ai ai 1 a1

bị chặn trên do đó tồn tại lim xn .


n 

Trường hợp 2. Nếu 0    1 .

ai 1  ai 1  1 1 

      * thật vậy *   ai11  ai 1  ai   ai1  ai .
ai 1ai   ai ai 1 

ai1  ai
  ai 1 ** . Ta chứng minh (**).
 1

ai 1  ai

Xét hàm số f  x   x Trên đoạn  ai ; ai 1  rõ ràng hàm số thoả mãn điều kiện của định lí Lagrăng nên
ai1  ai a  a a  a
tồn tại số c   ai ; ai 1  thoả mãn f '  c     c 1  i 1 i   ai11  i 1 i đpcm.
ai 1  ai ai 1  ai ai 1  ai

Từ đó ta có.
1
 dãy  xn n 1 bị chặn trên do đó tồn tại lim xn .

 xn 
 a1
 n 

Bài 8. Cho dãy số  xn  được xác định bởi : x4  1 và.

xn 1  xn  1 n  2   2  n  3  3  n  4     n  2 1, với mọi n  4. .

xn
Tính giới hạn lim ..
n  n4
Hướng dẫn giải
Ta có: 1 n  2   2  n  3  3  n  4   ...  n  2  .1 .

  n  1  1  2  n  1  2    3  n  1  3  ...   n  2   n  1   n  2   .


  n  1 1  2  3  ...   n  2    12  22  32  ...   n  2   .
2
 

=  n  1 .
 n  2  n  1   n  2  n  1 2m  3  n  n  1 n  2  .
2 6 6
n  n  1 n  2 
Do đó ta suy ra : xn 1  xn   xn  Cn3 * .
6
Ta chứng minh xn  Cn4 . Thật vậy với n  4 , ta có x4  1  C44 .

Giả sử với n  4 ta có : xn  Cn4 .

Ta có : xn 1  xn  Cn4 theo (*) hay xn 1  xn  Cn3  Cn4  Cn3  Cn4 trong.

xn n! 1
lim  lim  ..
n  n 4 n  4! n  4  ! n 4
6

1 
Bài 9. Cho hàm số f :  0;     0;   thỏa mãn điều kiện f  3x   f  f  2 x    2 x với mọi x  0
2 
. Chứng minh rằng f  x   x với mọi x  0 .

Hướng dẫn giải

1 
Ta có: f (3x)  f  f (2 x)   2 x (1) .
2 

1  2x   2x 2x
Từ (1) suy ra f ( x)  f  f     f ( x)  , x  0 (2).
2  3  3 3

 1  2x   2x 2 1  2x  2x 1  2x  2x  4 2 
Khi đó f ( x)  f  f      . f    f     x .
 2  3  3 3 2  3  3 3  3  3  27 3 
2 1 2
Xét dãy (an ) ,  n  1, 2,  được xác định như sau: a1  và an 1  an2  .
3 3 3
Ta sẽ chứng minh bằng quy nạp theo n rằng với mỗi n  *
luôn có.
f ( x)  an x với x  0 (3).

Thật vậy, khi n  1 thì theo (2), ta có ngay (3).


Giả sử mệnh đề (3) đúng với n  k . Khi đó.

 1  2x   2x 1  2x  2x 1 2x 2x
f ( x)  f  f      a .f    a .a . 
 2  3  3 2 k  3  3 2 k k 3 3
.
a2  2
 k .x  ak 1.x
3
Vậy (3) đúng với n  k  1 .

Tiếp theo ta chứng minh lim an  1 . Thật vậy, ta thấy ngay an  1 n  *


. Do đó:
1
an 1  an  (an  1)(an  2)  0 , suy ra dãy (an ) tăng ngặt.
3
1 2
Dãy (an ) tăng và bị chặn trên nên hội tụ. Đặt lim an  l thì l  l 2  với l  1 , suy ra l  1 . Vậy
3 3
lim an  1 .

Do đó từ (3) suy ra f ( x )  x với mỗi x  0 (đpcm).

Bài 10. Tìm tất cả các hàm số f :  thỏa mãn đồng thời các điều kiện sau đây.

1. f  x  y   f  x   f  y  với mọi x, y  .

2. f  x   e x  1 với mỗi x  .

Hướng dẫn giải


f  x  0   f  x   f  0   f  0   0 và bởi vì f  0   e0  1  0 cho nên f  0   0 .

f  x    x   f  x   f   x   f  x   f   x   0 1 .
 x  x  x 
f  x  f    f    2  e 2  1 .
2 2  

 2x   x  x  4x 
f  x   2  e  1  f  x   f    f    4  e  1 .
  2 2  

 xn 
Dùng quy nạp theo n  1, 2,... ta CM được f  x   2n  e 2  1 .
 
 

 x0n 
Cố định x0  ta có f  x0   2n  e 2  1 .
 
 

 x0n 
Xét dãy an  2n  e 2  1 ta có:.
 
 

 x0n 
 e2 1 
lim an  lim  .x0   x0 .
x0
 n 
 2 

Vậy f  x0   x0 x0   2 .
Vậy f  x   f   x   x    x   0  3 .
Kết hợp (1) và (3) ta được f  x   f   x   0 .

Từ (2)  f   x    x  f  x   x  4  . Kết hợp (2) và (4) ta được f  x   xx  . Thử lại f  x   x


ta thấy đúng. Vậy f  x  f x  x  x  0  3 .
Kết hợp (1) và (3) ta được f  x   f   x   0 .

Từ (2)  f   x    x  f  x   x  4  . Kết hợp (2) và (4) ta được f  x   xx  . Thử lại f  x   x


ta thấy đúng.
 2015
 x1  2016

Bài 11. Cho dãy số xác định bởi  2 . Chứng minh rằng dãy số đã cho có giới hạn
 x  x   xn  , n  1
 n 1 n  
 n
hữu hạn.

Hướng dẫn giải


Trước hết, bằng quy nạp, ta dễ dàng có xn  0 n  1 và dãy số đã cho là dãy tăng.

Ta có :.
x2  x1  x12  2 x1 ;
x22 .
x3  x2   2 x1  x12  3 x1 ;
4
xk2
Giả sử xk  kx1 với k  1 . Ta có: xk 1  xk  2  kx1  x12  (k  1) x1 .
k
Theo nguyên lý quy nạp ta có xn  nx1 n  1 .

Ta có : xm  m  1 m  2017 thật vậy :


1 1
mx1  m  1  m 1  x1   1  m  m  m  2016 ;.
1  x1 1
2015
2016
Do đó xm  mx1  m  1 .

xn2
1 1 x x 2 x 1 1 1 1
Ta có với n  2 thì   n 1 n  n  2 n  2    .
xn xn 1 xn xn 1 xn xn 1 n xn 1 n n(n  1) n  1 n

1 1 n2018  1 1 
Do đó n  2018 thì      
x2017 xn i  0  x2017 i x2018i 
n  2018
 1 1  1 1 1
i 0
    
 2016  i 2017  i  2016 n  1 2016
.

1 1 1 2016 x2017
Suy ra    0  xn  .
xn x2017 2016 2016  x2017

Vậy dãy đã cho tăng và bị chặn trên nên có giới hạn hữu hạn.

u1  1; u2  2

Bài 12. Cho dãy số (un ) xác định như sau  3 1 .
u  u 
 n 1 2 n 2 n 1u n  2

a) Xác định số hạng tổng quát u n .

b) Tính lim un
n  .
Hướng dẫn giải
1 1
Biến đổi ta được: un 1  un   un  un1  với vn1  un 1  un khi đó: vn1  vn , n  2 .
2 2
1
nghĩa là dãy v2 , v3 ,...vn ,... là một cấp số cộng của v2  1; q  .
2
vn  un  un 1 
vn 1  un 1  un  2 
  un  u1  v2  v3  ...vn
........................ 
 .
v2  u2  u1
 1 1 
n2
1
n2

 un  1  1   ...     3   
 2  2   2

  1 n  2 
lim un  lim  3      3 .
x   
 2 
x 

Bài 13. Cho dãy số  un  được xác định như sau.

u1  2011; un 1  n 2  un 1  un  ,.

với mọi n  *
, n  2 . Chứng minh rằng dãy số  un  có giới hạn và tìm giới hạn đó.

Hướng dẫn giải


Từ công thức truy hồi của dãy ta được.

 1   1  1   1  1   1 
un  1  2  un 1   1  2  1   u  ...   1    1   ... 1  2  u1 .
 n  
 n    n  1 
2
 n  2 2
 2

 n    n  1   2 

Do đó un 
 n  1 n  1 .  n  2  n ... 4.2 . 3.1 .2011  n  1 .2011. Từ đó 2011
lim un  .
 n  1 3 2
2 2 2 2
n 2n 2

un4  20132
Bài 14. Cho dãy số  un  xác định bởi  u1   2014, un 1  , n  *
.
un3  un  4026

n
1
Đặt vn   , n  *
. Tính lim vn .
k 1 u  2013
3
k

Hướng dẫn giải

un4  20132
Cho dãy số  un  xác định bởi  u1   2014, un 1  3 , n  *
.
un  un  4026
n
1
Đặt vn   , n  *
. Tính lim vn .
k 1 u  2013
3
k

u 4  20132  un  2013  un  2013 3

Ta có un 1  2013  3 n  2013  .
un  un  4026 un  un2  1  4026

Từ đó bằng quy nạp ta chứng minh được un  2013, n  *


.
 un  2013  un3  2013
un 1  2013  3 1 .
 un  2013   un  2013
1 1 1 1 1 1
Từ 1 suy ra   3  3   .
un 1  2013 un  2013 un  2013 un  2013 un  2013 un 1  2013
n
 1 1  1 1 1
Do đó vn        1 .
k 1  uk  2013 uk 1  2013  u1  2013 un 1  2013 un 1  2013

Ta chứng minh lim un   .

 u  2013  0, n 
2
u 2  4026un  20132
Thật vậy, ta có un1  un  n 3  3n *
.
un  un  4026 un  un  4026

Suy ra  un  là dãy tăng, ta có 2014  u1  u2  ... .

Giả sử ngược lại  un  bị chặn trên và  un  là dãy tăng nên lim un  a   thì a  2014 . Khi đó
a 4  20132
a  a  2013  2014 (vô lý). Suy ra  un  không bị chặn trên, do đó lim un   .
a3  a  4026

 1 
Vậy lim vn  lim 1   1 .
 uk 1  2013 

Bài 15. Tìm số hạng tổng quát của dãy số  un  biết.


 1
u1 
 2
u 
 2 673 .

un  2  2(n  2) un 1  (n  4n  5n  2)un
2 3 2


n  , n  1
n3
Hướng dẫn giải
2(n  2) 2 un 1  (n3  4n 2  5n  2)un
Vì un  2  nên ta có:.
n3
(n  3)un  2  2(n  2) 2 un 1  (n  2)(n  1) 2 un .

n3
 un  2  2(n  2)un 1  (n  1) 2 un .
n2
n3
 un  2  (n  3)un 1  (n  1)un 1  (n  1) 2 un . .
n2
Đặt un  n !vn , n  , n  1 thu được.

(n  3)vn  2  (n  3)vn 1  (n  1)vn 1  (n  1)vn .

 (n  3)(vn  2  vn 1 )  (n  1)(vn 1  vn ). .

Đặt wn  vn  vn 1 , n  , n  2 thu được.

(n  1) wn  (n  1) wn 1 .
 (n  1)nwn  n(n  1) wn 1 .

Do đó.
(n  1)nwn  n(n  1) wn1  (n  1)(n  2) wn 2  ...  3.2.w2
.
 6(v2  v1 )  2016.

2016 1 1 
Như vậy wn   2016    ,n ,n  2 .
n(n  1)  n n 1 
Từ đó, với n  , n  1 , ta có.

1 1  n 1
vn  v1  2016     2016 .
 2 n 1  n 1
4033n  4031
 vn  .
2(n  1)

4033n  4031
Vậy un  n ! , n  , n  1.
2(n  1)

3 n4 
Bài 16. Cho dãy số  un  xác định bởi u1  1; u n 1   un  2  , n 
*
.
2 n  3n  2 
Tìm công thức số hạng tổng quát u n của dãy số theo n .

Hướng dẫn giải


3 n4 
Vì u n 1   un  2  nên.
2 n  3n  2 

3 n4 1,5n  6
2 u n1  3un   . 2  .
2 n  3n  2  n  1 n  2 

1,5 1,5
 2 u n 1  3un  2.  3. .
n2 n 1
1,5 1,5
 2 u n 1  2.  3un  3. .
n2 n 1

 1,5  3  1,5 
  u n 1     un  3. .
 n2 2 n 1 
1,5 3
Đặt vn  un  , khi đó ta có: vn 1  vn .
n 1 2
1,5 1
Lại có: v1  u1   .
2 4
n 1
3 1
Từ đẳng thức trên ta có công thức tổng quát của dãy  vn  là: vn    . .
2 4
n 1
1,5  3  1 3
Từ đó ta có công thức tổng quát của dãy  un  un  vn    . 
là: n 1  2  4 2  n  1 .
Bài 17. Cho dãy số  un  xác định bởi u1  1 và un 1  3un 2  2 với mọi n  1 .

a) Xác định số hạng tổng quát của dãy số  un  .

b) Tính tổng S  u12  u22  u32  ...  u2011


2
.

Hướng dẫn giải


a) Dễ thấy un  0, n  N * .

Từ un 1  3un2  2  un21  3un2  2 .

Đặt vn  un2 thì có: vn 1  3vn  2  vn 1  1  3  vn  1 .

Đặt xn  vn  1 thì ta có: xn 1  3xn . Từ đây suy ra  xn  là cấp số nhân với x1  2 , công bội là 3.

Nên: xn  2.3n 1  vn  2.3n 1  1  un  2.3n 1  1 .

b) S  2.30  2.31  2.32  ...  2.32010  2011 .

 2  30  31  32  ...  32010   2011 .

2  32011  1
  2011  32011  2012 .
3 1
Bài 18. Cho dãy số  un  được xác định bởi u1  1 và un 1  un  2n với mọi n  1 .

a) Chứng minh rằng: un  2n  1 .

b) Tính tổng S  u1  u2  u3  ...  un theo n .

Hướng dẫn giải


a) Khi n  1 : u2  u1  21  1  2  22  1 đúng.

Giả sử uk  2k  1 đúng với k  1, k  N .

Ta chứng minh: uk 1  2k 1  1 .

Thật vậy: uk 1  uk  2k  2k  1  2k  2k 1  1 .

b) S   21  1   22  1  ...   2n  1  21  22  ...  2 n  n .

2n  1
S  2.  n  2n 1  n  2 .
2 1
u1  2

Bài 19. Cho dãy số(un) xác định như sau:  un  2  1 .
un 1  (n  1, n  )
 1  ( 2  1)un


a) Chứng minh: tan  2  1.
8
b) Tính: u2015 .

Hướng dẫn giải



2 tan
   8  tan 2   2 tan   1  0 .
a) Ta có: 1  tan  tan    
4  8 8  1  tan 2  8 8
8

 
 tan 8  2  1  
  tan  2  1 (Vì tan dương).
 tan    2  1 8 8
 8
  
tan a  tan tan(a  )  tan
8  8 8  tan(a  2.  ) .
b) Đặt u1  2  tan a , ta có: u2   tan(a  ) , u3 
 8   8
1  tan a.tan 1  tan tan(a  )
8 8 8

Ta chứng minh: un  tan(a  (n  1) ), n  1, n  (*).
8
Với n  1 : u1  tan a đúng.


Giả sử (*) đúng với n  k , k  1 , hay ta có: uk  tan(a  (k  1) ) .
8
 
tan(a  (k  1) )  tan
Ta có: uk 1 
uk  2  1
 8 8  tan(a  k .  ) .
1  ( 2  1)uk 1  tan(a  (k  1)  ).tan  8
8 8

Vậy (*) đúng với n  k  1 . Vậy un  tan(a  (n  1) ), n  1, n  .
8
 3 3
Cho n  2015 , ta có: u2015  tan(a  2014. )  tan(a   251 )  tan( a  ) .
8 4 4

 2 1 
 tan(a  )   ( 2  1) 2  tan 2 .
4 2 1 8

u1  1

Bài 20. Cho dãy số thực  un  với u2  1 (n  N * ) .
u  2u  u
 n2 n 1 n

a) Chứng minh un  3  2n với mọi n  N * .

b) Tính tổng S  u1  u2  ...  u2012 .

Hướng dẫn giải


a) Dùng phương pháp qui nạp.
u1  1  3  2.1 , u2  3  2.2  1 .
Giả sử uk  3  2k  k  3 .

Ta có: uk 1  2uk  uk 1  2(3  2k )  (3  2( k  1)) .

 1  2k  3  2( k  1) .

Vậy un  3  2n với mọi n  N * .

b) S  (3  2.1)  (3  2.2)  ...  (3  2.2012) .

 3.2012  2(1  2  ...  2012)  6036  2013.2012  4044120 .

v1  8

Bài 21. Cho dãy số  vn  với v2  34 (n  N * ) .
v  8v  1996v
 n2 n 1 n

Tìm số dư khi chia v2013 cho 2011 .

Hướng dẫn giải

u1  8

Xét dãy số  un  với u2  34 (n  N * ) .
u  8u  15u
 n2 n 1 n

Ta có vn  un  mod 2011 với mọi n  N * .

Xét phương trình đặc trưng: t 2  8t  15  0 .


Phương trình trên có nghiệm t  5, t  3 .

5 A  3B  8
 un  có dạng un  A.5n  B.3n . Vì u1  5, u2  13 nên  .Ta có: A  B  1 .
25 A  9 B  34
Ta có: un  5n  3n .

Ta có 2011 là số nguyên tố Theo định lý Fecma ta có: 52010  1 mod 2011 .

32010  1 mod 2011 .

Suy ra 52013  125  mod 2011 , 32013  27  mod 2011 .

Vậy khi chia u2013 cho 2011 ta được số dư là 152 .

Suy ra khi chia v2013 cho 2011 ta được số dư là 152 .

u1  1

Bài 22. Cho dãy số  un  :  n .
3  2un 1  un   2, (n 
*
 )

a) Chứng minh dãy số  un  là dãy số giảm.

b) Lập công thức số hạng tổng quát của dãy số  un  .

Hướng dẫn giải


a) Chứng minh dãy số  un  là dãy số giảm.

un 1
Ta có: un 1   ; Chứng minh: un 1  un n  *
bằng phương pháp quy nạp.
2 3n

u1  1

Ta có:  5  u2  u1 .
u2  6

Giả sử: uk 1  uk ; k  và k  1 . Chứng minh: uk  2  uk 1 .

uk 1 1 u 1 u 1
Ta có: uk  2   k 1  k  k 1  k  k  uk 1 . Vậy un 1  un n  *
.
2 3 2 3 2 3

b) Lập công thức số hạng tổng quát của dãy số  un  .

3
Ta có: 3n (2un 1  un )  2  3n 1.un 1  3n.un  3 .
2
3 3
Đặt vn  3n un  6 , ta được: vn 1  6  (vn  6)  3  vn 1  vn .
2 2

v1  9
 3
Ta được: (vn ) :  3 là cấp số nhân có công bội q  .
vn 1  2 vn , (n 
*
) 2

n 1 n 1
3 3
Suy ra: vn  v1.    9.   .
2 2
vn  6  1 1
Vậy un   6.  n  n  .
2 3 
n
3

Bài 23. Tìm số hạng tổng quát của dãy  xn  biết rằng:.

 x0  1; x1  5; x2  125
 2 ( n  N ).
*

 xn  2 xn xn 1  3  xn 1  xn 1  10 xn 1  xn 
2

Hướng dẫn giải


Từ đề bài ta có: xn  0 với mọi n  N .

xn  2 3xn 1 10 xn
Ta có:   với mọi n  N * .
xn 1 xn xn 1

xn
Đặt yn  ta được yn  2  3 yn 1  10 yn  0 với mọi n  N * .
xn 1

Vì phương trình đặc trưng của dãy  yn  có hai nghiệm phân biệt 2;5 nên yn  A  2   B.5n với mọi
n

n N* .
 x1
 y1  x  5 B  1

. Suy ra yn  5n với mọi n  N * .
0
Với  ta có 
x
 y  2  25  A  0
 2 x1

n2  n
n 1 n  ( n 1) ...1
Ta có xn  5 .xn 1  5 .5 ....5.x0  5
n n
5 2
với mọi n  N * .
n2  n
Kết hợp với x0  1 , ta suy ra xn  5 2
với mọi n  N .

 7
u1  2
Bài 24. Cho dãy số  un  :  .
7u  4
un 1  n , n *

 2un  5

a) Chứng minh dãy số  un  là dãy số giảm.

b) Lập công thức tổng quát của dãy số  un  .

Hướng dẫn giải


a) Chứng minh dãy số  un  là dãy số giảm.

7 19
Ta có: u1  ; u2   u1  u2 .
2 8
Giả sử: uk  uk 1 với k >1. Cần chứng minh: uk 1  uk  2 .

7uk  4 7 27 1 7 27 1
Ta có: uk 1    .  uk  2   .
2uk  5 2 2 2uk  5 2 2 2uk 1  5 .

1 1
Mà uk  uk 1  
2uk  5 2uK 1  5 .

7 27 1 7 27 1
  .   .  uk 1  uk  2 (điều phải chứng minh).
2 2 2uk  5 2 2 2uk 1  5

b) Lập công thức tổng quát của dãy số  un  .

7
Ta có 0  un  , n  *
.
2
un  2 1
Xét dãy số xn  , ta có: x1 
un  1 3.

un 1  2 1  un  2  1 1
xn 1      xn  ( xn ) là cấp số nhân  xn  n
un 1  1 3  un  1  3 3 .

un  2 1 2.3n  1
   3  1 un  2.3  1  un  n
n n
.
un  1 3n 3 1 .
 1
u1  2016
Bài 25. Cho dãy số  un  :  .
u  2015un  1 , n  *
 n 1 2016

a) Chứng minh rằng un  1, n  *


.

b) Lập công thức tổng quát của dãy số  un  .

Hướng dẫn giải


a) Chứng minh rằng un  1, n  *
.

1
Ta có: u1  1
2016 .
Giả sử: uk  1, (k  1) ; Cần chứng minh: uk 1  1
.
2015uk  1
Ta có: uk  1  2015uk  1  2016   1  uk 1  1 . Vậy un  1, n  *
.
2016

b)Lập công thức tổng quát của dãy số  un  .

2015
Đặt xn  un  1 ta có x1  
2016 .

2015un  1 2015 2015


xn 1  un 1  1  1   un  1  xn
2016 2016 2016 .
n
 2015 
  xn  là cấp số nhân  xn    
 2016  .
n
 2015 
Vậy un  1    , n 
*
..
 2016 

u1  2

Bài 26. Cho dãy số  un  xác định bởi: u2  3 .
u  nu  n  2 u  2n  4, n  3
 n n 1   n2

a) Tìm số hạng tổng quát của dãy  un  .

b) Tìm số dư khi chia u2016 cho 2015 .

Hướng dẫn giải

v1  1

a) Đặt vn  un  n ta có: v2  1 .
v  n(v  n  1)  (n  2)(v  n  2)  3n  4  nv  n  2 v , n  3
 n n 1 n2 n 1   n2
Khi đó vn  vn 1  (n  1)vn 1  (n  2)vn  2 .

Lại có:.
vn  v2  (vn  vn 1 )  (vn 1  vn  2 )  ...  (v4  v3 )  (v3  v2 ) .

  (n  1)vn 1  (n  2)vn  2    (n  2)vn  2  (n  3)vn 3   ...  (3v3  2v2 )  (2v2  1v1 ) .

 ( n  1)vn 1  v1 .

Do đó vn  (n  1)vn 1 . Hay vn  (n  1)(n  2)vn  2  ...  (n  1)( n  2)...1.v1  ( n  1)! .

Vậy un  (n  1)! n .

b) Ta có u2016  2015! 2016 chia cho 2015 dư 1.

 x1  3

Bài 27. Xác định công thức số hạng tổng quát của dãy số  xn  :  xn 1 .
 xn  , n  2
 1  1  xn 1
2

Hướng dẫn giải

1 1 1 1 1
Ta có:   1  2 . Đặt yn  , khi đó ta được dãy  yn  xác định như sau: y1  và
xn xn 1 xn 1 xn 3
yn  yn 1  1  yn21 .


1    1  cos 3 
Vì y1   cot  y2  cot  1  cot 2   cot .
3 3 3 3  2.3
sin
3
 
Bằng quy nạp ta chứng minh được: yn  cot n 1
 xn  tan n 1
, n  1 .
2 .3 2 .3
.
1.2. DÃY SỐ CHO BỞI CÔNG THỨC TRUY HỒI.

u  2
Bài 1. Cho dãy số (un ) biết  1 . Xác định số hạng tổng quát của dãy.
un  3un 1  1, n  2
Hướng dẫn giải
1 3 1 1
un  3un 1  1  un   3un 1   un   3(un 1  )(1) .
2 2 2 2
1 1 5
Ñaët v n  un   v1  u1  
2 2 2 .
(1)  vn  3vn 1 , n  2

Dãy (vn ) là cấp số nhân với công bội là q  3 .

5 n 1
Nên vn  v1.q n 1  .3 .
2
1 5 n 1 1
Do đó un  vn   3  , n  1, 2,... .
2 2 2

Bài 2. a) Tính giới hạn A  lim  3


n3  n 2  1  n . 
u1  11
b) Cho dãy số (un) xác định bởi :  . Tìm công thức tính u n theo n .
un 1  10un  1  9n, n 
Hướng dẫn giải

a) Tính giới hạn A  lim  3


n3  n 2  1  n . 
Ta có: A  lim  3
n  n  1  n  lim
3 2
 n2  1
.
 n3  n2  1  n. 3 n3  n2  1  n2
2
3

1
1
 lim n2 .
2

3 1
1 1   1 1
  6   3 1   3   1
 n n   n n 
4

1
Vậy A  .
3
b) Ta có:.
u1  11  10  1
u2  10.11  1  9  102  100  2 .
u3  10.102  1  9.2  1003  1000  3

Dự đoán: un  10n  n 1 .


Chứng minh:.
Ta có: u1  11  101  1 , công thức (1) đúng với n  1 .

Giả sử công thức (1) đúng với n  k ta có: uk  10k  k .

Ta có: uk 1  10 10k  k   1  9k  10k 1   k  1 . .

Công thức (1) đúng với n  k  1 .

Vậy un  10 n  n, n  N . .

u1  4

Bài 3. Cho dãy số (un ) xác định bởi:  1 . Tìm công thức của số hạng
un 1  9 (un  4  4 1  2un ), n 
*

tổng quát (un ) ?.

Hướng dẫn giải


xn2  1
Đặt xn  1  2un  x  1  2un , xn  0  un 
2
n .
2
Thay vào giả thiết:.
xn21  1 1 xn2  1
 (  4  4 xn )  (3xn 1 ) 2  ( xn  4) 2  3xn 1  xn  4, n  N * , xn  0 .
2 9 2
Ta có 3xn 1  xn  4  3n 1 xn 1  3n xn  4.3n .
Đặt yn  3n.xn  yn 1  yn  4.3n , n  N * .
 yn 1  y1  4(3n  3n 1  ...  3)  yn 1  y1  6  2.3n 1 .
Ta có x1  3  y1  9  yn  3  2.3n .

1 1 4 1
Suy ra xn  2  n 1
, n  N *  un  (3  n 1  2 n 2 ), n  N * .
3 2 3 3
un
Bài 4. Cho dãy số  un  xác định bởi: u1  1; un 1  , n  *
. Tìm công thức số hạng tổng quát
2un  1
u n theo n. .

Hướng dẫn giải


un 1 1
Ta có un  0, n  *
. Khi đó un 1    2 ..
2un  1 un 1 un

1
Với mọi n  *
, đặt vn   v1  1; vn 1  vn  2, n  *
..
un

Suy ra, dãy số  vn  là cấp số cộng có v1  1 và công sai d  2. .

Do đó, vn  v1   n  1 d  2n  1, n  *
..

1 1
Vậy un   ..
vn 2n  1
Bài 5. Cho dãy số (un ) xác định bởi: u1  1; un 1  2un  3n , n  *
. Tìm công thức số hạng tổng quát
u n theo n .

Hướng dẫn giải


Với mọi n  *
, ta có.

un 1  2un  3n  un 1  3n 1  2(un  3n ) .

Xét dãy số (vn ), với vn  un  3n , n  *


. Ta có: vn 1  2vn . Do đó, dãy số (vn ) là một cấp số nhân có
công bội q  2 và số hạng đầu bằng 2. .

Suy ra vn  v1.q n 1  2n. .

Vậy un  vn  3n  3n  2n. .

3 n4 
Bài 6. Cho dãy số (un ) xác định bởi: u1  1; un 1   un  2  , n 
*
. Tìm công thức số
2 n  3n  2 
hạng tổng quát u n theo n .

Hướng dẫn giải


Với mọi n  *
, ta có.
n4 2 3
2un 1  3(un  )  2un 1  3(un   ).
(n  1)(n  2) n  2 n 1
3 3 3 3 3
 2(un 1  )  3(un  )  un 1   (un  ). .
n2 n 1 n2 2 n 1
3 3 1
dãy số (vn ), vn  un  là cấp số nhân có công bội q  và v1   .
n 1 2 2
n 1 n 1
3  1 3 13
vn    .    , n  *
 un     , n  *
.
2  2 n 1 2  2 

u1  3

Bài 7. Cho dãy số (un) xác định bởi:  5un  3 .
un 1  3u  1 , n 
*

 n

un  1
Xét dãy số  vn  với vn  , n  *
. . Chứng minh dãy số  vn  là một cấp số cộng. Tìm số hạng
un  1
tổng quát của dãy số  un  . .

Hướng dẫn giải


un  1 v 1
Ta có vn   un  n thay vào hệ thức truy hồi ta có.
un  1 vn  1

vn  1
5. 3
vn 1  1 vn  1 v  1 2vn  8 v  1 2vn  8
  n 1   n 1  .
vn 1  1 3. vn  1  1 vn 1  1 2vn  4 2 4
vn  1
hay vn 1  vn  3 và v1  2 . Suy ra dãy số  vn  là một cấp số cộng có v1  2 và công sai d  3. .

Ta có vn  v1   n  1 d  2  3  n  1  3n  1. .

3n  1  1 3n
Do đó un   . Thử lại thấy dãy số này thỏa mãn.
3n  1  1 3n  2
3n
Vậy số hạng tổng quát của dãy số  un  là un  n *
..
3n  2
Bài 8. Cho dãy số (un ) xác định bởi:.

u1  4

 1 .
un 1  9 (un  4  4 1  2un ), n 
*

Tìm công thức của số hạng tổng quát (un ) ?.

Hướng dẫn giải


xn2  1
Đặt xn  1  2un  xn2  1  2un , xn  0  un  .
2
Thay vào giả thiết:.
xn21  1 1 xn2  1
 (  4  4 xn )
2 9 2
 (3 xn 1 ) 2  ( xn  4) 2 .
 3 xn 1  xn  4, n  N * , xn  0

Ta có 3xn 1  xn  4  3n 1 xn 1  3n xn  4.3n .

Đặt yn  3n.xn  yn 1  yn  4.3n , n  N * .

 yn 1  y1  4(3n  3n 1  ...  3)
.
 yn 1  y1  6  2.3n 1

Ta có x1  3  y1  9  yn  3  2.3n .

Suy ra.
1
xn  2  , n  N *
3n 1
.
1 4 1
 un  (3  n 1  2 n  2 ), n  N *

2 3 3
1.3. PHƯƠNG TRÌNH ĐẶC TRƯNG.
un 1
Bài 1. Cho dãy số  un  xác định bởi u1  1, u2  2, un  2  un  2un 1 , n  1. Tìm lim .
n  u
n

Hướng dẫn giải


Dễ thấy dãy đã cho là dãy số dương, do đó không có số hạng nào của dãy bằng 0. Từ công thức truy hồi
u u
của dãy ta có n  2  2  n , n  1. .
un 1 un 1

un 1 1
Đặt vn  , n  1, ta được dãy số v1  2, vn 1  2  , n  1. .
un vn

Dễ thấy dãy  vn  là dãy số dương và vn  2, n  1 . Do đó.

1 1 1 5 5 5
  2    vn 1  , n  1. Vậy ta có 2  vn  .
vn 2 vn 2 2 2

1  5 1
Xét hàm số f  x   2  , x   2;  . Ta có f '  x    2  0, x. Do đó có hai dãy con đơn điệu của dãy
x  2 x
 vn  và hai dãy con này đều bị chặn nên chúng có giới hạn. Giả sử a  lim v2 n và b  lim v2 n 1 thì ta có
n  n 

hệ.

 1 a  b  1  2
a  2  b  a  b 
   a  b  1  2 .
b  2  1  ab  1 
 a  ab  1

Ta thấy chỉ có a  b  1  2 thỏa mãn và đây là giới hạn cần tìm.

un 1  4un2  4un  0, n  1



Bài 2. Tìm số các dãy số  un  thỏa mãn điều kiện:  1 .
u2004 
 2

Hướng dẫn giải


ết lại un 1  4un 1 – un   f  un  với f  x   4 x 1 – x  .

Nhận xét: f  x    0;1  x   0;1 . .

1
Vì vậy: u2004    0;1  u2003   0;1  u2002   0;1  ....u1   0;1 . .
2

ới 0  u1  1 tồn tại duy nhất : 0  a  và u1  sin 2 a .
2
Lúc đó: u2  4sin 2 a(1 – sin 2 a)  sin 2 2a ; u3  4sin 2 2a(1 – sin 2 2a )  sin 2 4a .

1 1
Quy nạp ta được: un  sin 2 (2n 1 a )   cos(2n ) .
2 2
1 1 1 1
u2004    cos(22004 )  .
2 2 2 2
 
 cos(22004 )  0  22004    k    (2k  1), k  Z . .
2 22005
   1 1
Vì 0  a  nên 0  2005 (2k  1)     k  22003  .
2 2 2 2 2

Do k  Z nên: k  0;1; 2;...; 22003 –1 .

  
Từ đó có tất cả 22003 giá trị u1 thỏa bài toán: u1  sin 2  2005 (2k  1)  , k {0;1;....;22003  1} .
2 

Do đó có tất cả 22003 dãy số  un  thỏa điều kiện đã cho.

Bài 3. Cho x1 , x2 ,..., xn ,... là các nghiệm dương của phương trình tan x  x được sắp theo thứ tự tăng
dần. Tính lim  xn  xn 1  .
n 

Hướng dẫn giải

    1
Xét hàm số f ( x)  tan x  x , với x     k ;  k  . Ta có f '( x)   1  0 => f ( x) tăng từ
 2 2  cos 2 x
 đến  .

   
Suy ra: trong khoảng    k ;  k  phương trình tan x  x có nghiệm duy nhất xk .
 2 2 

   
xk  yk  k với yk    ;  => tan yn  tan xn  yn  n   => lim yn  .
 2 2 n  2

lim  xn  xn 1  = lim
n  n 
 y n 
 n    yn 1   n  1   = lim   yn  yn 1    .
n 

u1  2014
Bài 4. Cho dãy số (un ) xác định như sau:  . Tìm điều kiện của
un 1  un  (1  2a)un  a n  1, 2,...
2 2

a  để dãy số (un ) có giới hạn hữu hạn và tính giới hạn đó.

Hướng dẫn giải


Ta có: un 1  un  (un  a ) 2  0  un 1  un ; n  1, 2,3,... .

* Suy ra dãy số (un ) tăng; từ đó dãy số (un ) có giới hạn hữu hạn khi và chỉ khi dãy bị chặn trên.

Giả sử tồn tại lim un  L ( L  ) , thì chuyển qua giới hạn hệ thức un 1  un2  (1  2a)un  a 2 ta có:
L  L2  (1  2a) L  a 2  L  a .

- Nếu có chỉ số k  *
mà uk  a thì un  a; n  k nên L  a trái với kết quả lim un  L  a .

Do đó: uk  a với mọi k  1, 2,... hay un2  (1  2a)un  a 2  a, n  1, 2,3,... nói riêng
u12  (1  2a )u1  a 2  a  a  1  u1  a  a  1  2014  a từ đó ta được 2014  a  2015 .

* Đảo lại: Nếu 2014  a  2015  a  1  u1  a .


 (u1  a  1)(u1  a )  0  u12  (1  2a)u1  a 2  a  0  u2  a .

và u1  u2  a  1  u2  a .

Bằng quy nạp ta chứng minh được a  1  un  a, n  1, 2,3,... (H/s trình bày ra).

Như vậy dãy (un ) tăng, bị chặn trên bới a , do đó dãy (un ) có giới hạn hữu hạn.

Kết luận: Với điều kiện 2014  a  2015 thì dãy số (un ) có giới hạn hữu hạn và lim un  a .

Bài 5. Cho hai dãy số  an  và  bn  được xác định như sau:.

2an .bn
a1  2, b1  1 , an 1  ; bn 1  an 1.bn , n  1, 2,  .
an  bn

Chứng minh rằng  an  và  bn  có cùng giới hạn, tìm giới hạn đó.

Hướng dẫn giải


Bằng quy nạp, ta chứng minh rằng:.
 
2n.sin 2n.sin
1 ; bn 
n
an  2 .3 2n.3 (2).
  
sin .cos n
sin
3 2 .3 3
Từ (1), (2) tồn tại lim an và lim bn .
n  n 

 
2n.sin n
2 .3 3 2 3
Ngoài ra: lim an  lim   .
n  n     9
sin .cos sin
3 2n.3 3

 2 3
lim bn  lim an .lim cos n
 .
n  n  n  2 .3 9

2 3
Vậy hai dãy an  , bn  có cùng giới hạn chung là .
9
 1
 x1 
2
Bài 6. Cho dãy số (xn) thỏa mãn:  2
. Chứng minh dãy số trên có giới hạn.
 x  x  n ; n  1
x
 n 1 n
n2

Hướng dẫn giải


n  n  1
*) Ta chứng minh xn  n 2  với mọi n  1 (1).
2
Thật vậy: n  1 đúng.
k  k  1
Giả sử (1) đúng với n  k  1: xk  k 2  .
2
xk2
  k  1 = k2  xk  k 2    k  1 .
x
 xk 1   k  1  xk 
2 2 2
2
k k

 k  1  k  k  1 3  k  1 k  k  1
2

  1   k  1 
2
 .
k 2  2 2 2

k  1  3  k  1   k  1 k  2 
  k  (đpcm).
2  2  2

*) Ta chứng minh  xn  có giới hạn.

NX:  xn  tăng và xn  0 với mọi n .

1 1 1 2 1 1  1 1
Ta có       2 1    2  xn  với mọi n  1.
xn xn 1 xn  n 2
n  n  1 x1 xn  n 2 2

Vậy  xn  có giới hạn.

Bài 7. Tam giác mà 3 đỉnh của nó là ba trung điểm của ba cạnh tam giác ABC được gọi là tam giác
trung bình của tam giác ABC. .

Xây dựng dãy các tam giác A1 B1C1 , A2 B2C2 , A3 B3C3 ,.... sao cho tam giác A1 B1C1 là một tam giác đều
cạnh bằng 1 và với mỗi số nguyên n  2, tam giác An BnCn là tam giác trung bình của tam giác
An 1 Bn 1Cn 1 . Với mỗi số nguyên dương n , kí hiệu rn tương ứng là bán kính của đường tròn ngoại tiếp
tam giác An BnCn . Chứng minh rằng dãy số  rn  là một cấp số nhân. Hãy xác định số hạng tổng quát của
cấp số nhân đó?.
Hướng dẫn giải
1 1
+  rn  là một cấp số nhân với công bội q  và số hạng đầu r1  ..
2 3
1
+ Số hạng tổng quát: rn  ..
3.2n 1

Bài 8. Cho dãy số  an  được xác định bởi: a1  1 và an 1  an  2n  1 với mọi n  1. Xét dãy số  bn 
mà: bn  an 1  an với mọi n  1 .

a) Chứng minh rằng dãy số  bn  là một cấp số cộng. Hãy xác định số hạng đầu và công sai của cấp số
cộng đó.
b) Cho số nguyên dương N . Hãy tính tổng N số hạng đầu tiên của dãy số  bn  theo N . Từ đó, hãy suy
ra số hạng tổng quát của dãy số  an  . .

Hướng dẫn giải

a) Từ giả thiết  bn  2n  1   bn  là một cấp số cộng với số hạng đầu b1  1 và công sai d  2. .

b) + Tổng N số hạng đầu của dãy  bn  là: S N  N 2 . .


+ Số hạng tổng quát của dãy  an  là: an  n 2  2n  2. .

u1  1, u2  2, u3  40

Bài 9. Cho dãy số  un  được xác định bởi  10un21.un 3  24un 1.un2 2 .
u
 n  n  4,5, 6,...
 un  2 .un 3

Tìm số n nhỏ nhất để u n chia hết cho 2048.

Hướng dẫn giải

un 10un 1.un 3  24.un22 10un 1 24un 2 u


Từ công thức truy hồi cuả dãy    , đặt vn  n , thì dãy ( vn )
un 1 un 2 .un 3 un  2 un 3 un 1
v2  2, v3  20
xác định bởi  .
vn  10vn 1  24vn 2 , n  4,5, 6...

Phương trình đặc trưng : x 2  10 x  24  0 , từ đó suy ra : vn  6n 1  4n 1 .


( n 1) n
un  vn .vn 1.vn  2 ....v2  2 2
(3n 1  2n 1 ).(3n  2  2n  2 )...(3  2) .
( n 1) n
Do (3n 1  2n 1 ).(3n  2  2n  2 )...(3  2) là số là số lẻ nên un 2048  2 2
2048 .

n(n  1)
  11  n  6 .
2
Vậy n  6 là giá trị cần tìm thỏa mãn điều kiện bài toán.
1.6. SỬ DỤNG PHÉP THẾ LƯỢNG GIÁC

U1  3

Bài 1. Cho dãy số U n  định bởi  Un  2 1 * . Tính U 2013 .
U  ; n  1, 2,3,...


n 1
1  1 2 U n 
Hướng dẫn giải.

Tính đúng tan  2  1.
8

2.tan
   8  tan   2  1 .
1  tan  tan  2  
4  8  1  tan 2  8
8

U n  tan
Từ * ta viết được U n 1  8 1 .

1  U n .tan
8

 
Theo quy nạp từ 1 và U1  3  U n  tan    n  1 .  .
3 8

   6047 
Vậy U 2013  tan   2013.   tan  .
3 8  24 

u1  2

Bài 2. Cho dãy số xác định như sau:  u 2 3
un 1  n  n   . Tính u
* 2014 .


1  3  2 un  
Hướng dẫn giải.
 
tan  tan
    3 4  3 1  2  3 .
Ta có: tan  tan    
12  3 4  1  tan  tan  1  3
3 4

un  tan
Nên từ giả thiết ta có: un 1  12 .

1  un .tan
12

tan   tan
Đặt 2  tan   u1  tan  , suy ra u2  12  tan      .
  
1  tan  .tan  12 
12

  
Theo quy nạp ta dễ dàng suy ra: un  tan     n  1  , n  *
.
 12 

    
Suy ra: u2014  tan    2013.   tan    168   .
 12   4

tan   tan
  4  1.
 tan     
 4  1  tan  .tan  3
4
1.7. CÁC DẠNG KHÁC
Bài 1.

 p
 xi  4
 i 1
 p
a/Tìm p  N * sao cho hệ  x11  4 có nghiệm.
 i 1
 x  0, i 1, p
 i

b/Với p tìm được ở câu a/,hãy xác định tập hợp tất cả các giá trị của tổng:.
p p
ai

i 1 1  ai
2
với ai  0 và a i 1
2
i  1.

HƯỚNG DẪN GIẢI


Câu a.

 p  p 1 
Do: 16    xi  .    p  p  4.
2

 i 1   i 1 xi 

p  4 :Khi đó: xi  1, i 1, 4 . Vậy hệ có nghiệm.

 x2  x3  3
p  3 :Chọn x1  1 và  có nghiệm. Nên  x1 , x2 , x3  là nghiệm của hệ.
 x2 .x3  1

 x1  x2  4
p  2: có nghiệm. Nên  x1 , x2  là nghiệm của hệ.
 1 2
x . x  1

p  1 :Vô nghiệm.

Vậy hệ có nghiệm khi p  2, p  3, p  4 .


Câu b
p
ai2
Ta có: f  a1 , a2 ,..., a p    .
i 1 ai (1  a1 )
2

Xét hàm: g  x   x 1  x 2  , 0  x  1; g   x   0  x 
1 2
. Ta có: max g ( x)  .
3 (0;1) 3 3

3 3 p 2 3 3
Do đó: f  a1 , a2 ,..., a p   
p
ai  . Dấu đẳng thức xảy ra khi:  1 hay p = 3. .
2 i 1 2 3

a1 a2 1 1
p  2 : f  a1 , a2    2 2  2 2 vì a12  a22  1 . Dấu đẳng thức ra khi a1  a2  ,
2
a2 a1 a1.a2 xảy 2

a1 1  a12
f (a1 , a2 )   liên tục trên  0;1 . Khi a1  0 thì f (a1 , a2 )   .Vậy p  2 , tập giá trị là:
1  a12 a12
 2 2;  . .
 
1
p  3 :Chọn a1  1  2 x ; a 2  x ; a 3  x , 0<x< .Thỏa giả thiết:.
2

1 2x x x 1
a12  a22  a32  1  2 x  x  x  1. f(a1 , a2 , a3 )     g ( x) tục trên (0; ) ;
2x 1 x 1 x liên 2
1 3 3 3 3 
g   , lim g(x)=+ .Vậy tập giá trị là:  ;   .
3 2 x 0
 2 

p  4 : f  a1 , a2 ,..., a p  
3 3
. Chọn a1  1  2 x ; a 2  x ; a 3  x , a 4  x thỏa giả thiết:
2
1 1 2x x x x
a12  a22  a32  a42  1  3x  x  x  x  1 với 0<x< ; f(a1 , a2 , a3 , a4 )      g ( x)
3 2x 1 x 1 x 1 x
1 3 3 3 3 
liên tục trên (0; ) ; lim g(x)= ; lim g(x)=+ .Tập giá trị là:  ;   .
3 x1 2 x 0 2
3  
n 1
Bài 2. Kí hiệu H n là tập hợp các đa thức bậc n dạng: f ( x)  x n   ai xi , a i  R . Chứng minh:
i 0

f H n 
min max | f ( x) | 
x 1;1  1
2n 1
.

HƯỚNG DẪN GIẢI

Xét đa thức Trêbưsép T  x   cos  n.arccosx  .

Chứng minh T  x  là đa thức bậc n có hệ tử bậc n là 2n –1 .

Chứng minh bằng quy nạp dựa vào công thức: cosnt  cos  n  1 t  2cost.cos  n  1 t .

T ( x) T ( x) 1 1
Do đó: n 1
 H n . Ta có max n 1  n 1 . Nếu tồn tại f  x   H n sao cho f ( x)  n 1 ,.
2 2 2 2

x   1;1 . Lúc đó ta xét g  x   f  x  


T ( x)
đa thức bậc nhỏ hơn hay bằng n –1 , g  x  đổi dấu n  1
2n 1
k
lần tại các điểm cos , k  0, n .
n

Do đó maxf ( x) 
2
1
n 1 f H n
x 1;1 2
1
. Vậy min max | f ( x) |  n 1 .
  

Bài 3. Cho dãy số  xn  không âm thỏa mãn x1  0 ,

và  n  1 xn21   2n  4   n  1 xn 1  2n 1  22 n 2  9n 2 xn2  36nxn  32 , n  1 .


2

Chứng minh rằng xn là số nguyên với mọi nguyên tố lớn hơn hoặc bằng 5 .

HƯỚNG DẪN GIẢI

Viết lại đẳng thức trong đầu bài về dạng  n  1 xn 1  2n 1  2    3nxn  6 
2 2

Từ xn không âm dẫn đến  n  1 xn 1  2n 1  2  3nxn  6 , với mọi n .


Biến đổi về  n  1 xn 1  2n  2  3  nxn  2n 1  2  ,.

Bài 4. Cho dãy số dương  xn  thoả mãn: xn  xn 1  2 xn  2 với mọi số tự nhiên n  1 . Chứng minh rằng
dãy {xn} hội tụ.

Hướng dẫn giải


Đặt yn  max  xn ; xn 1 . .

Từ (1) và (2) suy ra yn  yn 1  0; n  *


 a  lim(y n ) .

Với   0 tuỳ ý, khi n đủ lớn, ta có   yn  a  0 .


Nếu yn  a thì   yn  a  xn  a  0 .
Nếu xn  a thì xn 1  a  yn 1  xn 1 .

Mà xn  xn 1  2 xn 1  2a  xn  xn 1  2a  a  x n  2a  xn 1  a   . .

Tóm lại, cả hai trường hợp đều dẫn đến xn  a   .

Vậy dãy số {xn} hội tụ.


Bài 5. Cho phương trình x 2   x  1  0 với  là số nguyên dương. Gọi  là nghiệm dương của
phương trình. Dãy số  xn  được xác định như sau:.

x0   , xn 1    xn  , n  0,1, 2,3,... .

Chứng minh rằng tồn tại vô hạn số tự nhiên n sao cho xn chia hết cho  .

Hướng dẫn giải


Đầu tiên ta chứng minh  là số vô tỉ. Thật vậy, nếu  là số hữu tỉ thì  là số nguyên (do hệ số cao nhất
của x 2 là 1) và  là ước của 1. Do đó   1 suy ra   0 , trái giả thiết.

Do đó   xn 1    xn 1    xn 1   1 .

 xn   xn 1  xn  1.

xn xn 1 1 xn
  xn 1    xn 1    xn 1 .
    

x  1
  n   xn1  1 (1). Lại có  2    1  0 , suy ra     .
  

xn  x  x 
  xn   xn   xn 1   xn  n    xn   n    xn  xn 1  1 (do (1)).
    
Vậy xn 1  xn 1  1 (mod  ) . Từ đó bằng quy nạp ta có với mọi k *
, n  2k  1, thì
xn 1  xn (2 k 1)  (k  1) (mod  ) (2).

Chọn k  1  l  l  *
 , n  1  2l , từ (2) ta có x 2 l  x0  l    l  0 (mod  ) .

Vậy x2l chia hết cho  , l  *


..
Bài 6. Cho dãy  an  với n > 0 được xác định bởi:.

a1  1; a2  2; a3  6; a4  12

an  4  2a n 3  an  2  2a n 1  an  n  1
a) Chứng minh an chia hết cho n với mọi giá trị nguyên dương của n .
an
b) Đặt bn  . Chứng minh tồn tại vô số số nguyên dương n để 2015 là một ước của bn . .
n
Hướng dẫn giải
a) Ta có b1  1; b2  1; b3  2; b4  3 .

Dễ thấy bn  Fn với n  1; 2;3; 4. Bằng quy nạp ta chứng minh dãy  bn  trùng với dãy  Fn  .

Thật vậy:.
Mệnh đề đúng với n  1; 2;3; 4. Giả sử mệnh đề đúng đến n  3 . Khi đó ta có:.

 n  4  bn 4  2  n  3 Fn3   n  2  Fn 2  2  n  1 Fn1  nFn . .


Dùng công thức của dãy Fibonaci : Fm  2  Fm 1  Fm ta dễ dàng biến đổi vế phải thành  n  4  Fn  4 .

suy ra bn  4  Fn  4 . .

Vậy mệnh đề đúng với n  4 , do đó nó đúng với mọi n nguyên dương.


Điều đó chứng tỏ an luôn chia hết cho n với mọi n nguyên dương.

b) Gọi rn là số dư của bn cho 2015 với n  1; 2;3... .

Trước tiên ta chứng minh  rn  là một dãy tuần hoàn. Thật vậy: Ta có
bn  2  bn 1  bn  rn  2  rn 1  rn  mod 2015  . .

Vì có vô hạn các cặp  r1 ; r2  ,  r2 ; r3  , .,  rn ; rn 1  nhưng chỉ nhận hữu hạn giá trị khác nhau nên tồn tại ít
nhất hai phần tử của dãy trùng nhau. Ta giả sử là  rm ; rm1    rmT ; rm T 1  (với T là một số nguyên
dương).

Ta chứng minh  rn  tuần hoàn với chu kỳ T . .

+) Ta có: rm  2  rm 1  rm  mod 2015  ; rmT  2  rmT 1  rmT  mod 2015 .

 rm  2  rm T  2  mod 2015   rm  2  rm T  2 . .

Tiếp tục như vậy ta chứng minh được: rm  k  rm T  k với mọi k  0. (1).

+) Ta có: rm 1  rm 1  rm  mod 2015  ; rm T 1  rm T 1  rm T  mod 2015 .

 rm 1  rm T 1  mod 2015  .

 rm 1  rm T 1. .

Bằng quy nạp ta chứng minh được: rm  k  rm T  k với k  1; 2;3;...; m  1. (2).


Từ (1) và (2) suy ra  rn  , n  0 là một dãy tuần hoàn.

Bổ sung vào dãy  bn  phần tử b0  0 thỏa mãn b0  b1  b2 suy ra r0  0. .

Khi đó dãy  rn  là dãy tuần hoàn bắt đầu từ phần tử đầu tiên r0  0. Do đó tồn tại vô số phần tử trong dãy
 rn  bằng 0.Như vậy câu b) được chứng minh xong.
Bài 7. Cho dãy số  un  được xác định như sau: u0  0, u1  1, un  2  2un 1  un , n  0,1, 2,... Chứng minh
rằng 22014 un khi và chỉ khi 22014 n .

Hướng dẫn giải


Công thức tổng quát un 
2
1
2
1 2   1 2   .
n n

     b  ab   1 .
n n
Đặt 1  2  a, 1  2
n

Ta có un 
2 2
1
 a  b  , u2 n 
2
1
2
a 2
 b 2   un  a  b  .

   1  2  . Khi đó ta được dãy  S  được xác định như sau: S


n n
Đặt Sn  a  b  1  2 n 1  2, S 2  6,
Sn  2  2Sn 1  Sn , n  1, 2,... .
Do S1  2  mod 4  , S 2  2  mod 4  nên bằng quy nạp ta được: S n  2  mod 4  hay
a  b  2  mod 4   a  b  2t ,  t , 2   1 .

Do đó u2 n  2un .t ,  t , 2   1 .
Giả sử n  2k .t ,  t , 2   1  un  u2k .t  2 k.ut . Ak , trong đó ut , Ak đều lẻ.

Bài 8. Cho dãy số  an  : a1  *


, an 1  an3  2019, n  *
. Chứng minh có nhiều nhất 1 số hạng của
dãy là số chính phương.

Hướng dẫn giải


So sánh đồng dư của an , an 1 và an  2 theo modun 4 ta có (chú ý 2019  3  mod 4  ).

an 0 1 2 3

an 1 3 0 3 2
an  2 2 3 2 3

Một số chính phương khi chia 4 có số dư là 0 hoặc 1.


Vì vậy từ số hạng thứ 3 trở đi, dãy không có số chính phương nào.
Nếu cả a1 và a2 đều chính phương, giả sử a1  a 2 , a2  b 2 ,.

suy ra b 2  a 6  2019   b  a 3  b  a 3   2019 .

Hơn nữa khi phân tích 2019 thành tích chỉ có 2 cách 2019  1.2019  3.673 .
b  a 3  1
 b  1010
Trường hợp 1:    3 , vô lí do 1009 không là lập phương.
b  a  2019
 a  1009
3

b  a 3  3
 b  338
Trường hợp 2:    3 , vô lí do 335 không là lập phương.
b  a  673
 a  335
3

Vậy điều giả sử sai, nghĩa là dãy trên có nhiều nhất 1 số chính phương.

un  N
u  u  u  {0;1}
 m  n m n
Bài 9. Cho dãy  un  thỏa mãn các điều kiện sau : u2  0 . Tìm u2013 .
u  0
 3
u9999  3333

Hướng dẫn giải


Ta có : um  n  um  un   (  {0;1}) .

Bằng quy nạp ta chứng minh được un1  n2 ... nk  un1  un2  ...  unk , với mọi n1 , n2 ,..., nk .

Ta có: u2  u1  u1  u1  0 .

u3  u2  u1    0    u3  1 .

Ta chứng minh rằng nếu n  3333 thì u3n  n (1).

Thật vậy:.
Với n  1 thì (1) đúng.
Ta có u3n  n.u3  n, n .

Giả sử, tồn tại n0  3333 , mà u3n0  n0  u3( n0 1)  u3n0 3  u3n0  u3  n0  1 , điều này chứng tỏ, với mọi
n  n0 thì u3n  n . Điều này mâu thuẫn với u9999  3333 .

Vậy, với n  3333 thì u3n  n. .

Do đó u2013  671 .

17 1
Bài 10. Cho dãy số xn xác định bởi: x1  5; x2  ; xn 1  xn .xn21  2 xn  4 . Tìm n chẵn thỏa mãn
2 4
n  N * và  xn   3 là lập phương của 1 số tự nhiên.

Hướng dẫn giải


Nhận xét thấy :.
11
1 4 21
1 4
x1  22  211 1
; x2  22  221 1
;.
2 2
n1
1 4
Khi đó, giả sử : xn  22  2n1 1
n  k ; k  N *. .
2
1 4
Cần chứng minh: xk 1  22 
k

2k 1
. (1) thật vậy ta có.
2
1 1 k 1 4 k 2 4 k 1 4
xk 1  xk xk 12  2 xk  4  (22 1  2k 1 1 )(22 1  2k 2 1 )2  2(22 1  2k 1 1 )  4 .
4 4 2 2 2
1 4

k
= 22 2k 1
suy ra (1) đúng.
2
n1 4
 xn  22 1
 2n1 1
n  N *
2 .

Khi đó  xn   3  22  3 , giả sử tồn tại n chẵn để  xn   3 là lập phương của 1 số tự nhiên:.


n1
1

n1
Khi đó 22 1
 3  c 3 . Mặt khác n chẵn suy ra n  1 lẻ suy ra 2 n1  1 3 khi đó đặt.

  c  c.2k  22 k   3 mà c 2  c.2k  22 k  c  2k nên:.


n 1
22 1
 23 k  2 3 k  3  c 3  c  2 k 2

c  2k  1; c 2  c.2k  22 k  3 (2). Giải hệ (2) ta được hệ không có nghiệm nguyên với mọi k  0 suy ra
không tồn tại n chẵn.

Vậy không tồn tại n chẵn để  xn   3 là lập phương của một số tự nhiên.

Bài 11. Cho dãy số  un  được xác định như sau: u0  0, u1  1, un  2  2un 1  un , n  0,1, 2,... Chứng minh
rằng 22014 un khi và chỉ khi 22014 n .

Hướng dẫn giải

Công thức tổng quát un 


2
1
2
1 2   1 2   .n n

     b  ab   1 .
n n
Đặt 1  2  a, 1  2
n

Ta có un 
2 2
1
 a  b  , u2 n 
2
1
2
a 2
 b 2   un  a  b  .

     Sn 
n n
Đặt Sn  a  b  1  2  1  2 . Khi đó ta được dãy được xác định như sau:
S1  2, S 2  6, S n  2  2Sn 1  S n , n  1, 2,... .

Do S1  2  mod 4  , S2  2  mod 4  nên bằng quy nạp ta được: S n  2  mod 4  hay


a  b  2  mod 4   a  b  2t ,  t , 2   1 .

Do đó u2 n  2un .t ,  t , 2   1 .

Giả sử n  2k .t ,  t , 2   1  un  u2k .t  2 k.ut . Ak , trong đó ut , Ak đều lẻ.

Từ đẳng thức này ta được 2k un khi và chỉ khi 2k n .


 x1  1

Bài 12. Cho dãy số thực  xn  được xác định như sau:  1 . Chứng minh rằng:
 xn 1  xn  2 x , n  1
 n

 25 x625   625 ( kí hiệu  x  là phần nguyên của số thực x ).

Hướng dẫn giải


1 1 1
Ta chứng minh rằng: n  n xn  n  H n , n  1 , với H n  1    .
8 2 n
1
xn21  xn2   1 , x12  1 quy nạp xn2  n .Với n  1 đúng giả sử đúng đến n . Tức là xn2  n . Từ đó suy
4 xn2
ra.
1
xn21  n  1   n  1  nxn  n .
4 xn2
n 1
1 1 1 n 1 1
xn2  xn21  1   x12   n  1    n  
4 xn21 2
k 1 4 xk 4 k 1 k
2
1  1  1
 n  Hn   n  H n   nxn  n  H n .
4  8 n  8

Việc tiếp theo ta chứng minh H 625  8 . Ta có BĐT H n  1  ln n thật vậy,.

1  1 1
Xét hàm số f  x   ln  x  1  ln x   ln 1    x  0 .
x 1  x  x 1
1 1
f  x     0 , x  0 hàm số f  x  giảm trên khoảng.
x  x  1  x  12

1
 0;    f  x   0, x  0 , ta suy ra  ln  x  1  ln x * áp dụng.
x 1
1 1
1    1  ln 2  ln1  ln 3  ln 2   ln 625  ln 624  1  ln 625  8 .
2 625

Từ đó: 625  625 x625  625  H 625  626   25 x625   625 .


1
8
2. MỘT SỐ DẠNG TOÁN LIÊN QUAN ĐẾN TÍNH CHẤT CỦA DÃY SỐ.
Bài 1. Cho cấp số cộng  un  với n là số nguyên dương thoã mãn u2013  2013; u2014  2014 . Tính
1 1 1
tổng: S    ....  .
u1u2 u2u3 u2013u2014

Hướng dẫn giải


Dễ dàng chứng minh được số hạng tổng quát của cấp số cộng  un  là un  n .

Khi đó.
1 1 1 1 1 1
S   ....     ...
u1u2 u2u3 u2013u2014 1.2 2.3 2013  2014
1 1 1 1 1 1 1006 503
     ...    
2 3 3 4 2013 2014 2014 1007 .

 x0  a

Bài 2. Cho dãy số thực  xn  được xác định bởi.   n   . Tìm tất cả các giá trị của a
 xn 1  2 xn  1

2
.
để xn  0 với mọi số tự nhiên n .

Hướng dẫn giải


Giả sử xn  0 với n  .

2
Từ xn  2  2 xn21  1  0 có   xn 1  0 .
2

2 2  2 2 1
Lại từ   2 xn2  1  0 có   xn   1  xn   , n  .
2 2 2 4
1 3 1
Suy ra xn   và xn   1, n  .
2 4 2

1 1 1 1 1 3 1
Từ đó xn 1   2 xn2  1   2 xn2   2 xn  . xn   xn  , n  .
2 2 4 2 2 2 2
Áp dụng liên tiếp bất đẳng thức này, ta có:.
2 n n
1 1 2 1 2 1 2 1 2
a   x0   x1     x2   ...    xn     , n  .
2 2 3 2 3 2 3 2 3
n
2 1 1
Mà lim    0 nên phải có a   0  a   .
 
n 3 2 2
1 1
Thử lại với a   thì xn    0, n .
2 2
1
Vậy a   là giá trị duy nhất cần tìm.
2
 x0  20; x1  30
Bài 3. Cho dãy số  xn  xác định bởi  . Tìm n để xn 1.xn  1 là số chính
 xn  2  3xn 1  xn , n 
phương.

Hướng dẫn giải


Từ công thức truy hồi của xn ta có.

 
n  , x n21  x n2  3xn 1 xn  x n21  xn x n  3xn 1  x n21  xn  2 xn

và x n21  x n2  3xn 1 xn  xn 1  x  3x   x
n 1 n
2
n
 x n2  xn 1 xn 1
Suy ra x n21  xn  2 xn  x n2  xn 1 xn 1  ...  x12  x0 x2  500
.
 x n21  x n2  3xn 1 xn  500
 x n21  x n2  3xn 1 xn  500

 
2
 x n1  x n  xn 1 xn  500

Vậy xn 1 xn  500 là số chính phương.

Giả sử n là số thỏa mãn xn 1 xn  500 là số chính phương.

Đặt xn 1 xn  500  b 2 , xn 1 xn  1  a 2 , a, b  , a  b .

Ta có a 2  b 2  501   a  b  a  b   1.501  3.167 .

Khi đó ta tìm được a = 201, b=1 thì xn 1 xn  12600  n  2 .

7224
Với a = 85, b =82 thì xn 1 xn  n.
5
Vậy n = 2 thì xn 1.xn  1 là số chính phương.

u1  2
Bài 4. Dãy số  un  xác định như sau:  . Chứng minh rằng
un 1  un  un  1,  n  *.
2

1 2016 1 1
1 2015    1  2 2016 .
22 k 1 u
k 2

Hướng dẫn giải

Ta có: un 1 – un  u n2 –2un  1   un –1 . (1).


2

Do u1  2  u2 – u1  1  u2  u1 .

Từ đó bằng phép quy nạp ta suy ra  un  là dãy đơn điệu tăng thực sự, và un nhận giá trị nguyên dương
lớn hơn hoặc bằng 2 với mọi n  1, 2,.... .

Ta viết lại điều kiện truy hồi xác định dãy số dưới dạng sau đây:.

un 1 – 1  u n2 –un  un  un –1 (2).

1 1 1 1 1 1 1
Từ đó dẫn đến:       , (3) Bây giờ từ (3), ta có:.
un 1  1 un (un  1) un  1 un un un  1 un 1  1
n 
n
1 1 1  1
 u
k 1 k
  
 u
k 1  k  1

u  1 
 1
u  1
. (4) .
k 1  k 1

Từ (4) suy ra bất đẳng thức cần chứng minh tương đương với.
1 1 1 n1
1  1  1  22  un 1  1  22 (5) .
n

n1
un 1  1
n
22 22
(ở đây n  2016 ). Ta sẽ chứng minh (5) đúng với mọi n . Khi đó nó sẽ đúng với n  2016 .
Do u n nguyên dương với mọi n , (5) tương đương.
n1
22  1  un 1  1  22 . (6).
n

Xét khi n  k  1 . Theo (2), ta có: uk 2 –1  uk 1  uk 1 –1 .

Vì thế theo giả thiết quy nạp suy ra:.


k 1
uk  2  1  22 (22  1)  22 .22  22
k k k k

k 1 k 1 k 1 k 1
.
uk  2  1  (22  1).(22  1  1)  2 2 .2 2  22
k

Như thế với n  k  1 , ta thu được:.


k 1
2 2  uk  2  1  2 2
k

k 1
 2 2  1  uk  2  1  2 2 . (8) .
k

Từ (8) suy ra (6) đúng với mọi n  2,3,... .

Vì vậy (5) đúng n  2016 . Ta có điều phải chứng minh!.

an2  5an  10
Bài 5. Cho dãy ( an ) n 1 : a1  1; an 1  n  1 .
5  an

a) Chứng minh dãy (an ) hội tụ và tính lim an .

a1  a2  ...  an 5  5
b) Chứng minh  n  1 .
n 2
Hướng dẫn giải

a) Bằng phương pháp chứng minh qui nạp ta có: 1  an  3 n .


2

5 5 x 2  5 x  10 10
Đặt A  và xét hàm f ( x)    x( x  5) .
2 5 x 5 x
10  3 1 
Suy ra f '( x)   1  0x  1;  , như vậy f ( x) nghịch biến trên đoạn  2 ;1 .
5  x   2
2

a  a3  a5  ...  a2 k 1  ...  A  lim a2 k 1  b  A


Dẫn đến  1  .
 a2  a4  a6  ...  a2 k  ...  A   lim a2 k  c  A
 c 2  5c  10
 b 
5c 5 5
Kết hợp công thức xác định dãy ta được:   b  c  .
c  b 2
 5b  10 2
 5b

5 5
Vậy lim an  .
2

 5 5 
b) Nhận xét: t  1;  thì t  f (t )  5  5 .
 2 

Dẫn đến a2 k 1  a2 k  5  5 k  1 .

5 5
 a1  a2  ...  a2 k 1  a2 k  2k (1).
2
Như vậy bất đẳng thức đúng với n  2k .

5 5
Trường hợp n  2k  1 , chú ý a2 k 1  , kết hợp với (1) thu được:.
2

5 5
a1  a2  ...  a2 k 1  a2 k  a2 k 1  (2k  1) .
2
Vậy bất đẳng thức được chứng minh.

u1  1

Bài 6. Cho dãy số  un  như sau u2  2 .

nun  2   3n  1 un 1  2  n  1 un  3, n 
*

a) Chứng minh un  2n  3n, n  * .


n 1
b) Đặt Sn   uk . Chứng minh rằng nếu n là số nguyên tố và n > 2 thì S n chia hết cho n.
k 1

Hướng dẫn giải


a) Với n  1 , u1  21  3.1  1 .

n  2 , u1  22  3.2  2
.

Giả sử uk  2k  3k ; uk 1  2k 1  3  k  1 .

Chứng minh uk  2  2k  2  3  k  2  , k  *
.

Ta có.
kuk  2   3k  1 uk 1  2  k  1 uk  3
.

 kuk  2   3k  1  2k 1  3  k  1   2  k  1  2k  3k   3
.

 uk  2  2 k  2  3  k  2  .
Vậy uk  2  2k  2  3  k  2  , k  *
.
.
n 1
b) Đặt Sn   uk . Chứng minh rằng nếu n là số nguyên tố và n  2 thì S n chia hết cho n .
k 1

n 1
Ta có: Sn   uk  2  22  ...  2n 1  3 1  2  ...  (n  1) 
k 1 .

1  2n 1 (n  1)n (n  1)n
Sn  2.  3.  2  2n1  1  3
1 2 2 2 .

Với n là số nguyên tố  2  1 chia hết cho n .


n1

(n  1)n
Do n là số nguyên tố lớn hơn 2  chia hết cho n .
2
Vậy S n n .

u1  0

Bài 7. Cho dãy số  un  u2  18 . Chứng minh rằng nếu n là số nguyên tố và

un  2  5un 1  6un  24, n 
*

n  3 thì u n chia hết cho 6n .

Hướng dẫn giải


Đặt vn  un  12 hay un  vn  12, n  *
.

Khi đó vn  2  5vn 1  6vn .

v1  12

Ta được  vn  v2  30 .
v  5v  6v
 n2 n 1 n

Phương trình đặc trưng  2  5  6  0 có nghiệm   2    3 .

Khi đó vn  a.2n  b.3n .

v  12 2a  3b  12 a  3
Ta có  1   .
v2  30 4a  9b  30 b  2

Suy ra vn  3.2n  2.3n .

Khi đó un  vn  12  3.2n  2.3n  12 .

Ta có un  6  2n 1  3n 1  2  nên u n chia hết cho 6 .

Mặt khác n là số nguyên tố nên theo định lý Fermat.

2n  2(mod n)
 3.2n  6(mod n)

 n hay  n .
3  3(mod n)
 2.3  6(mod n)

Từ đó un  (3.2n  2.3n  12)  0(mod n) .


Suy ra u n chia hết cho n .

Với n là số nguyên tố và n  3  (n, 6)  1 .

Suy ra u n chia hết cho 6n .

 x 1
 1
Bài 8. Cho dãy số  xn  với  .
 x  xn  xn  5   xn 2  5 xn  8   16 n  N 
*
 n 1

a) Chứng minh xn  5n 1 , với mọi n  2 .


n
1
b) Đặt yn   . Tìm lim yn .
k 1 xk  3 n 

Hướng dẫn giải


a) Chứng minh xn  5n 1 , với mọi n  2 .

x2  10  5  521 .

Giả sử ta có xn  5n 1  n  2  .

xn 1  xn  xn  5   xn 2  5 xn  8   16  x
n
2
 5 xn  xn 2  5 xn  8   16
.
 xn 2  5 xn  4  5 xn  5.5n 1  5n

Suy ra xn 1  5n .

Vậy theo qui nạp xn  5n 1 với n  2 .


n
1
b) Đặt yn   . Tìm lim yn .
k 1 xk  3 n 

Ta có:.
xn 1  xn 2  5 xn  4  xn 1  2  xn2  5 xn  6   xn  2  xn  3 .

1 1 1 1
   
xn 1  2  xn  2  xn  3 xn  2 xn  3
.
1 1 1
  
xn  3 xn  2 xn 1  2
n
1 n
 1 1  1 1 1 1
yn          .
k 1 xk  3 k 1  xk  2 xk 1  2  x1  2 xn1  2 3 xn1  2

1 1  1 1
lim yn  lim     (vì xn 1  5n  lim  0 ).
n  n  3
 xn 1  2  3 n  x
n 1

1
Vậy lim yn  .
n  3
u1  2
Bài 9. Cho dãy số (un ) được xác định như sau:.  . Chứng minh
un  3un 1  2n  9n  9n  3, n  2
3 2

p 1
rằng với mọi số nguyên tố p thì 2014 ui chia hết cho p .
i 1

Hướng dẫn giải


Với mọi n  2 ta có: un  n3  3  un 1  (n  1)3  .

Từ đó có: un  n3  3  un 1  (n  1)3   32  un  2  (n  2)3   ...  3n 1  u1  13   3n .


Vậy un  3n  n3 , n  2 , lại có u1  2  31  13 nên un  3n  n3 , n  1 .
+ Nếu p  2 : có ngay đpcm.
p 1
+ Nếu p là số nguyên tố lẻ: u i  (3  32  ...  3 p 1 )  13  23  ...  ( p  1)3  .
i 1

1 1 p 1 1 p 1
3 
 (3 p  3)   i 3   p  1   (3 p  3)   i 3   p  i    .
3

2 2 i 1   2 i 1
 

Theo Định lí Fermat nhỏ, suy ra 3 p  3 chia hết cho p . Mặt khác i 3   p  i  cũng chia hết cho
3

p 1
p, i  1, p  1 nên: (3 p  3)   i 3   p  i   chia hết cho p . Từ đó.
3

i 1
 
p 1
 p 1
3 
2014 ui  1007 (3 p  3)   i 3   p  i    chia hết cho p .
i 1  i 1
 
Vậy bài toán được chứng minh cho mọi trường hợp.

 x0  20; x1  30
Bài 10. Cho dãy số  xn  xác định bởi  . Tìm n để xn 1.xn  1 là số chính
 xn  2  3xn 1  xn , n 
phương.

Hướng dẫn giải


Từ công thức truy hồi của xn ta có.


n  , x n21  x n2  3xn 1 xn  x n21  xn x n  3xn 1  x n21  xn  2 xn
và x n21  x n2  3xn 1 xn  xn 1  x  3x   x
n 1 n
2
n
 x n2  xn 1 xn 1
Suy ra x n21  xn  2 xn  x n2  xn 1 xn 1  ...  x12  x0 x2  500
.
 x n21  x n2  3xn 1 xn  500
 x n21  x n2  3xn 1 xn  500

 
2
 x n1  x n  xn 1 xn  500

Vậy xn 1 xn  500 là số chính phương.

Giả sử n là số thỏa mãn xn 1 xn  500 là số chính phương.

Đặt xn 1 xn  500  b 2 , xn 1 xn  1  a 2 , a, b  , a  b .

Ta có a 2  b 2  501   a  b  a  b   1.501  3.167 .


Khi đó ta tìm được a  201, b  1 thì xn 1 xn  12600  n  2 .

7224
Với a  85, b  82 thì xn 1 xn   n .
5
Vậy n = 2 thì xn 1.xn  1 là số chính phương.

Bài 11. Bài 3. Cho phương trình x 2   x  1  0 với  là số nguyên dương. Gọi  là nghiệm dương
của phương trình. Dãy số  xn  được xác định như sau x0   , xn 1    xn  , n  0,1, 2,3,... .
Chứng minh rằng tồn tại vô hạn số tự nhiên n sao cho xn chia hết cho  .

Hướng dẫn giải


Đầu tiên ta chứng minh  là số vô tỉ. Thật vậy, nếu  là số hữu tỉ thì  là số nguyên (do hệ số cao nhất
của x 2 là 1) và  là ước của 1. Do đó   1 suy ra   0 , trái giả thiết.

Do đó   xn 1    xn 1    xn 1   1 .

 xn   xn 1  xn  1.

xn xn 1 1 xn
  xn 1    xn 1    xn 1 .
    

x  1
  n   xn1  1 (1). Lại có  2    1  0 , suy ra     .
  

xn  x  x 
  xn   xn   xn 1   xn  n    xn   n    xn  xn 1  1 (do (1)).
    
Vậy xn 1  xn 1  1 (mod  ) . Từ đó bằng quy nạp ta có với mọi k *
, n  2k  1, thì
xn 1  xn (2 k 1)  (k  1) (mod  ) (2).

Chọn k  1  l  l  *
 , n  1  2l , từ (2) ta có.
x2l  x0  l    l  0 (mod  ) .

Vậy x2l chia hết cho  , l  *


..

a0  a1  2004 a  10
Bài 12. Cho dãy số  an  xác định bởi  . Chứng minh rằng n là
 n2
a  7 an 1  an  3978, n  . 2014
số chính phương.

Hướng dẫn giải


Ta có.
an  2  10 a  10 an  10
an  2  7an 1  an  3978   7. n 1   2. .
2014 2014 2014

an  10 v0  v1  1
Đặt vn  . Ta được dãy số  vn  xác định bởi  .
2014 vn  2  7vn 1  vn  2, n  .
Ta phải chứng minh vn là số chính phương.
 x0  1; x1  1
Thật vậy, xét dãy số ( xn ) xác định bởi  .
 xn  2  3xn 1  xn , n  .

Hiển nhiên dãy số  xn  là dãy số nguyên.

n  , xn21  xn2  3 xn 1 xn  xn21  xn ( xn  3xn 1 )  xn21  xn xn  2 .


và xn21  xn2  3 xn 1 xn  xn 1 ( xn 1  3 xn )  xn2  xn2  xn 1 xn 1.
Ta có .
 xn21  xn xn  2  xn2  xn 1 xn 1  x12  x0 x2  1.
 xn21  xn2  3 xn 1 xn  1, n  . (2)

Ta sẽ chứng minh vn  xn2 , n  (1) bằng quy nạp.

Thật vậy, rõ ràng với n  0, n  1 , (1) đúng.

Giả sử (1) đúng đến n  k  1, k  , tức là vn  xn2 , n  1, 2,..., k  1. .

ta chứng minh (1) đúng với n = k+2, nghĩa là chứng minh vk  2  xk2 2 .

Thật vậy, theo công thức truy hồi của dãy số  an  , giả thiết quy nạp, tính chất (2) của dãy số  xn  , công
thức truy hồi của dãy số  xn  , ta có.

vk  2  7vk 1  vk  2  7 xk21  xk2  2  7 xk21  xk2  2( xk21  xk2  3xk 1 xk )


.
 9 xk21  6 xk 1 xk  xk2  (3xk 1  xk ) 2  xk2 2 .

Do đó vn là số chính phương. Vậy ta có điều phải chứng minh.

Bài 13. Cho dãy số ( xn ) được xác định bởi xn  2013n  a 3 8n3  1, n  1, 2,... a là số thực

a)) Tìm a sao cho dãy số có giới hạn hữu hạn.


b) Tìm a sao cho dãy số ( xn ) là dãy số tăng (kể từ số hạng nào đó).

Hướng dẫn giải


a) Ta có xn  (2a  2013)n  ayn , trong đó yn  3 8n3  1  2n .

8n3  1  (2n)3 1
   0 Khi n   .
3
(8n  1)  2n 8n  1  4n
3 2 3 3 2 3
(8n  1)  2n 3 8n3  1  4n 2
3 2

2013
Do đó tồn tại giới hạn hữu hạn lim xn khi và chỉ khi a   .
n  2
b) Từ lý luận phần a) ta suy ra)
 2013
 khi a   2

 2013
lim xn  0 khi a   .
n  2

 2013
 khi a   2

2013
Bởi vậy điều kiện cần để tồn tại m  N * sao cho xm  xm 1  xm  2  .... là a   .
2
2013
Ta đi chứng minh a   là điều kiện đủ để có kết luận trên.
2
2013
Thật vậy: Với a   .
2

xn 1  xn  2013(n  1)  a 3 8(n  1)3  1  2013n  a 3 8n3  1


 2013  a( 3 8( n  1)3  1  3 8n3  1) 
2013 3
2013  ( 8(n  1)3  1  3 8n3  1)  .
2
2013
[2  ( 3 8(n  1)3  1  3 8n3  1)] 
2
2013
(2  3 8n3  1  3 8(n  1)3  1)  0
2
Vì.

 
2
(2  3 8n3  1)3  8  12 3 8n3  1  6 3
8n 3  1  8 n 3  1 

8  12.2n  6(2n) 2  8n3  1  8(1  3n  3n 2  n3 )  1 .


 8(n  1)  1
3

Suy ra x1  x2  x3  .... .

2013
Vậy dãy số ( xn ) là dãy số tăng kể từ số hạng nào đó với a   và trong trường hợp đó ( xn ) là dãy số
2
tăng từ x1 .
3.1. TÍNH GIỚI HẠN BẰNG ĐỊNH NGHĨA.

 1
 a1  a  a

Bài 1. Cho dãy số  an  xác định bởi :  . Chứng minh rằng với mọi số thực
 an 1  2an  2an  2
3 2

 3an 2  4an  1
a  0 thì dãy  an  hội tụ. Tùy theo a , hãy tìm giới hạn của dãy  an  .

Hướng dẫn giải


1
Nếu a  0 thì a   2 (do bất đẳng thức AM-GM).
a
1 1
Nếu a  0 thì a   2 (do bất đẳng thức AM-GM) nên a   2 .
a a

Nếu a  1 thì a1  2 . Ta chứng minh: an  2, n  *


.

nn n a1  2 .

2.23  2.22  2
Giả sử ak  2  ak 1  2.
3.22  4.2  1
Vậy lim an  lim 2  2 .

a  0
. Nếu  thì a1  2 . Ta chứng minh an  2 n  *
.
a  1
Rõ ràng a1  2 . .

Giả sử ak  2 . Ta chứng minh ak 1  2 .

2ak 3  2ak 2  2
ak 1  2   2  2ak  ak  2   0 ( đúng).
2

3ak  4ak  1
2

Ta chứng minh  an  là dãy giảm, thật vậy :.

an3  2an 2  an  2   an  1  an  2 
2

n, an 1  an    0.
3an 2  4an  1 3an 2  4an  1

( do tử âm, mẫu dương vì.

 2 7
 an 
3
3an 2  4an  1  0   .
 2 7
 an 
 3

2 7
Mà an  2   3an 2  4an  1  0 ).
3

 an  giảm và bị chặn dưới   an  có giới hạn là L .


2an 3  2an 2  2 2 L3  2 L2  2
lim an 1  lim 
3an 2  4an  1 3L2  4 L  1 .
 L  2  an  2  L  1 

Vậy lim an  2 .

. Nếu a  0 thì a1  2 . Tương tự, ta có:.

an3  2an 2  an  2   an  1  an  2 
2

n, an 1  an    0.
3an 2  4an  1 3an 2  4an  1

nên  an  tăng. ơn nữa  an  bị chặn trên bởi 1 , thật vậy.

2ak 3  2ak 2  2
ak 1  1   1   ak  1 (2a  3)  0 .
2

3ak  4ak  1
2

Vậy  an  tăng và bị chặn trên   an  có giới hạn là L .

an  1, n , an 1  an  0, n
2 L3  2 L2  2 .
L  L  1  a   1  L  2 
3L2  4 L  1
n

Vậy lim an  1 .

Tóm lại: + Nếu a  1 thì lim an  2 .

a  0
+ Nếu  thì lim an  2 .
a  1
+ Nếu a  0 thì lim an  1 .

 x1  0

Bài 2. Cho dãy số  xn  được xác định bởi 
n  
1 2 3 2015 . Tìm giới
 xn 1  xn   2 3  *

 xn xn xn xn2015
hạn của dãy nxn khi n   , với  là số thực c o trước.

Hướng dẫn giải


Dễ dàng c ứng m n được xn  0, n  1 bằng qu nạp.

Ta có.
2
1  1 1
xn 1  xn  , n  1 xn21   xn    xn2  2  2  xn2  2 ; n  1 .
xn  xn  xn

Bở vậy n  , n  2 thì xn2  xn21  2  xn2 2  4    x12  2  n  1 .

 xn  1, n  2 và lim xn   .
n 

1 2 3 2015
Vớ n  *
, đặt xn 1  xn   tn trong đó tn  2  3  2015 .
xn xn xn xn
t
xn  1; n  2  0  tn  , vớ t  2  3  2014  2015 (1), suy ra.
xn2
2
 1  1 2t
x2
n 1  x   xn   tn   xn2  2  tn2  2  2 xntn  n  2 . khi n   .
2
n
 xn  xn xn

 b1  x12
Áp dụng địn lý trung bìn Cesaro c o dãy  bn  vớ  .
bn  xn  xn 1 , n  2.
2 2

b1  b2  bn
ta có lim bn  2 suy ra lim  lim bn  2. .
n  n  n n 

xn2  xn  xn 1    xn 1  xn 2    x2  x1   x1 b1  b2  bn


2 2 2 2 2 2 2
n 1
Mà   suy ra lim 2  . .
n n n n  x 2
n

n 1
T ật vậy ta có t c ứng m n trực t ếp lim  n ư sau (c ứng m n địn lý trung bìn Cesaro).
n  xn2 2

Xét dãy  cn  : c1  x12  2; cn  xn2  xn21  2 vớ n  2,3 .


lim cn  0 nên   0 tồn tạ m  *
sao cho cn  ,  n  m. .
n  2

Gọ M  max  ci  vớ 1  i  m 1 .

 2  m  1 M  2  m  1 M  m  1 M   .
Vớ  ở tr n tồn tạ m     1 thì  m ' hay
    m 2

Xét n  max m, m ' . ta có.


|  i 1ci |  
m 1
 n  m  1
2   m  1 M     m  1 M       . o đó t eo địn
n n
c | ci |
 im i
 i 1

n n n n n 2 m 2 2
|  i 1ci |
n

ng ĩa lim  0.
n  n

xn2  xn  xn 1    xn 1  xn 2    x2  x1   x1 c1  c2  cn


2 2 2 2 2 2 2
n 1
   2 . suy ra lim 2  . .
n n n n  x 2
n

1
Nếu   2 thì n.xn  n.xn2  khi n   .
2

Nếu   2 thì n.xn  xn  2 .n.xn2   khi n   .

Nếu   2 thì n.xn  xn  2 .n.xn2  0 khi n   .

Cho hai số a1 , b1 với 0  b1  a  1 .Lập hai dãy số  an  ,  bn  với n  1, 2,.. .Theo quy tắc
Bài 3. 1

1
sau: giả ng ĩa cá đó là:. an 1  (an  bn ) , bn 1  an 1.bn Tính: lim an và lim bn .
2 . n  n 

Hướng dẫn giải



Tính a2 , b2 vớ 0  b1  a1  1 ta có t c ọn 0  a  sao cho: b1  cosa ,.
2
Suy ra a1  cos 2 a .

1 1 a
a2  (cos 2 a  cos a)  cos a(cos a  1)  cos a.cos 2 .
2 2 2

a a
b2  cos a.cos 2 .cos a  cos a.cos .
2 2
Bằng quy nạp, c ứng m n được:.
a a a a a
an  cos a.cos ...cos n 1 cos n 1 (1) bn  cos a.cos ...cos n 1 (2) .
2 2 2 2 2
a
N ân a vế của (1) và (2) c o sin và áp dụng công t ức sin 2a được:.
2n 1
a
sin 2a.cos
an  2n 1 , bn 
sin 2a
.
a a
2n.sin n 1 n
2 .sin n 1
2 2
Tín g ớ ạn:.
sin 2a sin 2a
lim an  , lim bn  .
n  2a n  2a
1 a
Bài 4. Cho dãy số  an  , a1  1 và an 1  an  .Chứng minh: lim n  2 .
an n  n

Hướng dẫn giải


n n 1 n 1
1 1
ak21  ak2  2
ak
 2  
i 2
ai
2
 
j 1
a 2
j   2
j 1 a j
 2(n  1). .

n 1
1
an2  2n  1   2
. Vậy an  2n  1 , n  2. .
j 1 a j

1 1 1 1 1 1 1
ak2  2k  1 k  2        .
4
a k (2k-1) 2
(2k-1)  1 4k(k+1) 4  k  1 k 
2

n 1 n 1
1 1 1 1 1 1 5
Suyra:   (1  )     1  .
4
k  2 ak 4 n 1 4 4
j 1 a j 4 4

n 1 n 1
1 1 5
Suyra:  2
 ( n  1)  4
 (n  1) (n  2). .
j 1 a j j 1 a j 4

5(n  1)
Vậy: an2  2n  1  (n  2) .
2

5(n-1) 1 a 5(n-1)
Suyra: n  2; 2n-1<a n < 2n-1+  2- < n  2n-1+ .
2 n n 2
an
Dođó: lim 2.
n  n
 
Bài 5. Cho hai số a1 , b1 với a1  cos 2 , b1  cos . Lập hai dãy số  an  ,  bn  với n  1, 2,... theo quy
8 8
1
tắc sau:. an 1  (an  bn ) , bn 1  an 1.bn . Tính: lim an và lim bn .
2 n  n 

Hướng dẫn giải


+Tính a2 , b2 :.

1   1    
a2  (cos 2  cos )  cos (cos x  1)  cos .cos 2 .
2 8 8 2 8 8 8 16

    
b2  cos cos 2 cos  cos cos .
8 16 8 8 16
+ Bằng quy nạp, c ứng m n được:.
      
an  cos cos 2
...cos n
cos n
(1) bn  cos cos 2
...cos (2) .
2.4 2 .4 2 .4 2 .4 2.4 2 .4 2n .4

+N ân a vế của (1) và (2) c o sin và áp dụng công t ức sin 2a được:.
2n .4
  
sin .cos n
sin
an  4 2 .4 , bn  4 .
n  n 
2 .sin 2 .sin
2n .4 2n .4
+Tín g ớ ạn:.
 
4sin 4sin
lim an  4 , lim bn  4.
n   n  
Bài 6. Cho dãy số  un  biết:.

 u1  1

 un , n  N * .
u 
 n 1 1  u 2
 n

Hãy tính lim (un n ) .


n 

Hướng dẫn giải


Ta có: u1  0  un  0 , n  N * .

un  1  un  un / (1  un2 )  un  (un3 ) / (1  un2 )  0 n  N * .

  un  là dãy số g ảm và bị c ặn dướ bở 0 .

 lim un  a (a  R, a  0) .
n 
Từ un  1  un / (1  un2 ), cho n   ta được:.

a  a / (1  a 3 )  a  0. Vậy lim un  0 .
x 

Đặt vn  1/ (un2  1)  1/ (un2 ), n  N * .

Ta có vn  ((1  un2 ) / un ) 2  1/ (un2 )  2  un2  2 khi n   ? Áp dụng địn lí trung bìn Cesaro ta có:.

1 1

v1  v2  vn u 2
u12
lim  2  lim n 1
2.
n  n n   n

 1 1  1 1
 2  2  2  2
 lim  n 1
u u n  u n u1
 2.
n  n
1 1 1
2

u u 2n v u2 1
Mà lim n 1
 lim n  0 ; lim 1  lim  0 .
n  n n  n n  n n  n

1
u2 1 1
 lim n  2  lim 2
 2  lim (un n )  .
n  n n  n.u n   2
n

U1  2

Bài 7. Cho dãy U n  xác định bởi:  U n2  2009U n  n  N  .
*

U n 1 
 2010

 n
Ui 
Ta lập dãy S n  vớ  Sn    .Tính lim Sn .
i 1 U i 1  1 
x 

Hướng dẫn giải
a0
Tacó a1    0.
2

G ả sử a1 , a2 ,..., an 1  0 .

Tacó.

 an an 1 a0
 1  2  ...  n  1  0 1 1  1 1 1 1 
  an     an 1     an  2  ...     a0 .
 an 1  an 2  ...  a0  0 1 2   2 3  n n 1 
 1 2 n
an 1 an 2 a1 a0
Hay an    ...   .
1.2 2.3 (n  1)n n(n  1)

Do a1 , a2 ,..., an 1  0 nên.
 an 1 an  2 a1   2an 1 3an  2 na 
   ...     ...  1 
 1.2 2.3 (n  1)n   1 2 n 1 
2
.
 an 1 an  2 a1  a02
   ...    2
 1 2 (n  1)  n

a a a1  a02
  n 1  n  2  ...    .
 1.2 2.3 (n  1)n  2  2an 1 3an  2 na1 
n    ...  
 1 2 n 1 
Ta lạ có.
2an 1 3an  2 na  2a 3a a 
  ...  1  n  n 1  n  2  ...  1 
1 2 n 1  n 2n n 1 
.
a a a   a 
 n  n 1  n  2  ...  1   n   0    a0 .
 1 2 n 1   n

a a a1  a0
  n 1  n 2  ...   2 .
 1.2 2.3 (n  1)n  n

an 1 an 2 a1 a0 a a0
 an    ...     02  0.
1.2 2.3 (n  1)n n(n  1) n n(n  1)
Từ đó suy ra đ ều p ả c ứng m n .

1  un2  1
Bài 8. Cho dãy số  un  xác định bởi u1  1, un 1  , n  1.
un

a) C ứng m n :.

un  tan , n  1. .
2n 1
b) Suy ra tín đơn đ ệu và bị c ặn của  un  .

ƯỚNG DẪN GIẢI


a) C ứng m n bằng quy nạp toán ọc.
   
b) N ận xét 0  n 1
 , n  1 và àm số tanx đồng b ến tr n  0;  .
2 4  4

n n dãy số  un  g ảm và bị c ặn dướ bở số tan 0  0 .


và bị c ặn tr n bở số tan  1.
4 .

Bài 9. Cho dãy số  xn  xác định bởi:.

1 2 3 2014 2015
x1  0; xn 1  xn   2  3  ...  2014  2015 , n  *
..
xn xn xn xn xn

n
1.Với mỗi n  *
,đặt yn  .Chứng minh dãy số  yn  có giới hạn hữu hạn và tính giới hạn đó.
xn2
2.Tìm các số  đ dãy  nxn  có giới hạn hữu hạn và giới hạn là một số khác 0 .

HƯỚNG DẪN GIẢI


1 1
1.Từ giả thiết suy ra xn 1  xn   0  xn21  xn2  2  2  xn2  2
xn xn
.
Suy ra xn21  xn2  2  xn21  2  ...  x12  2n do đó lim xn   .

Xét
 1 2 3 2014 2015  1 2 3 2014 2015 
xn21  xn2   xn1  xn  xn 1  xn    2 xn   2  3  ...  2014  2015   2  3  ...  2014  2015 
 xn xn xn xn xn  xn xn xn xn xn 

 1 2 3 2014 2015  2 3 2014 2015 


  2  2  3  4  ...  2015  2016 1   2  ...  2013  2014 
 xn xn xn xn xn  xn xn xn xn 
.

Suy ra lim  xn21  xn2   2


.

xn2  xn  xn 1    xn 1  xn 2   ...   x2  x1   x1
2 2 2 2 2 2 2

Ta có  .
n n
Áp dụng định lý trung bình Cesaro ta có.

xn2
lim  lim
 xn2  xn21    xn21  xn22   ...   x22  x12   x12
2 .
n n
n 1
Do đó lim 
xn2 2
.
n  2
2.Xét zn  nxn  xn
xn2
.
Từ đó:.
+) Nếu   2 thì lim zn  

.
+)Nếu   2 thì lim zn  0

.
1
+) Nếu   2 thì lim zn 
2
.
Vậy   2 là giá trị cần tìm thỏa mãn đề bài.

Bài 10. Cho dãy số  yn  thỏa mãn y1  0, yn31  y1  y2  ...  yn , n  1 .


y 
Chứng minh rằng dãy số  n  có giới hạn bằng 0 khi n   .
n
Hướng dẫn giải
Từ giả thiết ta có yn31  yn  yn3 , n  2 , do đó dãy số  yn n  2 là dãy tăng, vì.

vậy yn31  yn  yn3  yn ( yn2  1)  yn 1 ( y2n  1) .

 yn21  yn2  1 , n  2  yn21  yn2  1  ...  y22  n  1 .

y22  n  1
2
 y  y2  n 1
  n1   2 . Mà lim  0 n n t eo định lý kẹp ta có.
 n 1  (n  1) 2 (n  1)2
2
 y  y y
lim  n1   0  lim n1  0  lim n  0 .
 n 1  n 1 n

un  (0;1)
Bài 11. Tìm tất cả các hằng số c  0 sao cho mọi dãy số dãy số (un ) thỏa mãn:  n  1
un1 (1  un )  c
.

đều hội tụ. Với giá trị c tìm được hãy tính giới hạn của dãy (un ) .

Hướng dẫn giải


Ta xét các trường hợp sau.
1 c cun
+ Nếu c  , thì từ giả thiết, ta có un 1    4cun ; n  1 .
4 1  un un (1  un )

1
Từ đây bằng quy nạp, ta suy ra un  (4c) n 1 u1 . Do 4c  1 nên un   khi n   . Do đó, c 
4
không thỏa mãn.

1  1  1  4c 1  1  4c  a(1  b)  c
+ Nếu 0  c  , thì tồn tại a, b   ;  , a  b sao cho  . Thật vây, lấy

4  2 2  b(1  a)  c
 1  1  4c 1  1  4 c 
a   ;  , đặt b  a  x ( x  0) , thì.
 2 2 
a(1  a)  c
a(1  b)  c  a(1  a  x)  c  x  .
a
Chú ý là b(1  a )  a (1  a )  c. Do đó, ta c ỉ cần chọn x  0 n ư tr n và b  a  x, t ì được 2 bất đẳng
thức nêu trên.
Xét dãy số (un ) xác định bởi.

a nêu n  2m
un   .
b nêu n  2m  1
1
thì dãy (un ) thỏa mãn giả thiết n ưng k ông ội tụ. Thành thử, 0  c  cũng k ông t ỏa mãn.
4
1 1 un
+ Nếu c  , thì un 1    un . Suy ra dãy (un ) tăng và bị chặn. Do đó, (un ) hội tụ.
4 4(1  un ) 4un (1  un )

1 1 1
Đặt x  lim un , thì từ giả thiết ta có x(1  x)  hay x  . Vậy lim un  . .
4 2 2

 1
 x1  2
Bài 12. Cho dãy số (xn) thỏa mãn:  2
. Chứng minh dãy số trên có giới hạn.
 x  x  n ; n  1
x
 n 1 n
n2

Hướng dẫn giải


n  n  1
*) Ta chứng minh xn  n 2  với mọi n  1 (1).
2
Thật vậy: n  1 đúng.
k  k  1
Giả sử (1) đúng với n  k  1: xk  k 2  .
2

xk2
  k  1  k2  xk  k 2    k  1 .
x
 xk 1   k  1  xk 
2 2 2
2
k k

 k  1  k  k  1 3  k  1 k  k  1
2

  1   k  1 
2
 .
k 2  2 2 2

k  1  3  k  1   k  1 k  2 
  k  (đpcm).
2  2  2

*) Ta chứng minh  xn  có giới hạn.

NX:  xn  tăng và xn  0 với mọi n .

1 1 1 2 1 1  1 1
Ta có       2 1    2  xn  với mọi n  1.
xn xn 1 xn  n 2
n  n  1 x1 xn  n 2 2

Vậy  xn  có giới hạn.

un4  20132
Bài 13. Cho dãy số  un  xác định bởi u1  2014, un 1  , n  *
. Đặt
un3  un  4026
n
1
vn   , n  *
. Tính lim vn .
k 1 u  2013
3
k

Hướng dẫn giải

un4  20132 (un  2013)(un3  2013)


+ Ta có un 1  2013   2013  (1).
un3  un  4026 (un3  2013)  (un  2013)

Từ đó bằng quy nạp ta chứng m n được un  2013, n  *


.
1 1 1 1 1 1
+ Từ (1) suy ra   3  3   .
un 1  2013 un  2013 un  2013 un  2013 un  2013 un 1  2013
n
 1 1  1 1 1
Do đó vn        1 .
k 1  uk  2013 uk 1  2013  u1  2013 uk 1  2013 uk 1  2013

+ Ta chứng minh lim un   .

un2  4026un  20132 (un  2013)2


Thật vậy, ta có un1  un    0, n  *
.
un3  un  4026 un3  un  4026

Suy ra  un  là dãy tăng, ta có 2014  u1  u2  ... .

a 4  20132
Giả sử  un  bị chặn trên và lim un  a thì a  2014 . K đó a  .
a3  a  4026
 a  2013  2014 ( vô lí). Suy ra  un  không bị chặn tr n, do đó lim un   .

1
Vậy lim vn  lim (1  ) 1 .
uk 1  2013

u1  2013 un21


Bài 14. Cho dãy số  un  xác định bởi:  . Tìm lim .
un1  un  2, n 
2 * n  u 2 .u 2 ...u 2
1 2 n

Hướng dẫn giải


1
- Vì u1  2013  2 n n đặt u1  a  , a > 1.
a
2
 1 1
Ta có u2  u  2   a    2  a 2  2 .
2
1
 a a
Bằng quy nạp, ta chứng m n được.
1
un 1  a 2  , n 
n

n .
a2
- Xét.
1 1
n n
 i1 1   1   1  n  2i1 1   1   2n 1 

i 1
ui    a 2  2i1
i 1  a

 
 
a   
a  
a     a  2i1     a    a  2n  1.0
a  i 1  a   a  a 
2
 1  2n 1 
 a   a  2n  2 2
.
a   lim un 1   a  1    a  1   4  20132  4 1.0
2 2
 2 n21 2  
u a 
   
 a  a
2 n  u 2 .u 2 ...u 2
u1 .u2 ...un  2n 1 
 2n 
1 2 n
a
 a 
Bài 15. Cho dãy số (an ) thỏa mãn: lim(5an 1  3an )  4 . Tính lim an .

Hướng dẫn giải


Đặt an  2  bn . Từ giả thiết suy ra lim (5bn 1  3bn )  0 .

Với số dương  bé tùy ý, tồn tại số N sao cho với n  N thì ta có:.

5bn 1  3bn  (1).
5

- Nếu bn 1.bn  0 thì từ (1) dẫn đến 5bn 1  3bn   bn   .
5
- Xét trường hợp bn 1.bn  0 hay bn 1 , bn cùng dấu, chẳng hạn c úng cùng dương.

 
. Nếu 2bn 1  bn  0 thì kết hợp với (1): 3(2bn 1  bn )  bn 1  dẫn đến bn 1  .
5 5

Mà từ (1) ta có 3bn  5bn 1   bn   .
5
5 1 
. Nếu 2bn 1  bn  0 thì kết hợp với (1): (bn 1  bn )  bn  dẫn đến bn   .
2 2 5

Tóm lại luôn có bn   , hay lim(bn )  0 .

Vậy lim(an )  2 .

un2015  2un  4
Bài 16. Cho dãy (un ) xác địn n ư sau: u1 = 3 và un 1  2014 , n  1, 2,3... . Với mỗi số
un  un  6
n
1
nguy n dương n , đặt vn   . Tìm lim vn .
i 1 ui
2014
4 n 

Hướng dẫn giải

un2015  2un  4 (un  2)(un  4)


Đặt   2014 ta có un 1  2   2  , (*) .
un2014  un  6 (un  4)  (un  2)

Bằng quy nạp ta chứng m n được un  3, n  1 .

un 1  2un  4 (un  2) 2


Xét un 1  un   u   0, un  3 .
un  un  6 n
un  un  6
Do đó (un ) là dãy tăng và 3  u1  u2   un  .

a 1  a  4
Giả sử (un ) bị chặn trên, suy ra lim un  a , a  3 . K đó ta có a   a  2  3 (vô lí), suy ra
n  a  a  6
(un ) không bị chặn trên. Vậy lim un   .
n 

1 1 1 1 1 1
Từ (*) suy ra    , hay    .
un 1  2 un  2 un  4 un  4 un  2 un1  2
n
1 n
 1 1  1
vn         1 .
i 1 ui
2014
 4 i 1  ui  2 ui 1  2  un1  2
1
Vậy lim vn  li m (1  ) 1 .
n  n  un1  2
u1  3
Bài 17. Cho dãy số  un  được xác định bởi  3 . Chứng minh rằng dãy  un 
un 1  3un 1  2  un , n  1
có giới hạn hữu hạn và tìm giới hạn đó.

Hướng dẫn giải

u1  3
Dãy số  un  được xác định bởi  3 .
un 1  3un 1  2  un , n  1

Ta chứng minh un  2, n  1 .

Thật vậy ta có u1  3  2 .

Giả sử uk  2, k  1 , k đó uk31  3uk 1  2  uk  2  2  2 nên.

uk31  3uk 1  2  0   uk 1  1  uk 1  2   0  uk 1  2 .
2

Do đó t eo nguy n lý quy nạp thì un  2, n  1 .

Xét hàm số f  t   t 3  3t trên khoảng  2,    .

Ta có f '  t   3t 2  3  0, t  2 .

Do đó àm số f  t  đồng biến trên khoảng  2,    .

Mặt khác ta có u13  3u1  18  5  u23  3u2  f  u1   f  u2   u1  u2 .

Giả sử uk  uk 1  k  1  2  uk  2  uk 1  uk31  3uk 1  uk3 2  3uk  2 .

 f  uk 1   f  uk  2   uk 1  uk  2 .

Do đó un  un 1 , n  1  Dãy  un  là dãy giảm và bị chặn dưới bởi 2 nên dãy  un  có giới hạn hữu
hạn.

Giả sử lim un  a  a  2  . Từ hệ thức truy hồi un31  3un 1  2  un chuy n qua giới hạn ta được:.

   2  a   a  2   a 5  2a 4  2a 3  4a 2  a  1  0 .
2
a 3  3a  2  a  a 3  3a

 
  a  2  a 2  a 3  4   2a 3  a  1  a  1  0  a  2  a  2  .

Vậy lim un  2 .

 xn  thỏa   n  N 
2
Bài 18. Cho dãy số mãn: x1  2015 và xn 1  xn . xn  1 *
(*).
n
1
Tìm: lim  .
i 1 xi  1

Hướng dẫn giải


* Ta có: xn  0 n  N * .
xn 1
 
xn  1  0 n  N *   xn  là dãy số tăng.
2
Và: 
xn

* Đặt un  xn .

 u n xác định vì xn  0 n  N * và un  0 n  N * .

 un 1  xn 1  xn 1  un21 .

Nên từ giả thiết (*) ta có:.

un21  un2 .  un  1   un .  un  1  .
2 2

 un 1  un2  un n  N * (1).

* Xét dãy số  un  ta có:.

. un 1  un  un2  0 n  N *   un  tăng.

. Giả sử  un  có giới hạn là a . Từ (1) ta có:.

a  a 2  a  a  0 (loại).

  un  tăng và k ông bị chặn  lim un   .

* Ta có:.

1 un2 u u u u 1 1
  2n 1 n  n 1 n   .
un  1  un  1 un  un  un  .un
2
un 1.un un un 1

n
1 1 1
  u 1  u
i11

un 1
.
i 1

n
1 1 1  1
 lim   lim    .
i 1 ui  1  u1 un1  2015
n
1 1
Vậy: lim   ..
i 1 xi  1 2015

u1  5
Bài 19. Cho dãy số un  ; (n = 1; 2;.) được xác định bởi:  . Chứng minh dãy số un  có
un 1  un  12
giới hạn. Tìm giới hạn đó.

Hướng dẫn giải


Dự doán giới hạn của dãy số,bằng cách giả p ương trìn :.

a  0
a  a  12   2 a4.
 a  a  12

Nhận xét u1  5 .

u2  u1  12  17  u1 .
u3  u2  12  17  12  u2 ... .

Ta dự đoán dãy số un  là dãy số giảm và bị chặn dưới bởi 4 tức là un  4 .

Chứng minh dãy số u n bị chặn: tức là un  4 .

khi n  1, u1  5  4 vậy n  1 đúng.

Giả sử uk  4 , ta chứng minh: uk 1  4 .

Thật vậy ta có:.

uk 1  uk  12  0  uk21  uk  12  uk21  12  uk  4  uk21  16  uk 1  4 .

Vậy dãy số u n bị chặn dưới.

Ta chứng minh dãy số un  là dãy số giảm.

Ta có:.

un2  un  12 (un  4)(un  3)


un 1  un  un  12  un   un 1  un   0 (vì un  4 ).
un  12  un un  12  un

Vậy dãy số un  giảm và bị chặn dưới nên có giới hạn.

Đặt lim un  a thì lim un 1  a. .

Ta có:.

un 1  un  12  lim un 1  lim un  12  a  a  12  a  4 .

Vậy lim un  4. .

Bài 20. Cho dãy số  xn  được xác định bởi.


 x1  2,1

 xn  2  xn2  8 xn  4 .
 xn 1  * , n  1, 2,...
 2
n
1
Với mỗi số nguy n dương n, đặt yn   2 . Tìm lim yn .
i 1 xi  4

Hướng dẫn giải


Ta có kết quả sau: vớ số t ực a  2 bất kì, ta có.

a2 a  8a  4 a2 a  4a  4 a  2   a  2
2 2

  a.
2 2 2

Do đó 2,1  x1  x2  ...   xn  là dãy tăng, g ả sử bị c ặn tr n tức là có g ớ ạn lim xn  L  2 .

C uy n qua g ớ ạn đ ều k ện (*) ta có p ương trìn .

x2 x  8x  4
2

x  x  4   x  3  x  2  .
2

2
p ương trìn này k ông có ng ệm ữu ạn lớn ơn 2.
Suy ra dãy  xn  tăng và k ông bị c ặn tr n n n lim xn   .

xn  2  xn  8 x n  4
2

Ta có xn 1   2 xn 1  xn  2  xn  8 x n  4 .
2

  2 xn 1  xn  2   xn  8 xn  4  xn  2  4   xn  3  xn  2  .
2 2 2

1 xn  3 xn  2  1 1 1
     .
xn  2 xn 1  4 xn 1  4 xn 1  2 xn 1  4
2 2 2

1 1 1
   .
xn 1  4 xn  2 xn 1  2
2

n
1 1 1 1
Suy ra yn      10  .
xi  4 x1  2 xn 1  2 xn 1  2
2
i 1

Vậy lim yn  10 .

Bài 21. Cho dãy số  xn  được xác định bởi x1  2016, xn 1  xn2  xn  1, n  1, 2,3,... .

a)C ứng m n rằng  xn  tăng và lim xn   .

1 1 1
b)Vớ mỗ số nguy n dương n , đặt yn  2016    ...   . Tính lim yn . .
 x1 x2 xn 

Hướng dẫn giải

xn 1  xn  xn2  2 xn  1   xn  1  0  xn 1  xn , n  1.  xn 
2

a)Ta có Do đó tăng.
Ta c ứng m n bằng quy nạp t eo n rằng xn  n  1, n  1 (1).

T ật vậy, (1) đúng vớ n  1 .G ả sử (1) đúng vớ n ( n  1) thì.

xn 1  xn  xn  1  1  n  n  1  1  n 2  n  1  n  2 .

Vậy (1) đúng vớ mọ n. Từ  xn  tăng ngặt và xn  n  1, n  1 suy ra lim xn  . .

1 1 1 1
b)Ta có xn 1  1  xn  xn  1 . Suy ra    .
xn 1  1 xn  xn  1 xn  1 xn

1 1 1
Từ đó  
xn xn  1 xn 1  1 .

1 1 1  1 1   1 1 
 yn  2016    ...    2016     2016   
 x1 x2 xn   x1  1 xn 1  1   2015 xn 1  1  .

1 2016
Từ lim xn    lim  0 . Vậy lim yn  ..
xn 2015

1 1 1 a 
Cho dãy  an n 1 : an  sin1  2 sin  32 sin  ...  n2 sin

Bài 22. 2
n  1 . Chứng minh dãy  n2 
2 3 n  n n 1
a
hội tụ và tính lim n2 .
n

Hướng dẫn giải


1
Bổ đề 1: x  sin x  x  x3x  0 .
6
1 1 1
1    ... 
Bổ đề 2: lim 2 3 n  0.
n
1 1 1 1 1 1
Đặt xn  n 2 sin . Áp dụng bổ đề 1:  sin   3  k  xk  k  .
n k k k 6k 6k
1 1 1
 1  2  ...  n  an  1  2  ...  n  1   ...   .
6 2 n

1 1
1  ... 
1 an 1 2 n.
C a các vế c o n 2 :   
2 n2 2 6n 2

an 1
Cho n   , và lấy g ớ ạn, suy ra lim  .
n2 2 .

 n  1
2
un
Bài 23. Cho dãy số u1  2, un 1  n  1 . Tính giới hạn lim .
un  1 n  n

Hướng dẫn giải

n2
Ta chứng minh quy nạp  un  n  1 , n  1 .
n 1
Rõ ràng khẳng địn đã đúng với u1 .

 k  1  u  k  2 .
2
k2
Giả sử đã có  uk  k  1, k  1. Ta chứng minh k 1
k 1 k 2

(k  1)2  k  1
2

Thật vậy: uk  k  1  uk 1   .
uk  1 k 2

(k  1) 2  k  1
2
k2 1
uk   uk 1   2  k 2 2  k  2..
k 1 uk  1 k k  k 1
1
k 1
n2 u
Vậy ta có  un  n  1, n  1  lim n  1 .
n 1 n  n
x 1  
Cho   2 và dãy số  xn 

Bài 24. với:  n 3 .
2x n 1  3x n 
2
n  N 
*

 n

a) C ứng m n : x n  1 vớ n  N * .

b) C ứng m n dãy số x n  có g ớ ạn và tìm g ớ ạn đó.

Hướng dẫn giải


Ta chứng minh x n  1 với n  N * bằng quy nạp.

Ta có: x 1   nên x 1  1 .

G ả sử: x k  1 vớ k  N * .

n 1 n3
Ta có: 3x 2k  3 và  1 nên 3x 2n   2 . Suyra: x n 1  1 .
n n
Vậy x n  1 vớ n  N * .

Ta chứng minh  xn  là dãy giảm bằng quy nạp.

Vì   2 nên 3 2  4  2 .Ta có x 2  x 1 .

n 1
G ả sử: x k 1  x k . Ta có: 3 x 2k 1  3x 2k và f n  = là àm ng ịc b ến n n:.
n
k4 k3
3x 2k 1   3x 2k  .
k 1 k

Suy ra: x k  2  x k 1 . Vậy  xn  là dãy g ảm.

xn  lả dãy g ảm và bị c ặn dướ bở 1 n n ộ tụ.

 x1  1

Đặt lim x n   .Ta có 2  3  1    1 .  xn  3xn  4 (n  N )  un  un  x2 n 1  n  N 
2 * *
 .
x
 n 1 
 xn  1

Vậy lim x n  1 .

u1  2011
Bài 25. Cho dãy số  un  được xác định:  n .
2 un 1  2 un  1 , n  N
n *

C ứng m n rằng dãy số  un  có g ớ ạn ữu ạn và tín g ớ ạn đó.

Hướng dẫn giải


1
Ta có 2 n un 1  2 n.un  1  un 1  un  .
2n

C ứng m n : un  21– n (bằng quy nạp).

*vớ n  1 ta có u1  2011  20 .
*G ả sử uk  21– k (vớ k  1 ).

*Cần c ứng m n : uk 1  2 – k .

Ta có uk 1  uk  2  k  21k  2  k  2  k . Suy ra đ ều p ả c ứng m n .

1
Từ đó ta có un – 2 – n  0 vớ mọ n  un 1  un  .
2n
1 1 1 1
Ta có u2  u1  ; u3  u2  2 ; u4  u3  3 ;...; un  un 1  n 1 .
2 2 2 2

1 1 1 1 
 un  u1    2  3  ...  n 1  .
2 2 2 2 
n 1
1
1   n 1

Công t ức tổng quát : un  2011  .  


1 2 1
 2011  1    .
2 1 2
2
Vậy lim un  2010 .

u1  a

Bài 26. Cho số thực a   0;1 , xét dãy số  un  với:  1 2013 
.
u   u 2
 u , n  

n 1 n n
2014 2014

a) C ứng m n rằng: 0  un  1, n    .

b) C ứng m n rằng  un  có g ớ ạn ữu ạn. Tìm g ớ ạn đó.

Hướng dẫn giải


a) Chứng minh: 0  un  1, n    1 .

n  1: u1  a   0;1  1 đúng vớ n=1.

1 1
G ả sử 0  uk  1 vớ k  1, k   . Ta có: 0  uk2  1  0  uk2  .
2014 2014
2013 2013
0  uk  1  0  uk  .
2014 2014
1 2013
0 uk2  uk  1  0  uk 1  1 .
2014 2014
Vậy: 0  un  1, n    .

b) Chứng minh rằng  un  có giới hạn hữu hạn. Tìm giới hạn đó.

Ta c ứng m n :  un  là dãy tăng.

n    , un 1  un 
1
2014
un2 
2013
2014
un  un 
1 
2014 

un  un u n 
 un  2013   0 .

 un 1  un , n    hay  un  là dãy tăng.(2).

Từ (1),(2) suy ra  un  có g ớ ạn ữu ạn.G ả sử  un  có g ớ ạn là a,  o  a  1 .

1 2013
Ta có: a  a2  a  a  1 . Vậy lim un  1 .
2014 2014

 3
u1  2
Bài 27. Cho dãy số(un) xác địn n ư sau:  .
u  1 u 3  2 , n  N 
 n 1 3 n 3

a) C ứng m n rằng: 1  un  2, n    .

b) C ứng m n rằng  un  có g ớ ạn ữu ạn. Tìm g ớ ạn đó.

Hướng dẫn giải


3
a) Vớ : n  1: u1   1 đúng vớ n=1.
2
G ả sử: 1  uk  2 vớ k  1, k   .

Ta có: uk 1  2  uk3    uk  2   uk2  2uk  4   0  uk 1  2 .


1 8 1
3 3 3

uk 1  1 
3
 uk  1  0  uk 1  1 .
1 3

 1  uk 1  2 . Vậy: 1  un  2, n    .

1
b) n    , un 1  un   un  1  un  2   0  un1  un , n    hay  un  là dãy g ảm (2).
2

Từ (1),(2) suy ra  un  có g ớ ạn ữu ạn.

Gọ a là g ớ ạn của  un  , 1  a  2 .

1 2
Ta có a  a 3   a  1 . Vậy lim un  1 .
3 3

un2
Bài 28. Cho dãy số  un  xác định bởi: u1  1; un 1   un , n  N * . Tìm giới hạn sau:
2015
u u u 
lim  1  2  ...  n  .
n 
 u2 u3 un 1 

Hướng dẫn giải

un2 u 1 1 
Từ đề bà ta có: un 1  un  . Suy ra: n  2015   .
2015 un 1  un un 1 

u1 u2 u 1 1   1 
Ta có:   ...  k  2015     2015 1   .
u2 u3 uk 1  u1 uk 1   uk 1 
Ta có  un  là dãy đơn đ ệu tăng và u1  1 .

2
Nếu lim un   thì       0 .
n  2015
( vô lí vì  un  là dãy đơn đ ệu tăng và u1  1 ).

Suy ra: lim un   .


n 

u u u 
Kết luận: lim  1  2  ...  n   2015 .
n  u
 2 u3 un 1 

u1  2013
Bài 29. Cho dãy số  un  xác định bởi  2
  
 n  N *  . Chứng minh rằng dãy (un) có
 n
u 2u .u
n n 1 2013 0
giới hạn và tính giới hạn đó.

Hướng dẫn giải


Từ ệ t ức truy ồ suy ra 2un .un 1  un 2  2013 .

Bằng quy nạp c ứng m n được u n > 0, vớ mọ n.

Do đó ta có:.

un12  2013 1  2013  2013


un    un 1    un .  2013, n  1 .
2un1 2 un1  un

Mặt k ác ta có :.

un 1 un 2  2013 1 2013 1 1
      1.
un 2un 2 2 2un 2 2 2

(un) là dãy số g ảm và bị c ặn dướ bở 2013 , do đó (un) có g ớ ạn ữu ạn.


Đặt lim un  a .

a 2  2013
Ta có : a   a  2013 . Vậy lim un  2013 .
2a

xn4  9
Bài 30. Cho dãy số  xn  xác định bởi: x1  4, xn 1  3 , n  *
.
xn  xn  6

a) C ứng m n rằng lim xn   ;.


n 

n
1
b) Vớ mỗ số nguy n dương n , đặt yn   . Tính lim yn .
k 1 x 3
3
k

Hướng dẫn giải

xn4  9  xn  3  xn3  3
a) Xét xn 1  3  3   * .
xn  xn  6  xn3  3   xn  3

Bằng quy nạp c ứng m n được xn  3, n  1 .


xn4  9 xn2  6 xn  9
Xét xn 1  xn   x  .
xn3  xn  6 xn3  xn  6
n

 xn  3
2

 xn 1  xn   0, n  *
.
x  xn  6
3
n

Do đó  xn  là dãy tăng và 4  x1  x2  x3  ... .

G ả sử  xn  bị c ặn tr n  lim xn  a .

a4  9
Do đó: a   a  3  4 (vô lý). Suy ra  xn  k ông bị c ặn tr n. Vậy lim xn   .
a3  a  6
1 1 1 1 1 1
b) Từ (*), suy ra:   3  3   .
xn 1  3 xn  3 xn  3 xn  3 xn  3 xn 1  3
n
1 n
 1 1  1
Suy ra: yn         1 .
k 1 xk  3 k 1  xk  3 xk 1  3 
3
xn 1  3

 1 
Vậy lim yn  lim 1    1.
 xn 1  3 

 x1  1
 x12014 x22014 xn2014
Bài 31. Cho dãy số  2015
xn . Tìm giới hạn của dãy số u n với u n    ...  .
 n 1
x   x x2 x3 x n 1

n
2015

Hướng dẫn giải

xn2015 x 2015 x x xn2015


xn 1   xn  xn 1  xn  n  n 1 n 
2015 2015 xn 1 xn 2015 xn1 xn
.

1 1 xn2014 1 1  xn2014
    2015   
xn xn 1 2015 xn 1  xn xn 1  xn 1
.

 1 
Từ đó un  2015 1  .
 xn 1 

Dễ t ấy  xn  là dãy tăng và 1  x1  x2  x3  ... .

G ả sử  xn  bị c ặn tr n  lim xn  a .

a 2015
Do đó: a   a  a  0  1 (vô lý). Suy ra  xn  k ông bị c ặn tr n. Vậy lim xn   .
2015

 1 
Vậy limu n  lim 2015 1    2015 .
 xn 1 
 x1  1

Bài 32. Cho dãy số {xn } xác định bởi  xn2 . Tìm giới hạn của dãy ( S n ) với
 n 1
x  x 

n
2015
x x x
Sn  1  2  ...  n .
x2 x3 xn1

Hướng dẫn giải

xn2 xn 1  xn xn2 x 1 1 
xn 1  xn   2015  xn 1  xn   xn  2015
2
  n  2015   . .
2015 xn 1 xn xn 1 xn xn 1  xn xn 1 

x1 x2 x 1 1   1 
Suy ra: Sn    ...  n  2015     2015 1  .
x2 x3 xn 1  x1 xn 1   xn 1 

Dễ t ấy  xn  là dãy tăng và 1  x1  x2  x3  ... .

G ả sử  xn  bị c ặn tr n  lim xn  a .

a2
Do đó: a   a  a  0  1 (vô lý). Suy ra  xn  k ông bị c ặn tr n. Vậy lim xn   .
2015

 1 
Vậy limSn  lim 2015 1    2015 .
 xn 1 

 x1  1 n
1
Bài 33. Cho dãy số ( xn ) xác định bởi  . Đặt Sn   .
 xn 1  xn ( xn  1)( xn  2)( xn  3)  1 k 1 xk  2

Tìm limSn .

Hướng dẫn giải

xn 1  xn ( xn  1)( xn  2)( xn  3)  1  ( xn 2  3xn )( xn 2  3xn  2)  1  xn2  3xn  1


.
n
1 1 1 1 1 1 1 1
Ta có  
xn  2 xn  1 xn 1  1
 S n  
k 1 xk  2
   
x1  1 xn 1  1 2 xn 1  1
.

Dễ t ấy: xn 1  xn   xn  1  0, n  N * suy ra  xn  là dãy tăng và 1  x1  x2  x3  ... .


2

G ả sử  xn  bị c ặn tr n  lim xn  a .

Do đó: a  a 2  3a  1  a  1  1 (vô lý). Suy ra  xn  k ông bị c ặn tr n. Vậy lim xn   .

1 1  1
Vậy limSn  lim    .
 2 xn 1  1  2

 2016
u1  1 1 1
Bài 34. Cho dãy số (un) xác định bởi:  2015 . Đặt Sn   . . .
2u  u 2  2u , n  * u1  2 u2  2 un  2
 n 1 n n

Tính: limSn.
Hướng dẫn giải
un  un  2  1 1 1 1 1 1
2un 1  un  un  2   un 1       
2 un 1 un un  2 un  2 un un 1 .

n
1 1 1 2015 1
 Sn       .
k 1 uk  2 u1 un 1 2016 un 1

Bằng quy nạp ta dễ dàng c ứng m n được un  0, n  N * .

1 2016
K đó: un 1  un  un 2  0, n  N * suy ra  un  là dãy tăng và  u1  u2  u3  ... .
2 2015

G ả sử  un  bị c ặn tr n  limu n  a .

2016
Do đó: 2a  a 2  2a  a  0  (vô lý). Suy ra  un  k ông bị c ặn tr n.
2015
Vậy limu n   .

 2015 1  2015
Vậy limSn  lim    .
 2016 un 1  2016

xn4  9
Bài 35. Cho dãy số  xn  xác định bởi: x1  4, xn 1  , n  *
.
xn3  xn  6

a) C ứng m n rằng lim xn   ;.


n 

n
1
b) Vớ mỗ số nguy n dương n , đặt yn   . Tính lim yn .
k 1 x 3
3
k

Hướng dẫn giải

xn4  9  xn  3  xn3  3
a) Xét xn 1  3  3   * .
xn  xn  6  xn3  3   xn  3

Bằng quy nạp c ứng m n được xn  3, n  1 .

xn4  9 xn2  6 xn  9
Xét xn 1  xn   x  .
xn3  xn  6 xn3  xn  6
n

 xn  3
2

 xn 1  xn   0, n  *
.
x  xn  6
3
n

Do đó  xn  là dãy tăng và 4  x1  x2  x3  ... .

G ả sử  xn  bị c ặn tr n  lim xn  a .

a4  9
Do đó: a   a  3  4 (vô lý). Suy ra  xn  k ông bị c ặn tr n. Vậy lim xn   .
a3  a  6
1 1 1 1 1 1
b) Từ (*), suy ra:   3  3   .
xn 1  3 xn  3 xn  3 xn  3 xn  3 xn 1  3
n
1 n
 1 1  1
Suy ra: yn         1 .
k 1 xk  3 k 1  xk  3 xk 1  3 
3
xn 1  3

 1 
Vậy lim yn  lim 1    1.
 xn 1  3 
3.2. TÍNH GIỚI HẠN BẰNG CÁC CÔNG THỨC CƠ BẢN

u1  1

Bài 1. Cho dãy số  an  thỏa mãn  1 . Tìm tất cả các số thực a sao cho dãy
u  un 
 n 1 3 u
n  *
 n

una
số  xn  xác định bởi xn  ( n  * ) hội tụ và giới hạn của nó khác 0.
n
Hướng dẫn giải
Từ giả thiết ta có dãy số  un  là dãy số dương và tăng(1).

1
Giả sử  un  bị chặn trên suy ra nó hội tụ. Đặt L  lim un , ta có ngay L  L  3
(vô lý).
L

Vì vậy  un  không bị chặn trên (2).

Từ (1) và (2) ta có lim un   .

 43 4
 1
Xét lim  un 1  un3  . Đặt vn  4 ( n  * ), ta có lim vn  0 .
  un3
4
 1 
3 4
4
3
4
3  1 
un 1  un  3  vn  
4
1 1  vn  3 1

vn3  4vn2  6vn  4
.
  v 8 4
 v4
 n


n vn
1  vn  3  1  vn  3  1
4
 4 4
 4 u3 4
Suy ra lim  un31  un3   . Từ đó lim n  (sử dụng trung bình Cesaro).
  3 n 3

 4
 khi a 
3
 34
 
una  un a  43   4
Ta có lim  lim  .un   0 khi a  .
n  n   3
  4 4
3 khi a 
 3
4
Vậy a  là giá trị cần tìm.
3

 1
u1  2 ; u2  3
Bài 2. Cho dãy số  un  xác định như sau: 
u .u  1
un  2  n 1 n , n  N *
 un 1  un
a) Chứng minh rằng tồn tại vô số giá trị nguyên dương của n để un  1 .

b) Chứng minh rằng  un  có giới hạn hữu hạn, tìm giới hạn đó.

Hướng dẫn giải


a) Trước hết ta luôn có un  0, n  N * . Xét un  2  1 
 un1  1 un  1 (1).
un 1  un

Bằng phương pháp quy nạp ta chứng minh được u3n , u3n 1  1, n  N * và u3n  2  1, n  N * .

Từ đó suy ra điều phải chứng minh.


 u  1 un  1 (2).
b) Ta có un  2  1  n 1
un 1  un

un  2  1 un 1  1 un  1
Chia vế của (1) cho (2) có  . , n  N * .
un  2  1 un 1  1 un  1

un  1
Đặt vn  n  N * , ta có vn  2  vn 1.vnn  N * .
un  1

Bằng phương pháp quy nạp ta chứng minh được vn  v2Fn1 .v1Fn2 , với  Fn  là dãy số Phibonxi:
 F1  F2  1
 .
 Fn  2  Fn 1  Fn , n  N
*

Fn1 Fn2
1  1
Hay vn    .    0 khi n   , dẫn đến lim un  1 .
2  3

Bài 3. Cho dãy số  un  được xác định như sau.

u1  1
 .
un 1  un  un  2  un  4  un  6   16, n 
*

n
1
Đặt vn   , hãy tính lim vn .
i 1 ui  5

Hướng dẫn giải


Dễ thấy un  0, n  *
.

Theo bài ra ta có.

u  6un  un2  6un  8   16  u  6un  4   un2  6un  4 .


2
un 1  2
n
2
n

1 1 1
Suy ra un 1  1   un  1 un  5     .
un 1  1 un  1 un  5
n
1 n
 1 1  1 1 1 1
Do đó vn          .
i 1 ui  5 i 1  ui  1 ui 1  1  u1  1 un 1  1 2 un 1  1
Mặt khác, từ un 1  un2  6un  4 ta suy ra un 1  6un .
Kết hợp với u1  1 ta có.
1
un  6n 1 , n  *
 lim un    lim 0.
un 1  1
1 1  1
Từ đó ta có lim vn  lim    .
 2 un 1  1  2

Bài 4. Cho dãy số thực  un  với n  *


thỏa mãn ln 1  un2   nun  1, n  *
.

n 1  nun 
Tìm lim .
n  un

Hướng dẫn giải

Với mỗi n  *
, đặt f n  x   ln 1  x 2   nx  1, x  .

 x  1  n  1  0 .
2
2x
Ta có f  x  
'
n
1 x 1  x2
n 2

 x  1
f n'  x   0   .
n  1

Do đó f n  x  là hàm tăng thực sự trên .

 f n  0   1  0

Ta có   1   1  .
 nn
f  ln  1  2 
 0
    n 
1
Do đó !un  sao cho f n  un   0 và 0  un  .
n
Ta thấy lim un  0 .
n 

 1

 lim
Do đó:  n
ln  n  n 1
1  u 2 u2

.
 lim nun  lim 1  ln 1  un2    1
 n n   

n 1  nun  n ln 1  un2   1

Vậy lim  lim  lim  nun ln 1  un2  un2  1. .
n  n  n 
un un  

 4
a1  3
Bài 5. Cho dãy số  an  thỏa mãn:  n  1, n  .
 n  2 2 a  n 2 a   n  1 a a
 n n 1 n n 1

Tìm lim an .

Hướng dẫn giải

 n  2
2
n2
Dễ thấy an  0, n  *
. Từ giả thiết ta có    n  1 .
an 1 an

1 1
Với mỗi n  *
, đặt yn   ta có y1  1 và.
an 4
2 1 2 1 n2
    n1 
    
   
   n1   
2 2
n 2 y n  n
y n 1 n 2 y n y y y .
 n  2
n n 1 2 n
 4  4

4n 2  n  1
2
 n 1   n  2   1 
2 2 2
4
Do đó yn      ...   y1   an  .
 n 1   n 1   3   n  1 n 2 16  n 2  n  1
2 2

Vậy lim an  4 .

Bài 6. Tính các giới hạn sau:

x3  8 2x 1
a) lim . b) lim .
x 2 x 2  4 x 2 x2
Hướng dẫn giải

a).lim 2
x3  8
 lim
 x  2x  4  3 .
2

x 2 x  4 x 2  x  2
2x 1
b) lim   .
x 2 x2

x  x 2  ...  x n  n
Bài 7. Tính giới hạn lim .
x 1 x 1
Hướng dẫn giải

x  x 2  ...  x n  n ( x  1)  ( x 2  1)  ...  ( x n  1)
lim  lim
x 1 x 1 x 1 x 1
( x  1)[1  ( x  1)  ( x  x  1)  ...  ( x n1  ...  1)]
2
lim .
x 1 x 1

lim 1  ( x  1)  ( x 2  x  1)  ...  ( x n 1  ...  1)  .


x 1

n(n  1)
 1  2  3   n 
2 .
n
1  ax  1 a
Bài 8. Cho n là số nguyên dương và a  0 .Chứng minh rằng: Lim  ..
x 0 x n
Hướng dẫn giải

Đặt y  n 1  ax , khi đó từ x  0  y  1. .
n
1  ax y 1 y 1 a
Vậy Lim  aLim n  a Lim  ...  . .
x 0 x y 1 y 1 y 1
 y  1  y  y  ...  y  
n 1 n2
n

Bài 9. Tính các giới hạn sau:.


1
13  53  93  ...  (4n  3)3  cos 5 x  x sin x
a/ lim b/ lim   .
1  5  9  ...  (4n  3)  
n  2 x  0 cos 3 x

Hướng dẫn giải


Câu a.
n n
13  53  93  ...  (4n  3)3   (4i  3)3   (64i 3  144i 2  108i  27) .
i 1 i 1

n n n
= 64 i 3  144 i 2  108 i  27 n .
i 1 i 1 i 1

n(4n  2)
1  5  9  ...  (4n  3)   2n 2  n .
2

n(n  1) n 2 n(n  1)(2n  1) n 3  n(n  1) 


n 2

Mà ta có các công thức:  i  ; i  ; i   .


i 1 2 i 1 6 i 1  2 

Do đó: P( x)  13  53  93  ...  (4n  3)3 là một đa thức bậc 4 có hệ số bậc 4 là 64 / 4  16 .

Và Q( x)  1  5  9  ...  (4n  3) là một đa thức bậc 4 có hệ số bậc 4 là 4 .


2

13  53  93  ...  (4n  3)3 16


Do đó: lim   4.
1  5  9  ...  (4n  3)
n  2
4

Câu b.
cos5 x  cos3 x
1
 cos3 x
 x sin x .cos3 x
 cos 5 x   cos 5 x  cos 3 x  cos5 x  cos3 x
= lim 1  
x sin x
lim    .
x 0  

x  0 cos 3 x
  cos 3x 
 

cos 5 x  cos 3x 2sin 4 x sin x  sin 4 x sin x 8 


Vì lim  lim  lim  . .  8 .
x 0 x sin x.cos 3x x  0 x sin x.cos 3x x  0
 4x x cos 3x 
1
cos 5 x  cos 3x 1
 cos 5 x  x sin x
Vì lim  0 và áp dụng công thức lim 1  u  u  e , nên lim    e 8 .
x 0 cos 3x u 0

x  0 cos 3 x

 x1  2

Bài 10. Cho dãy số  xn  thỏa mãn  x1  2 x2  3x3  ...  (n  1) xn 1 . Tìm lim un với
 x  , n  1, n  .
n(n  1)
n 2

un  (n  1) 3 xn . .

Hướng dẫn giải


1
Ta có x2  .
3

Với n  3 : x1  2 x2  3x3  ...  nxn  n3 xn (1).

x1  2 x2  3x3  ...  (n  1) xn 1  (n  1)3 xn 1 (2).

Từ (1) và (2) ta có nxn  n3 xn  (n  1)3 xn 1 .

(n  1)3 xn 1 n 1 2 n
Suy ra xn  ( ). .xn 1 .
n n
3
n n 1
n 1 2 n  2 2 2 2 n n 1 3
 xn  ( ) .( ) ...( ) . . ... x2 .
n n 1 3 n 1 n 4
4 4(n  1) 2
 xn  suy ra lim u = lim  4.
n 2 (n  1)
n
n2

3x  1. 3 2  x  2
Bài 11. Tính giới hạn hàm số : L  lim .
x 1 x 1
Hướng dẫn giải
Ta có:.

3x  1. 3 2  x  2 3x  1. 3 2  x  3x  1  3x  1  2
lim  lim .
x 1 x 1 x 1 x 1
3
2  x 1 3x  1  2
= lim 3x  1  lim .
x 1 x 1 x 1 x 1

( 3 2  x  1)  3 (2  x) 2  3 2  x  1
= lim 3x  1    lim ( 3x  1  2)( 3x  1  2) .
x 1
( x  1)  3 (2  x) 2  3 2  x  1 x 1 ( x  1)( 3 x  1  2)
 
(2  x  1) (3 x  1  4)
= lim 3 x  1  lim .
x 1
( x  1)  3 (2  x) 2  3 2  x  1 x 1 ( x  1)( 3 x  1  2)
 

( 3 x  1) 3 1
= lim  lim = .
x 1  3 (2  x)  3 2  x  1
2 x 1 ( 3 x  1  2) 12
 

x 2  3  2011x  2009
Lim
Bài 12. Tính: x 1 x 1 .
Hướng dẫn giải
x  3  2  2011( x  1)
2
x 3 4 2
lim  lim[  2011]
x 1 x 1 x 1 ( x  1)( x  3  2) .
x 1 4021
 lim(  2011)  
x 1 x3 2 2

 4
a1  3
Bài 13. Cho dãy số  an  thỏa mãn:  n  1, n  . Tìm lim an .
 n  2 2 a  n 2 a   n  1 a a
 n n 1 n n 1

Hướng dẫn giải


 n  2
2
n2
Dễ thấy an  0, n  *
. Từ giả thiết ta có    n  1 .
an 1 an

1 1
Với mỗi n  *
, đặt yn   ta có y1  1 và.
an 4
 1 2 1 n2
 n  2              
2 2 2
 n 1
y  n  n
y  n 1 n 2 y n y y y .
 n  2
n 1 n n 1 2 n
 4  4
4n 2  n  1
2
 n 1   n  2   1 
2 2 2
4
Do đó yn        1
... y   a  .
 n 1   n 1   3   n  1 n 2 16  n 2  n  1
2 n 2

Vậy lim an  4 .

1 a
Bài 14. Cho dãy số  xn  thỏa mãn x1  0, xn  (3xn 1  3 ), n  2,3,... .
4 xn 1

Hướng dẫn giải


1 a
Ta có xn  ( xn 1  xn 1  xn 1  3 )  4 a với mọi n  2 .
4 xn 1

Do đó dãy  xn  bị chặn dưới.

xn 3 a 3 1
Với mọi n  3 , ta có   4    1  xn  xn –1 .
xn 1 4 4 xn 1 4 4

Do đó  xn  là dãy giảm.

Từ đó suy ra dãy  xn  có giới hạn và dễ dàng tìm được lim xn  4 a .

 x1  3

Bài 15. Cho dãy số thực  xn  :  1 . Xét dãy số  yn  cho bởi :
 xn 1  3  , n  1, 2,3,...
 xn

(3  5) n
yn  n ; n  1, 2,3,... Chứng minh dãy số  yn  có giới hạn hữu hạn và tính giớn hạn đó.
2 .x1.x2 .x3 ...xn

Hướng dẫn giải


1
 Ta có : xn 1  3   xn .xn 1  3xn  1 ; n  1, 2,3,... .
xn

 Đặt : zn  x1.x2 .x3 ...xn thì ta có zn  2  x1.x2 .x3 ...xn .xn 1.xn  2 .

 zn .xn 1.xn  2 .

 zn .(3 xn 1  1) .

 3 zn xn 1  zn .

 3zn 1  zn .

 z1  x1  3

 8
Khi đó :  z2  x1.x2  3.  8 . Suy ra  zn  là dãy truy hồi tuyến tính cấp 2.
 3
 zn  2  3 zn 1  zn ; n  1, 2,3,...

3 5
Xét phương trình đặc trưng : t 2  3t  1  0  t  .
2
n n
 3 5   3 5 
Dãy có số hạng tổng quát dạng zn         .
 2   2 

 3  5   3 5 
        3  53 5
 
 2   2   10
trong đó :   .
 7  3 5   73 5    5  3 5
 2     2    8  10
   
 Lúc này, ta có.
n n
 3 5   3 5 
   
(3  5) n  2   2  1
yn  n   n n
 n
.
2 .x1.x2 .x3 ...xn zn  3 5   3 5   3 5 
      
 2   2   3 5 

1 1 1 3 5 5
Suy ra : lim yn     .
 3 5 
n
 53 5 2
 .lim    10
 3 5 

3 5 5
 Vậy : yn  khi n   .
2
un
Bài 16. Cho dãy số  un  xác định bởi: u0  1 , un 1  n  . Tìm lim n3un  ? .
n un  un2  1
2 n 

Hướng dẫn giải


n
un un 1
Từ giả thiết un 1  n  ta có un 1   2 n  *
nên  vn  xác định bởi vn   uk có
n un  un2  1
2 2
n un n k 0

giới hạn hữu hạn, giả sử lim vn  c ( c hữu hạn).


n 

un 1 1
Cũng từ un 1  n  ta có  n 2  un  n  .
n un  un2  1
2
un 1 un

1 1
   n 2  un n  .
un 1 un

1 1
Do đó   02  u0 .
u1 u0

1 1
  12  u1 .
u2 u1

….
1 1
  (n  1)2  un 1 .
un un 1
1 1 (n  1)n (2n  1) n 1
Cộng theo vế ta được :     uk .
un u0 6 k 0

1 (n  1)n(2n  1) vn 1  1
 3
  .
n un 6n 3 n3

1  vn
Mà lim  0 ( do lim vn  c ) nên.
n  n3 n 

1 (n  1)n(2n  1) 1
 lim 3
 lim 3
 hay lim n3un  3 .
n  n u n  6n 3 n 
n

4
Bài 17. Cho dãy số  xn  xác định bởi : x1  1, xn 1  1  , n  1 . Chứng minh dãy  xn  có giới
1  xn
hạn hữu hạn và tìm giới hạn đó.
Hướng dẫn giải
4 4 4
Ta có x2  1   3; x3  1   2  x1 ; x4  1   x2 .
2 4 3
4
Hàm số f ( x)  1  liên tục và nghịch biến trên [0,+), 1  f ( x)  5 .
1 x
4
Ta có xn 1  1   f ( xn ), n  ( xn ) bị chặn.
1  xn

x1  x3  f ( x1 )  f ( x3 )  x2  x4  f ( x2 )  f ( x4 )  x3  x5  ... .

suy ra dãy ( x2 n 1 ) tăng và dãy ( x2 n ) giảm suy ra ( x2 n 1 ), ( x2 n ) là các dãy hội tụ.

Giả sử lim x2 n  a;lim x2 n 1  b (a, b  1) .

Từ x2 n 1  f ( x2 n )  lim x2 n 1  lim f ( x2 n )  b  f (a ) .

Từ x2 n  2  f ( x2 n 1 )  lim x2 n  2  lim f ( x2 n 1 )  a  f (b) .

 4
b  1  1  a
Giải hệ phương trình   a  b  4  2 . Vậy lim xn  2 .
a  1  4
 1 b
1 x
Bài 18. Cho x1  2014, x2  2013 và xn  2  (1  ) xn 1  n , n  2,3,... Tìm lim xn .
n n n 

Hướng dẫn giải

xn 1  xn (1) n (1) n n
(1) k
Ta có xn  2  xn 1    xn  2  xn 1  ( x2  x1 )   và xn  2  x1   .
n n! n! k 1 k!

(1) k 
(1) k 1
Dãy này rõ ràng hội tụ và có giới hạn là x1    x1  1    x1  1  .
k 1 k! k 0 k! e

1
Từ đó suy ra lim xn  2015  .
n  e
3.3. TÍNH GIỚI HẠN BẰNG ĐỊNH LÍ KẸP
1
Bài 1. Tìm lim .
n n
n!
Hướng dẫn giải
n n
Trước hết ta chứng minh bất đẳng thức : n! > ( ) (*) ( n  N*).
3
1
Bằng phương pháp qui nạp. Thật vậy : với n =1, ta có 1 > (đúng).
3
k k.
Giả sử (*) đúng với n = k tức là : k! > ( ) Ta đi chứng minh (*) đúng với.
3
n = k+1.
k k k  1 k+1 3 k  1 k+1
Ta có (k+1)! = k!(k+1) >( ) (k+1) = ( ) . >( ) .
3 3 1 k 3
(1  )
k
Bất đẳng thức cuối này đúng vì :.
1 k k k (k  1) 1 k (k  1)(k  2)....(k  k  1) 1
(1+ ) =1+ + . 2 +.+ . k =.
k k 2! k k! k
1 1 1 1 2 k 1 1 1 1 1
= 1+1+ (1  ) +.+ (1  )(1  )...(1  ) < 1+1+ +… + <1+1+ +.+ n1 <.
2! k k! k k k 2! n! 2 2
1 1 1
<1+1+ +.+ n1 +.< 1+ = 3.
2 2 1
1
2
n
n n
Vậy (*) đúng với n  k  1 . Do đó n !    , từ đây ta suy ra n
n!  .
3 3

1 3
=> 0 < n
< .
n! n
3
Vì lim = 0.
n n
1
Do đó theo định lý về giới hạn kẹp giữa ta suy ra: lim = 0.
n n
n!
1
Vậy lim(2014  n
) =2014.
n!

 x1  1; x2  2

Cho dãy số  xn   xn 1 
5
thoả mãn  .
 xn  2   ; n 
2 *

 
2
 4 x n

Tính I  lim xn .

Từ giả thiết suy ra mội số hạng của dãy đều dương.


 y1  0; y2  1
Đặt yn  log 2 xn , ta có dãy  .
2 yn  2  5 yn 1  2 yn ; n 
*

 z1  2, z 2  1
 z1  2; z2  1 
Lại đặt yn  zn  2 , ta có dãy  2 z n  2  5 z n 1  z n .
2 zn  2  5 zn 1  zn ; n 
*

1
Tìm được số hạng tổng quát của dãy là zn  4. .
2n
Từ đó ta có lim yn  2  lim xn  4 .

an2  5an  10
Bài 2. Cho dãy ( a ) 
n n 1 : a1  1; an 1  , n  1 .
5  an
a) Chứng minh dãy (an ) hội tụ và tính lim an .

a1  a2  ...  an 5  5
b) Chứng minh  , n  1. .
n 2
Hướng dẫn giải
3
a) Bằng phương pháp chứng minh qui nạp ta có: 1  an  , n .
2

5 5 x 2  5 x  10 10
Đặt A  và xét hàm f ( x)    x, ( x  5) .
2 5 x 5 x
10  3 1 
Suy ra f ( x)   1  0, x  1;  . , như vậy f ( x) nghịch biến trên đoạn  ;1 . .
5  x   2 2 
2

a  a3  a5  ...  a2 k 1  ...  A  lim a2 k 1  b  A


Dẫn đến  1  ..
 a2  a4  a6  ...  a2 k  ...  A   lim a2 k  c  A
Kết hợp công thức xác định dãy ta được.

 c 2  5c  10
 b 
5c 5 5
  b  c  .
c  b 2
 5b  10 2
 5b

5 5
Vậy lim an  ..
2

 5 5 
b) Nhận xét: t  1;  . thì t  f (t )  5  5. .
 2 

Dẫn đến a2 k 1  a2 k  5  5 , k  1 .

5 5
 a1  a2  ...  a2 k 1  a2 k  2k . (1).
2
Như vậy bất đẳng thức đúng với n  2k .
5 5
Trường hợp n  2k  1 , chú ý a2 k 1  , kết hợp với (1) thu được:.
2

5 5
a1  a2  ...  a2 k 1  a2 k  a2 k 1  (2k  1) .
2
Vậy bất đẳng thức được chứng minh.

1 u e un
Bài 3. Cho dãy số thực  un  : u1  , un 1  n un , n  *
. Chứng minh dãy trên có giới hạn hữu
2 1 e
hạn, tìm giới hạn đó.
Hướng dẫn giải
Chứng minh 1  un  0, n  2 1 .

1
e
Với n  2, u2  2   1;0  đúng.
1 e

Giả sử 1 đúng với n  k  2 , ta chứng minh 1 đúng với n  k  1 .

u n e un
Ta có un  0  eun  1  1  eun  0   0.
1  e un
u
1 un e n
un  1  eun  ;  1  un eun  eun  1  eun  un  1  1 (luôn đúng).
e 1 e un

Vậy (1) được chứng minh.

xe x e x 1  x  e x 
Xét hàm f  x   trên  ; 0  . Ta có f '  x   .
1  ex 1  e x 
2

Hàm g  x   1  x  e x có g '  x   1  e x  0 với mọi x   ;0  nên hàm này đồng biến trên  ; 0  .

e x 1  x  e x 
Suy ra g  x   g  0   0 , suy ra f '  x    0.
1  e 
x 2

hay hàm f  x  nghịch biến trên  ; 0  .

e
e 
2 1 e 
e
 
1
e
e 2 1  e
Ta có u2  2  , u3  , u4  u2 .
1 e 2 1 e  1 e
 e
21 e 

Suy ra f  u4   f  u2   u5  u3  0  u1 .

Quy nạp ta được dãy  u2 n 1  giảm và dãy  u2n  tăng.

Hơn nữa 1  un  0, n  2 nên mỗi dãy trên tồn tại giới hạn hữu hạn.

Giả sử lim u2 n  a, lim u2 n 1  b  a, b   1;0   , lấy giới hạn hai vế ta được.


 beb
 a  ae a
1  eb
 a
 1  e   e e .
a 2 a 1 e a

b  ae
 1  e a

  1  t
Đặt ea  t  t   ;1  , ta được phương trình 1  t   t.t 1t  2 1  t  ln 1  t   1  t  ln t  t ln t  0 .
2

  e 

Hàm h  t   2 1  t  ln 1  t   1  t  ln t  t ln t nghịch biến nên phương trình có nhiều nhất 1 nghiệm,


1
nhận thấy t  là nghiệm nên nó là nghiệm duy nhất.
2
1 1
Suy ra a  ln , thay vào được b  ln .
2 2
1
Vậy lim un  ln .
2
2n  3
Bài 4. Cho dãy số  an  , n  1 thỏa mãn a1  1, an  an 1 , n  2 và dãy  bn  , n  1 thỏa mãn
2n
n
bn   ai , n  1. Chứng minh dãy  bn  có giới hạn và tìm giới hạn đó.
i 1

Hướng dẫn giải

Ta có 2nan   2n  3 an 1  an 1  2  n  1 an 1  nan  , n  1 .

n
Do đó bn   2 iai   i  1 ai 1   2 1   n  1 an 1  .
i 1

1
Ta chứng minh bằng quy nạp rằng nan  , n  1.
n
Thật vậy:.
- Với n = 1, ta có a1  1 nên khẳng định đúng.

2  n  1  3  2n  1   1 
- Giả sử khẳng định đúng với n  n  1 . Ta có an1  an     , ta cần chứng
2  n  1  2n  2   n n 
 2n  1   1  1
minh       2n  1 n  1  2n n .
 2n  2   n n   n  1 n  1

  4n 2  4n  1  n  1  4n3  1  3n .

Bất đẳng thức cuối đúng nên khẳng định trên đúng với n  1 .
Theo nguyên lí qui nạp thì khẳng định được chứng minh.

 1 
Ta có 2 1    2 1   n  1 an 1   bn  2 .
 n 1 

Theo nguyên lí kẹp thì dãy  bn  có giới hạn và lim bn  2 .


 1
u1  2

Bài 5. Cho dãy số  bn  được xác định bởi:  .
 
un 1  1  un  un 2  1 
 2  4n 

Chứng minh dãy số hội tụ và tìm lim un .


x 

Hướng dẫn giải


1 
Ta chứng minh un  n
cot n 1 ; n  (*) .
2 2
1  1
Thật vậy: n  1 : u1  1
cot 11  .
2 2 2
 (*) đúng với n  1 .

1 
Giả sử (*) đúng tới n  k , k  *
, nghĩa là có : u  cot .
k
2 k 2 k  1

1 1 
Ta chứng minh (*) cũng đúng với n= k+1. Thật vậy uk 1   uk  uk 2  k  .
2 4 

1 1  1  1  1   2 

  k cot k 1  k cot 2 k 1  k   k 1  cot k 1  cot k 1  1  .
22 2 4 2 4  2  2 2 

 
1   1  
 k 1  cot k 1    ( vì khi k   thì k 1  0; sin  0 ).
2  2 sin k 1  2
 2 
 
cos 1 2 cos 2
1 2 k 1 1 2k  2 1 
 .   k 1 cot k  2 .
2 k 1
 k 1
2 2sin  
sin k 1 cos k  2 2 2
2 2k  2 2
 (*) cũng đúng với n  k  1 .
1 
Vậy un  n
cot n 1 ; n  .
2 2

     
 cos 2n 1 1    2 n 1
2  2
lim un  lim  . n   lim  cos n 1  .
x  x 

 2  x  2    
n 1
 2   2n 1 
2
Vậy dãy hội tụ và có lim un  .
x  
Bài 6. Cho phương trình: x n  x 2  x  1  0 với n  N, n  2 .
1)Chứng minh rằng với mỗi số nguyên n  2 , thì phương trình có một nghiệm dương duy nhất xn .
2)Xét dãy số sau đây: U n  n  xn  1 , n  2,3, 4,... Tìm limU n ? .
Hướng dẫn giải
Xét phương trình: f  x   x n  x 2  x  1  0 , với n nguyên, n  2 (1).

+) Ta có: f ’  x   nx n 1 – 2 x –1 . Do n  2 , nên khi x  1 thì f ’  x   0 . Vậy f  x  là hàm số đồng biến


trên 1;  .

Lại có: f 1  2  0 ; f  2   2n – 7  0 ( vì n nguyên và n  2  n  3).

Ta có: f 1 f  2   0 và f  x  liên tục, đồng biến nên phương trình f  x   0 có nghiệm duy nhất trên
1;  .
+) Mặt khác với 0  x  1 thì x n  x 2 ( do n  2 ) suy ra f  x   0 với mọi 0  x  1 .

Như vậy ta đã chứng minh được (1) có nghiệm dương duy nhất với mọi n nguyên, n  2 .
Gọi xn là nghiệm dương duy nhất của phương trình x n – x 2 – x –1  0 .

Bây giờ xét dãy U n  với U n  n  x n  1 , n  3, 4,5, .

Ta có: xnn  xn2  xn  1  0 hay xn  n xn2  xn  1 .

Áp dụng bất đẳng thức AM – GM, ta có:.

x n2  x n  1
1  ....
 1
xn  n xn2  xn  1  n xn2  xn  1.1
n so 1
.1 1 <
....
n
(2).
n 1 sô 1

(Chú ý rằng ở đây 1  xn nên xn2  xn  1  1 , vì thế trong bất đẳng thức không có dấu bằng).

6
+) Mặt khác do xn  2 , nên xn2  xn  6 , nên từ (2) có: 1  xn  1  (3).
n
6
Bất đẳng thức (3) đúng với mọi n  3 và lim  0 nên từ (3) ta có: lim xn  1 .
n

ln xn2  xn  1
+) Ta có: x nn  x n2  x n  1  n ln xn  ln xn2  xn  1  n  .
ln xn

Từ đó: nxn  1 
xn  1 ln x 2  x  1 (5).
n n
ln xn

Đặt y n  xn  1  lim y n  0 .

Ta có: suy ra từ (5) lim U n  lim n  xn  1  ln 3 .


Vậy: lim U n  ln 3 .

ln xn ln  yn  1 ln  t  1
Bài 7. Cho số thực a , xét dãy số  xn n1 được lim  lim  lim  1 xác định
xn  1 yn t 0 t
xn3  6 xn  6
bởi x1  a, xn1  , n  1, 2,.... . Tìm tất cả các giá trị của a để dãy số có giới hạn
3xn2  9 xn  7
hữu hạn, tìm giới hạn đó?.
Hướng dẫn giải
Với a  1 thì xn  1, n  1 nên lim xn  1 .
n 

 xn1  1  xn1  2 
3 3

Với a  1 thì xn  1  , xn  2  , n  2 .
3xn21  9 xn1  7 3xn21  9 xn1  7
3 3n1
x  2  xn 1  2  a2
Do đó n    ...   , n  1.
xn  1  xn 1  1   a 1 
3n1 3n1
2  a  1   a  2
Từ đó, tính được xn  3n1 3n1
, n  1 ,.
 a  2   a  1

3
Kết luận + a    a  1  a  2  lim xn  2 .
2 n 

3
+ a    a  1  a  2  lim xn  1 .
2 n 

3 3 3
+ a    xn   , n  1  lim xn   . .
2 2 n  2

 2012
u1 
Bài 8. Cho dãy số (un ) xác định như sau:  2013 . Tìm lim un .
n 
u 2  2u  1  0 , n  1, 2,3,...
 n n 1

Hướng dẫn giải


un2 1
Ta có : un 2  2un 1  1  0  un 1   .
2 2

x2 1 x2 1 1
Xét hàm số : f ( x)     .
2 2 2 2
f '( x)  x .

. x 1 0 1
2
Ta có :. f  x  0 
f  x 3 0
8

1
2

1 1 1 3
 u1  1   u2  0   u3    0 .
2 2 2 8
Vậy : n  2 thì 1  un  0 .

un2  1
un 2  2un 1  1  0  un 1  .
2
x2 1 1
Gọi a là nghiệm của :  x ( x  ( ;0))  a  1  2 .
2 2
Ta có : un 1  a  f (un )  f (a ) .

Theo định lí La-grăng : f (un )  f (a)  f '(a) . un  a .

1 1
Do f '(a)   f (un )  f (a)  un  a .
2 2
2 n
1 1 1
 un 1  a  un  a    un 1  a  ...    u1  a .
2 2 2
n
1
Mà lim    0  lim (un 1  a)  0  lim un 1  a  1  2 .
 
n  2 n  n 

Vậy : lim un  1  2 .
n 

 1
u0  2

Bài 9. Cho dãy số un  xác định như sau:  . Chứng minh rằng dãy số un 
un 1  un  5 , n 
2

 2  un  2 
có giới hạn và tìm giới hạn đó.
Hướng dẫn giải
* Vì 0  u0  1 nên 0  un  1, n  .

9
* Áp dụng BĐT Cauchy ta có un  2   6 . Dấu bằng xảy ra  un  1 .
un  2
9
 un  2   6 , n  .
un  2
un 2  5 1 9 
* un 1    un  2    2  1, n  .
2  un  2  2  un  2 

1 9
* un 1  un   un  1  .
2 2  un  2 

1 9
Xét hàm số f  x    x  1  .
2 2  x  2

1 9
f ' x     0, x  1  f  x  nghịch biến trên 1;   .
2 2  x  2 2

* Vì un  1  f  un   f 1  0  un 1  un , n  *
.
 un  giảm và bị chặn dưới  un  có giới hạn hữu hạn.

un 2  5
* Giả sử lim un  a 1  a    . Từ un 1  chuyển qua giới hạn ta có.
2  un  2 
a2  5 a  1
a  .
2  a  2  a  5(loai)
* Vậy lim un  1 .

un 1
Bài 10. Cho dãy số (un ) được xác định bởi: u1  4 và un 1  un2  2 , với n  *
. Tìm lim .
n  u .u ...u
1 2 n

Hướng dẫn giải


Với mọi n  1, 2,... ; ta có.

un21  4   un2  2   4  un4  4un2  un2  un2  4   un2 .un21 (un21  4) .


2

 ...  un2un21...u22u12 (u12  4)  12  un .un 1...u1  (1).


2

2
 un 1  4
Từ (1) ta có:    12  ; n  1, 2,... (2).
 u1.u2 ...un 
2
 u1.u2 ...un 

Mặt khác, vì u1  4  2 nên từ un 1  un2  2 và chứng minh bằng quy nạp ta thu được un  2 với mọi
n  1, 2,... .

4 4
Do đó u1.u2 ...un  2n ; n  *
. Khi đó, 0   ; n  1, 2,... .
 u1.u2 ...un 
2
22 n

4
nên theo nguyên lý kẹp giữa ta có: lim  0.
 u1.u2 ...un 
n  2

2
 un 1 
Vậy, từ (2) suy ra: lim 
n  u .u ...u
  12 .
 1 2 n

Mặt khác, hàm số f ( x)  x liên tục trên nửa khoảng [0;  ) nên.
2 2
un 1  un 1   un 1 
lim  lim    lim    12 .
n  u u ...u n  n  u u ...u
1 2 n  u1u2 ...un   1 2 n

un 1
Kết luận: lim  12 .
n  u .u ...u
1 2 n

Bài 11. a) Chứng minh rằng có đúng một dãy số thực ( xn ) n 0 thỏa mãn.

xn  xn 1
x0  1, 0  xn  1n  1 và (1  xn ) 2  (1  xn 1 ) 2  n  1. .
2
b) Với dãy ( xn ) xác định như trên, xét dãy ( yn ) n  0 xác định bởi yn  x0  x1  ...  xnn  0. Chứng
minh rằng dãy ( yn ) n  0 có giới hạn hữu hạn khi n   . Hãy tìm giới hạn đó.

Hướng dẫn giải


a) Bằng quy nạp ta sẽ chỉ ra rằng xn xác định duy nhất với mỗi n  0. Để làm được điều này ta cần dùng
kết quả (chứng minh của nó là đơn giản) sau: Với mỗi số thực m  [0;1] , phương trình
tm
(1  t ) 2  (1  m) 2  có đúng một nghiệm trên [0;1] .
2
1 1 1 1 1
b) Để ý rằng yn  x0  ( x0  x1 )  ( x1  x2 )   ( xn 1  xn )  xn n  1. .
2 2 2 2 2
3
Ta có giới hạn cần tìm bằng ..
2

Bài 12. Giả sử  Fn  n  1, 2,... là dãy Fibonacci ( F1  F2  1; Fn 1  Fn  Fn 1 với ). Chứng minh


Fn 1
rằng nếu a với mọi n  1, 2,3,... thì dãy số  xn  , trong đó
Fn
1
x1  a, xn 1   n  1, 2,3... , là xác định và nó có giới hạn hữu hạn khi n tăng lên vô hạn.
1  xn
Tìm giới hạn đó.
Hướng dẫn giải
Giả sử x1 , x2 ,..., xm đã được xác định. Khi đó xm 1 được xác định khi xm  1 .

1
* Nếu xm  1 thì do xm  nên xm 1  2 .
1  xm1

F2 F
Từ giả thiết F1  F2  1; Fn 1  Fn  Fn 1 ta viết xm   , xm1   3 .
F1 F2

Fi  2
Giả sử xmi   , với i nào đó, 0  i  m  2 .
Fi 1

1 1 F F
Vì xm i  nên xmi 1   1   i 1  1   i 3 .
1  xm i 1 xmi Fi  2 Fi  2

Fm1 F
Khi đó x1   . Mâu thuẫn với giả thiết x1   m1 . Như vậy ( xn ) là dãy số xác định.
Fm Fm

1 5 1  5 1
Phương trình x   x 2  x  1  0 có hai nghiệm u  ,v  . Có hai trường hợp xảy ra:.
1 x 2 2

 5 1
Trường hợp 1: x1  v . Khi đó xn  x1 , n  1 . Do đó lim xn  .
n  2
1 1 v
Trường hợp 2: x1  v . Chú ý  v  xn   xn  v . Do đó xn  v, n  1 .
1  xn v

xn  u
Đặt zn  , ta có.
xn  v
1
u
xn 1  u 1  xn (1  u )  uxn u 2  uxn u xn  u u
zn 1     2  .  .zn .
xn 1  v 1
v (1  v )  vxn v  vxn v xn  v v
1  xn
n
u u
Từ đó có zn    .z1 nên zn  0 khi n   (vì  1 ).
v v
xn  u u  vzn
Từ zn  suy ra xn  dần tới u khi n   (do zn  0 ).
xn  v 1  zn

5 1
Tức là trong trường hợp này lim xn  .
n  2
Bài 13. Cho dãy số  yn  thỏa mãn y1  0, yn31  y1  y2  ...  yn , n  1 . Chứng minh rằng dãy số
 yn 
  có giới hạn bằng 0 khi n   .
n
Hướng dẫn giải
Từ giả thiết ta có yn31  yn  yn3 , n  2 , do đó dãy số  yn n  2 là dãy tăng, vì.

vậy yn31  yn  yn3  yn ( yn2  1)  yn 1 ( yn2  1) .

 yn21  yn2  1 , n  2  yn21  yn2  1  ...  y22  n  1 .

y22  n  1
2
 y  y2  n 1
  n1   2 . Mà lim  0 nên theo định lý kẹp ta có.
 n 1  (n  1) 2 (n  1)2
2
 y  y y
lim  n1   0  lim n1  0  lim n  0 .
 n 1  n 1 n

Bài 14. Cho  un  là một dãy số dương. Đặt S n  u13  u23  ...  un3 với n  1, 2,... Giả sử

un 1    Sn  1 un  un 1 
1
với n  2,3,.... Tìm lim un .
Sn 1

Hướng dẫn giải


Ta có Sn 1  Sn  un31  0, n  1, 2,...   Sn  là dãy số tăng.

Nếu dãy số  S n  bị chặn trên thì  S n  là một dãy hội tụ và lim un3  lim  S n 1  S n   0  lim un  0 .

Xét trường hợp dãy số  S n  không bị chặn trên thì lim S n   .

Từ giả thiết ta có S n 1un 1  un  S nun  un 1 , n  2,3,... .

Từ đây ta thu được Snun  un 1  S 2u2  u1 , n  2,3,... .

un 1 S2u2  u1 S u u
Do đó un    0  un  2 2 1 , n  2,3,... .
Sn Sn Sn

Theo nguyên lí kẹp ta có lim un  0 .


Vậy trong mọi trường hợp ta đều có lim un  0 .

u1  1

Bài 15. Cho dãy số (un ) xác định bởi công thức truy hồi:  1 . Chứng minh
 u n 1  u n   2, n  *

 un
rằng dãy (un ) có giới hạn hữu hạn và tính giới hạn đó.

Hướng dẫn giải


1 1 1
Đặt f ( x)  x   2; g ( x)  f ( f ( x))  x    2 2 . Khi đó.
x x x 1  2
x

 2 2
2  x    x  1
 2  1 1
g '( x)  2
 0  g ( x)  g ( )  0  f ( f ( x))  x, x  ( ;1) (*). .
4 1  2 2
x x  2
 x 
1
Mặt khác f '( x)  0, x  ( ;1) nên.
2
1 1 1 1 1
f ( x)  f ( )  f ( f ( x))  f ( )  , x  ( ;1) (**). .
2 2 2 2 2
1 1
Từ (*) và (**) suy ra:  f ( f ( x))  x, x  ( ;1). .
2 2
1 1
Vậy: 1  u1  u3   1  u1  u3  u5  ,... Do đó (u2 n 1 ) là đơn điệu giảm và bị chặn dưới nên tồn
2 2
1
tại lim u2 n 1  ..
n  2

 1 
Vì f ( x) liên tục trên  ;1 nên.
 2 

n 

u2 n  f (u2 n 1 )  lim u2 n  f lim u2 n 1 
n 
 1
2
..

Vậy dãy (un ) được phân tích thành hai dãy con hội tụ tới cùng một giới hạn. Do đó dãy (un ) có giới hạn
1
bằng ..
2
Bài 16. Tìm tất cả các hàm số f :  thỏa mãn đồng thời các điều kiện sau đây:.

1. f  x  y   f ( x)  f ( y ) với mọi x, y  .

2. f ( x)  e x  1 với mỗi x  .

Hướng dẫn giải


f  x  0   f ( x)  f (0)  f (0)  0 và bởi vì f (0)  e0  1  0 nên f (0)  0 .

f ( x  ( x))  f ( x)  f ( x)  f ( x)  f ( x)  0 (1) .
 x  x  2x 
f ( x )  f    f    2  e  1 .
2 2  

 x   x  x  x 
f ( x )  2  e 2  1  f ( x )  f    f    4  e 4  1 .
  2 2  

 2xn 
Dùng quy nạp theo n  1, 2,  ta CM được f ( x)  2  e  1 .
 
 

 2x0n 
Cố định x0  ta có f ( x0 )  2  e  1  .
n
 
 

 2x0n 
 n x0
  e 1 
Xét dãy an  2n  e 2  1 ta có : lim an  lim  x0 x0   x0 .
 
   2
n


Vậy f ( x0 )  x0 , x0  (2) .

Vậy f ( x)  f (  x)  x  ( x)  0 (3) .

Kết hợp ( 1) và (3) ta được f ( x)  f (  x)  0 .

Từ (2)  f ( x)   x  f ( x)  x (4) . Kết hợp ( 2) và (4) ta được f ( x)  x, x  .

Thử lại f ( x )  x ta thấy đúng.

 x1  1,

Bài 17. Cho dãy số  xn  được xác định như sau  3 x . Chứng minh rằng  xn  có
 xn 1  xn  2 n  1
n

 n
giới hạn hữu hạn khi n dần đến vô cùng.
Hướng dẫn giải
Dễ thấy xn  0 , với mọi n nguyên dương, nên dãy số đã cho là dãy tăng thực sự.

Vậy để chứng minh dãy số có giới hạn hữu hạn ta chỉ cần chứng minh nó bị chặn trên.
Ta chứng minh xn  8, n  *
.

Thật vậy, với n  1  x1  1  8 nên điều cần chứng minh đúng.

Giả sử ta có: xn  8 , với n nguyên dương. Ta cần chứng minh xn 1  8 .

n n
3 xk 1
Theo công thức xác định dãy số có: xn 1  x1   2
 1  2 2
 1  2.2  8 .
k 1 k k 1 k

Do đó xn  8 với mọi n nguyên dương từ đó suy ra điều phải chứng minh.


 1 3
 a1  4 ; a2  10
Bài 18. Cho dãy số thực  an  xác định bởi  2
. Chứng minh rằng dãy
 a   n  n 1 , n  , n  2
1 a a
 n 2 6 3
 an  có giới hạn hữu hạn. Hãy tìm giới hạn đó.

Hướng dẫn giải


Có a1 , a2   0;1 , giả sử a1 , a2 ,..., ak   0;1 , k  , k  2 . Từ công thức truy hồi ta có:.

1 1 a a2 1 1 1
 0  0  ak 1   k  k 1     1, vì 0  ak 1 , ak  1  ak 1   0;1 .
2 2 6 3 2 6 3
Vậy bằng phương pháp quy nạp ta chứng minh được an   0;1 , n  *
.

 1  3
 x1  x2  4  y1  y2  10
Xét hai dãy số mới  xn  :  2
và  yn  :  2
với n  ; n  2 .
 x  1  xn  xn 1  y  1  yn  yn 1
 n 1 2 6 3  n 1 2 6 3
1
Có 0  x1  x2   x3  1 , giả sử ta có 0  x1  x2  ...  xk  1, k  , k  3 , khi đó.
2

1 xk 1 xk2 2 1 xk xk21
xk        xk 1 .
2 6 3 2 6 3
Vậy bằng phương pháp quy nạp ta chứng minh được  xn  là dãy số tăng và bị chặn trên bởi 1, nên nó có
giới hạn hữu hạn lim xn   .

 3
1  2  
Chuyển công thức truy hồi qua giới hạn tìm được      2.
2 6 3 
  1

Do  xn    0;1 nên suy ra   1 .

Chứng minh tương tự đối với dãy số  yn  , ta cũng có lim yn  1 .

Cuối cùng ta chứng minh xn  an  yn , n  *


(1) bằng phương pháp quy nạp:.

Ta có x1  a1  y1 và a2  x2  y2 , với n = 1, 2 bất đẳng thức (1) đúng. Giả sử (1) đúng tới k  , k  2 ,
tức là xi  ai  yi , i  1, 2,..., k . Khi đó.

1 xk xk21 1 ak ak21 1 yk yk21


xk 1     ak 1        yk 1 .
2 6 3 2 6 3 2 6 3
Từ xn  an  yn , n  , n  1 và áp dụng định lý kẹp ta suy ra được lim an  1 .

Bài 19. Cho hai dãy số  an  ;  bn  xác định bởi a1  3, b1  2 , an 1  an2  2bn2 và bn 1  2anbn với n = 1,
2, 3,…. Tìm lim 2n bn và lim 2n a1a2 ...an .
n  n 

Hướng dẫn giải


Với mọi n = 1,2,3,… ta có.

 
2
an 1  bn 1 2  an2  2bn2  2 2anbn  an  bn 2 .

Do đó:.

   a       
2 22 2n1 2n1 2n
an  bn 2  an 1  bn 1 2 n2  bn 2 2  ...  a1  b1 2  3 2 2  2 1 .

 
2n
Tương tự ta có: an  bn 2  2 1 .

1
     1 
    
2n 2n 2n 2n
Từ đó: an   2 1 2 1  ; bn   2 1 2 1 .
2  2 2 

   
2n 2n

2n
2 1 n
2 1
 2 bn  2 an  2  1 và lim 2  2  1 , nên theo nguyên lí kẹp ta có:
n n
Chú ý:
4 2 n 4 2
lim 2 bn  lim 2 an  2  1 .
n n

n  n 

bn1 b b b b
Mặt khác: bn 1  2anbn hay an  (n  1) . Suy ra: a1a2 ...an  2 . 3 ... n 1  n n1 . Do đó
2bn 2b1 2b2 2bn 2

  1
2
lim 2n a1a2 ...an = lim 2n bn 1  2  1  3  2 2 (vì lim 2n  1 ).
n  n  n 2n

 1 3
 a1  4 ; a2  10
Bài 20. Cho dãy số thực  an  xác định bởi  2
. Chứng minh rằng dãy
 a  1  an  an 1 , n  , n  2
 n 2 6 3
 an  có giới hạn hữu hạn. Hãy tìm giới hạn đó.

Hướng dẫn giải

+ Ta Có a1 , a2   0;1 , giả sử a1 , a2 ,..., ak   0;1 , k  , k  2 . Từ công thức truy hồi ta có:.

1 1 ak ak21 1 1 1
 0  0  ak 1        1, vì 0  ak 1 , ak  1  ak 1   0;1 .
2 2 6 3 2 6 3
Vậy bằng phương pháp quy nạp ta chứng minh được an   0;1 , n  *
.

 1
 x1  x2  4
+ Xét hai dãy số mới  xn  :  2
.
 x  1  xn  xn 1 , n  , n  2
 n 1 2 6 3

 3
 y1  y2  10
và  yn  :  2
.
 y  1  yn  yn 1 , n  , n  2
 n 1 2 6 3

1
- Có 0  x1  x2   x3  1 , giả sử ta có 0  x1  x2  ...  xk  1, k  , k  3 , khi đó.
2
1 xk 1 xk2 2 1 xk xk21
xk        xk 1 .
2 6 3 2 6 3
Vậy bằng phương pháp quy nạp ta chứng minh được  xn  là dãy số tăng và bị chặn trên bởi 1, nên nó có
giới hạn hữu hạn lim xn   . Chuyển công thức truy hồi qua giới hạn tìm được
 3
1  2 
    2 . Do  xn    0;1 nên suy ra   1 .
2 6 3 
  1

- Chứng minh tương tự đối với dãy số  yn  , ta cũng có lim yn  1 .

- Cuối cùng ta chứng minh xn  an  yn , n  *


(1) bằng phương pháp quy nạp:.

Ta có x1  a1  y1 và a2  x2  y2 , với n = 1, 2 bất đẳng thức (1) đúng. Giả sử (1) đúng tới k  , k  2 ,
tức là xi  ai  yi , i  1, 2,..., k . Khi đó.

1 xk xk21 1 a a2 1 y y2
xk 1     ak 1   k  k 1   k  k 1  yk 1 .
2 6 3 2 6 3 2 6 3
+ Từ xn  an  yn , n  , n  1 và áp dụng định lý kẹp ta suy ra được lim an  1 .

1
Bài 21. Tìm giới hạn: lim(2014  n
).
n!
Hướng dẫn giải
n
n
Trước hết ta chứng minh bất đẳng thức: n !    (*) n  N * ).
3
1
Bằng phương pháp qui nạp. Thật vậy: với n  1 , ta có 1  (đúng).
3
k
k
Giả sử (*) đúng với n  k tức là: k !    . Ta đi chứng minh (*) đúng với.
3
n  k 1.

k  1 k 1 k  1 k 1
k
k 3
Ta có  k  1 !  k ! k  1     k  1  ( ) . ( ) .
3 3 1 k
(1  ) 3
k
Bất đẳng thức cuối này đúng vì:.

k k (k  1) 1 k (k  1)(k  2)....(k  k  1) 1
k
 1
1    1   . 2  ...  . 2
 k k 2! k k! k
1 1 1 1 2 k 1
 1  1  (1  )  ...  (1  )(1  )...(1  )
2! k k! k k k
1 1 1 1 1 1
 1  1   ...   1  1   ...  n 1  1  1   ...  n 1  ..
2! n! 2 2 2 2
1 .
 1 3
1
1
2
n
n n
Vậy (*) đúng với n  k  1 . Do đó n !    , từ đây ta suy ra n
n! > .
3 3

1 3 3
=> 0  n
 . Vì lim 0.
n n
n! n
1
Do đó theo định lý về giới hạn kẹp giữa ta suy ra: lim = 0.
n n
n!
1
Vậy lim(2014  n
) =2014.
n!
3.4. CÁC DẠNG KHÁC

 x1  2016

Bài 1. Tìm các giá trị thực của tham số m để dãy số  xn  :  m có giới hạn hữu
 x   n  N *

1  xn2
n 1

hạn.

Hướng dẫn giải


*) m  0  0  xn  m n  1 .

m 2mx
th số: f ( x)  t f '( x)   f  x  ghị h i tr  0; m  .
x 12
( x 2  1)2

Suy ra ( x2 n ), ( x2 n 1 ) đ đi u v ị h .

2017  x1  x3  x5  ... 
+ 0m  x1  x2 , x3   .
2016  x2  x4  x6  ... 
4m m
f ( f (1))   1, x2   1  x2 n  1 n  N * .
m 4
2
2017

a(1  b2 )  m

i s lim x2 n  a, lim x2 n 1  b  a  1,  (I ) .

b (1  a 2
)  m

 a  b
 3 ( II )
 a  a  m

( I )   b  1 .
  a

( III )
1
 a   m
  a

Khi o  m  2 h ghi u h t   xn  giới hạ hữu hạ .

2017
Khi 2  m  h ghi u h t ớ h 1 v h ghi th a b. D đ
2016
 lim x2 n  lim x2 n 1  ( xn ) h g giới hạ .

2017  x1  x3  x5  ...
  m  2017 2016  x1  x2 , x1  x3   .
2016  x2  x4  x6  ...
 lim x2 n  lim x2 n 1  ( xn ) h g giới hạ .

+ m  2017 2016  xn  2016 n  N *  l imxn  2016 .

 x1  x3  x5  ...
+ m  2017 2016  x1  x2 , x1  x3   .
 x2  x4  x6  ...
 lim x2 n  lim x2 n 1  ( xn ) h g giới hạ .

*) m  0 t g tự t 0  m  2 v m  2017 2016 .
xn3  6 xn  6
Bài 2. Cho số thực a , xét dãy số  xn n1 đ xá định bởi x1  a, xn1  , n  1, 2,.... .
3xn2  9 xn  7
Tìm t t c các giá trị của a để dãy số có giới hạn hữu hạn, tìm giới hạ đ ?.

Hướng dẫn giải


Với a  1 thì xn  1, n  1 nên lim xn  1 .
n 

 xn1  1  xn1  2 
3 3

Với a  1 thì xn  1  , xn  2  , n  2 .
3xn21  9 xn1  7 3xn21  9 xn1  7
3 3n1
xn  2  xn 1  2  a2
D đ    ...   , n  1.
xn  1  xn 1  1   a 1 
3n1 3n1
2  a  1   a  2
Từ đ , tí h đ c xn  3n1 3n1
, n  1 ,.
 a  2   a  1

3
K t luận + a    a  1  a  2  lim xn  2 .
2 n 

3
+ a    a  1  a  2  lim xn  1 .
2 n 

3 3 3
+ a    xn   , n  1  lim xn   . .
2 2 n  2

 1  an 1
an  bn  1  a
Bài 3. Cho hai dãy số g  an n0 ,  bn n0 xá định bởi: a0  3, b0  2 và  n 1 .
a 2  1  b 2
 n n

Với mọi n  0,1, 2,... . Chứng minh rằng hai dãy trên hội tụ và tìm giới hạn của chúng.

Hướng dẫn giải


 1
Ta chứng minh bằng quy nạp an  tan , bn  , n  0,1, 2,... (*) . Thật vậy.
3.2 n

cos
3.2n
  1
Với n  0 , ta có a0  3  tan  tan , b0  2  , vậy * đú g.
3 3.2 0

cos
3.20
1   2 1
Với n  1 , ta có a1   tan  tan 1 , b1   , vậy * đú g.
3 6 3.2 3 cos 
3.21
 1
Gi s khẳ g đị h đú g đ n n  k , k  1 , tức là an  tan , bn  .
3.2 n

cos
3.2n
 1
Ta chứng minh an 1  tan , bn 1  . Thật vậy. Từ 1 ta có.
3.2 n 1

cos
3.2n 1
    
sin  1 2sin cos n 1  sin 2  cos 2
1  an 1 3.2 n
3.2 n 1
3.2 3.2 n 1
3.2 n 1 
 
1  an 1   
cos n cos 2 n 1
 sin 2
3.2 3.2 3.2n 1
  
2

 sin n 1
 cos n 1 
  3.2 3.2 
      
 cos n 1  sin n 1  
cos n 1  sin 
 3.2 3.2   3.2 3.2n 1 
  
sin  cos tan 1
3.2 n 1
3.2 n 1
3.2n 1 
  a  tan
   n 1
3.2n 1
cos n 1
 sin n 1
1  tan n 1
3.2 3.2 3.2
 1 1
Khi đ từ  2  , suy ra bn21  an21  1  tan 2 1   bn 1  .
3.2 n 1
2  
cos cos
3.2n 1 3.2n 1
 1
Nh vậy theo nguyên lý quy nạp thì an  tan , bn  , n  0,1, 2,... .
3.2 n

cos
3.2n
 1 1
D đ lim an  lim tan  tan 0  0; lim bn  lim  1.
n  n  3.2 n
n  n   cos 0
cos
3.2n
K t luận: lim an  0; lim bn  1 .■.
n  n 

u1  2014
Bài 4. Cho dãy số (un ) xá đị h h s u :  . Tì điều ki n của
 n 1
u  u 2
n  (1  2 a )u n  a 2
; n  1, 2,...
a để dãy số (un ) có giới hạn hữu hạn khi n   và tính giới hạ đ .

Hướng dẫn giải


Ta có: un 1  un  (un  a ) 2  0  un 1  un ; n  1, 2,3,... .

* Suy ra dãy số (un ) tă g ; từ đ số (un ) có giới hạn hữu hạn khi và chỉ khi dãy bị ch n trên.

Gi s lim un  L ( L  ) , thì chuyển qua giới hạn h thức un 1  un2  (1  2a)un  a 2 ta có:
n 

L  L2  (1  2a) L  a 2  L  a .

- N u có chỉ số k  *
mà uk  a thì un  a; n  k trái với k t qu lim un  L  a .
n 

D đ : uk  a với mọi k  1, 2,... hay un2  (1  2a)un  a 2  a, n  1, 2,3,... .

 a  1  u1  a  a  1  2014  a .

* Đ o lại: N u a  1  2014  a  a  1  u1  a .

 (u1  a  1)(u1  a )  0  u12  (1  2a)u1  a 2  a  0  u2  a .

và u1  u2  a  1  u2  a .
Bằng quy nạp ta chứ g i hđ c a  1  un  a, n  1, 2,3,... (H/s trình bày ra).

Nh vậy dãy (un ) tă g , ị ch n trên bới a , đ số (un ) có giới hạn hữu hạn.

Kết luận: Với điều ki n a  1  2014  a thì dãy số (un ) có giới hạn hữu hạn khi n   và lim un  a .
n 

 x1  a

Bài 5. Cho dãy số  xn  th a mãn  2 xn3 . Tìm a sao cho dãy số xá định và có
 x   , n  1, 2,3,...
3xn2  1
n 1

giới hạn hữu hạn.

Hướng dẫn giải

2 x3 3
Đ t f  x  2 , x   . Ta có x1  a, xn 1  f  xn  . Ta có.
3x  1 3

6x4  6x2 6 x 2  x 2  1
f ' x   .
 3x  1  3x  1
2 2 2 2

B g i thi .

3
T xâ ự g số h s u a0  , a0  f  a1  , a1  f  a2  , a2  f  a3  ,... .
3
Nhậ th a1 , a3 ,..., a2 k 1 ,...  0; a0 , a2 ,..., a2 k ,...  0 .

 3   3
Dự v g i thi t th a1    ;0  , a2  f 1  a1    0;  .
 3   3 

 a2  a0  f  a3   f  a1   a3  a1  f  a4   f  a2   a4  a2 .

3
Bằ g qu ạp t hứ g i hđ  a2k  đ đi u gi , ị h ởi 0 v , dãy  a2 k 1  đ đi u
3
3
tă g v ị h ởi  v 0. Từ đ tồ tại lim  a2 k  , lim  a2 k 1  .
3 k  k 

Ta có an  f  an 1   f  f  an  2    lim an  f  f  lim an  2    l  f  f  l   .
3
 2l 3 
2 2 
3l  1   1
l   l  l 2    l 2  1 20l 4  15l 2  5   0 (*).
 5
2
 2l 3 
3 2  1
 3l  1 

2 x3 3  3   3
(do f  x   2 , x   i tụ tr   ;0  ,  0;  và l  nlim an ).
3x  1 3  3   3 


. Ta có f  f  an    an  an  2  an  0 nên * 
3 1 3 5
Xét 0  l   an  . Vậ l  .
3 5 3 5
5
T g tự t hứ g i hđ  a2 k 1  đ đi u tă g, hội tụ về  .
5
 5
 nÕu n ch½n
5  5
+ N u a thì x2   x1 , x3   x2 nên ta có dãy xn   .
5  5
 5 nÕu n lÎ

D h g hội tụ.

 5
 nÕu n ch½n
5  5
+ N u a ta có dãy xn   .
5  5
 5 nÕu n lÎ

D h g hội tụ.
+ N u tồ tại n sao cho a  an thì ta có.

3
x1  an  f  x1   f  an   x2  an 1  f  x2   f  an 1   x3  an 2 ,..., xn 1  a0  .
3
Khi đ h g tồ tại xn  2 .

Vậ u a  an thì h g xá đị h.

5
+ N u 0a thì hai dãy con  x2 k  ,  x2 k 1  ù g hội tụ về 0 giới hạ ủ 0.
5
N u a  1 thì x2  f  a   a  x1 v h số đồ g i đ đi u gi , ị h ới ởi 1. Khi đ
hội tụ về 1.

3
+ N u  a  1 thì x2  f  a   1 . Khi đ t thể h sát từ x2 . Tr ờ g h p đ đi u
3
gi v ị h ới ởi 1 hội tụ về 1 .
+ N u = 1 thì xn  1 n hội tụ về 1 .

5 3 5 3
+ N u a ta có  lim a2 n và a0  tồ tại a2 k , a2 k  2 sao cho a2 k  2  a  a2 k Thật
5 3 5 n  3
3
vậ , á số hạ g ủ  a2k  h g thể ù g ằ trái a0  , hú g ũ g h g thể ù g ằ
3
3 5
ph i a u th thì a  a2 n   lim a2 n  ).
3 n  5

3
Vậ a   a2 k  2 ; a2 k   x2   a2 k ; a2 k 2  ,..., x2 k   a2 ; a0  , x2 k  2   a0 ;    x2 k  2  . Khi đ t ại
3
đ đi u gi , ị h ới ởi 1 hội tụ về 1.

5 3
Vì f x h ẻ tr ờ g h p   a  0,  1  a   , a  1, a  1 t h sát t g tự.
5 3
K t uậ : Điều i để xá đị h v giới hạ hữu hạ .

3 5
a ;a ; a  an , n  1, 2,3,... .
3 5
n
Bài 6. Cho dãy số an  xá định bởi 0  a1  1 và an 1  an  , n  1 . Chứng minh rằng
an
lim  an  n   0 .
n 

Hướng dẫn giải


1
Áp dụng b t đẳng thức AM-GM ta có a2  a1   2 (do a1  1 ).
a1

Nhận xét: an  n, n  2 .

Ta sẽ chứng minh nhận xét này bằng ph g pháp qu p.


Thật vậy.
Với n  2 ta có a2  2 đú g .

Gi s ak  k .

k
Ta có ak 1  ak   k  1  ak2  k   k  1 ak .
ak

 ak2   k  1 ak  k  0 .

  ak  1 ak  k   0 đú g .

Suy ra ak 1  k  1 .

Nh vậy an  n, n  2 điều ph i chứng minh).

n n
M t khác, an 1   n  1  an    n  1  an  n   1 .
an an

an2   n  1 an  n  an  n  an  1 (1).
 
an an

Áp dụng (1) ta có.

  a2  2  a2  1
a3  3 
 a2
  a  3 a3  1
a4  4  3
 a3 .
 ...

  an  n  an  1
an 1   n  1 
 an

Suy ra  a3  3 a4  4  ...  an 1   n  1  


 a2  2  a2  1 a3  3 a3  1 ...  an  n  an  1 .
a2 a3 ...an
 an1   n  1 
 a2  2  a2  1 a3  1 ...  an  1 .
a2 a3 ...an

 1  1  1
 an 1   n  1   a2  2  1   1   ... 1   .
 a2  a3   an 
n
 1
 an1   n  1   a2  2   1   (2).
i 2  ai 

n
an  1
1 a 1 an an n
Ta lại có 1   n 1   (do an  n   1 ).
an 1 an 1 an 1 an 1 an
n
 1 a1 a2 an1 a1
Suy ra  1  a   a . ...  .
i 2  i  2 a3 an an

a1 a
Từ (2)  an 1   n  1   a2  2  .   a2  2  . 1 (vì an  n ).
an n

a1
 0  an 1   n  1   a2  2  . .
n
a1 a
Mà lim  0  lim  a2  2  1  0 .
n  n n  n

D đ lim  an 1   n  1   0 hay lim  an  n   0 .


n  n 

1 a 
Bài 7. Cho p  *
, a  0 và a1  0 . Xét dãy số (an ) đ xá định bởi: an 1  ( p  1)an  p 1  ,
p an 
với mọi n  1 . Chứng minh dãy số (an ) có giới hạn hữu hạn khi n  . Hãy tìm giới hạ đ .

Hướng dẫn giải


* Theo b t đẳng thức Côsi ta có:.

1  a  1 a
an 1  an  an  ...  an  p 1   p. p anp 1. p 1  a , với  n  1 . (1).
p

p an  p an
 p 1 

1 a 
an 1  an  ( p  1)an  p 1   an
p an 
D đ : .
a a anp  a
 n     0;  n  2 (2)
p p.anp 1 p.anp 1
p
Từ (1) và (2) ta có dãy số (an ) gi m và bị ch ới bởi a ;.

suy ra dãy số (an ) có giới hạn hữu hạn khi n  . .

lim an  L ; ( L  a ).
p
Gi s
n 
1 a 
Chuyển qua giới hạn h thức an 1  ( p  1)an  p 1  .
p an 

1 a 
t ph g trì h L   ( p  1) L  p 1   pLp  ( p  1) Lp  a .
p L 

 Lp  a  L  a (th
p
điều ki n).

Vậy lim an  a .
p
n 


1 
Bài 8. Ch tr ớc số thự g  và xét dãy số g  xn  th a mãn xn1     1   1 với
xn
mọi n  *
. Chứng minh rằng dãy  xn  hội tụ và tìm giới hạn của nó.

Hướng dẫn giải


1
Xét hàm số f ( x)  x  , x  0 .
x

1  x 1  1
1

Ta có f '( x)   x  1
 2 ; f '( x )  0  x  x0    1
.
x x2
Ta có b ng bi n thiên của hàm f(x):.

x 0 x0 +∞

f'(x) 0 +
+∞ +∞
f(x)

f(x0)
.
 1 
 
Suy ra f ( x)  f  x0     1
   1  (  1)  1
.

1  1
D đ xn 1     1 
  1
 xn1  .
xn xn 1

Suy ra xn 1  xn hay  xn  là dãy gi m. K t h p với xn  0 với mọi n ta suy ra dãy  xn  hội tụ.

1 

Đ t lim xn    0 . Chuyển qua giới hạ t đ c    (  1)  1
   x0 .

1

Vậy lim xn    1
.

un  (0;1)
Bài 9. Tìm t t c các hằng số c  0 sao cho mọi dãy số dãy số (un ) th a mãn  n  1
 n1
u (1  u n )  c
đều hội tụ. Với giá trị c tì đ c hãy tính giới hạn của dãy (un ) .

Hướng dẫn giải


T x t á tr ờng h p sau.
1 c cun
+N u c , thì từ gi thi t, ta có un 1    4cun ; n  1 .
4 1  un un (1  un )

1
Từ đâ ằng quy nạp, ta suy ra un  (4c) n 1 u1 . Do 4c  1 nên un   khi n   . D đ , c 
4
không th a mãn.

1  1  1  4c 1  1  4c  a(1  b)  c
+ N u 0  c  , thì tồn tại a, b   ;  , a  b sao cho  . Thật vây, l y
   
4  2 2  b (1 a ) c
 1  1  4c 1  1  4 c 
a   ;  , đ t b  a  x ( x  0) , thì.
 2 2 
a(1  a)  c
a(1  b)  c  a(1  a  x)  c  x  .
a
Chú ý là b(1  a )  a (1  a )  c. D đ , t hỉ cần chọn x  0 h tr v b  a  x, thì đ c 2 b t đẳng
thức nêu trên.
Xét dãy số (un ) xá định bởi.

a khi n  2m
un   .
b khi n  2m  1
1
thì dãy (un ) th a mãn gi thi t h g h g hội tụ. Thành th , 0  c  ũ g h g th a mãn.
4
1 1 un
+N u c , thì un 1    un . Suy ra dãy (un ) tă g v ị ch . D đ , (un ) hội tụ.
4 4(1  un ) 4un (1  un )

1 1 1
Đ t x  lim un , thì từ gi thi t ta có x(1  x)  hay x  . Vậy lim un  . .
4 2 2

Bài 10. Cho dãy số  un  xá đị h h s u: u1  2 , un 1  un 2  un  1 , n  *


. Tìm giới hạn của dãy
1 1 1
 sn  với sn    ...  , n  *
.
u1 u2 un

Hướng dẫn giải


Bằng phép quy nạp đ gi n ta th y rằng: un  2 .

t tí h đ đi u của dãy  un  . Từ h thức un 1  un 2  un  1 t su r đ c


n  , un 1  un   un  1  0 , vậy dãy số  un  tă g.
* 2

Tính tổng: Từ h thức truy hồi 1 t su r đ c un 1  1  un  un  1 .

1 1 1 1 1 1 1
       * với n  *
.
un 1  1 un  un  1 un  1 un un un  1 un 1  1

Thay n bởi 1, 2, 3,., n vào (*) và cộng v với v á đẳng thức ta suy ra :.
1 1 1 1
  ...   1  .
u1 u2 un un1  1

Do dãy  un  tă g h i h ă g s u x y ra:.

1) Dãy  un  bị ch n trên. Theo tiêu chuẩn Weierstrass, nên  un  tă g v ị ch n trên nên nó có giới hạn.
Gi s lim un  a  a  2 . Chuyển qua giới hạn h thức (1) khi n   ta có:
n 

a  a 2  a  1  a 2  2a  1  0  a  1 , vô lý.

2) Dãy không bị ch n trên, do  un  tă g v h g ị ch n trên nên


1
lim un    lim  un  1    lim  0.
n  n  n  un

1 1 1  1
Vì th từ (2) ta suy ra: lim    ...    lim 1    1 .
n  u un  n  un 
 1 u2

1 un3
Bài 11. Cho dãy số (un) th a mãn : u0  2016; un 1  un  . Tính lim .
un2 n  n

Hướng dẫn giải


3
 1  3 1
(un 1 )   un  2   un3  3  3  6 .
3

 un  un un

3 1
Do un  0 n => (un 1 )3  un3  3    un3  3 , n .
un3 un6

suy ra (un )3  u03  3n  20163  3n, n  (1).

Lại có.
3
 1  3 1
(un 1 )   un  2   un3  3  3  6
3

 un  un un
.
3 1 1 1
 un  3 
3
  un  3  
3

20163  3n  20163  3n 2 n  3n 2

1 1
=> (un 1 )3  un3  3   n  .
n  3n 2

Suy ra.
n 1
1 n 1 1 n
1 n 1
(un )3  u13  3(n  1)     2  u13  3n     2 .
k 1 k k 1 9k k 1 k k 1 9k

n
1 1 1 1 1
Do k
k 1
2
 1 
1.2 2.3
 ... 
(n  1)n
 2  2.
n
2
 n 1  n
1
và     n 2  2n (Bất đẳng thức Bunhiacopxki).
 k 1 k  k 1 k
2
suy ra (un )3  u13  3n   2n (2).
9
Từ (1) và (2) suy ra.
2
20163  3n  (un )3  u13  3n   2n , n 
9
.
20163 (u )3 u 3 2 2
 3  n  1 3  , n 
n n n 9n n

un3
D đ lim  3.
n  n

Bài 12. Cho số thực a, xét dãy số  xn  xá định bởi:


x1  a, xn 1  ln  3  cos xn  sin xn   2014, n  1, 2... Chứng minh rằng dãy số trên có giới hạn
hữu hạn khi n  . .

Hướng dẫn giải


Đ t f  x   ln  3  sin x  cos x   2014, x  .

cos x  sin x
 f ' x  .
3  sin x  cos x

   
 3 f '  x   2 cos  x    2 f '  x  sin  x  
 4  4
.
 9  f ' x   2  2  f ' x 
2
 f ' x 
2 2
 q, x 
7

Áp dụ g định lí Lagrange cho hàm số f  x  liên tụ v đạo hàm trên , thì với mọi số thực x,y tồn tại
z sao cho:.

f  x   f  y   f '  z  x  y  q x  y  f  x   f  y   q x  y , x, y  .

Với m  n  m, n  *
 , ta có: xm  xn  f  xm 1   f  xn 1   q xm 1  xn 1  ...  q m n 1 xm n 1  x1 .

M t khác: 2014  xn  2014  ln 5, n  *


  xn  bị ch n.

D đ :   0, N  *
: q mn 1 xm n 1  x1   , m  n  N . .

Vậy  xn  là dãy Cauchy, nên dãy số đ h hội tụ.

un 1  vn 1
Bài 13. Cho hai dãy số un  và vn  xá đị h h s u: u1  1, v1  2, và un  , vn  un vn 1 khi
2
n  2 . Chứng minh rằng hai dãy un  và vn  có giới hạn và tìm giới hạ đ .

Hướng dẫn giải


 1  a b
Ta có cos  suy ra u1  cos v1 mà an  n 1 n 1 , bn  anbn 1 khi n  2 .
3 2 3 2
 
u v
Suy ra u2  1 1  2 cos 2 3 , v2  u2 v1  2 cos 3 .
2 2 2
  
u2  v2 
u3   2 cos 3 cos 2 3 , v3  u3v2  2 cos 3 cos .
2 2 4 2 3
bằ g ph g pháp qu ạp ta chứ g i hđ c.
   
u v
un  n 1 n 1  2 cos 3 cos 32 cos 33 ....cos 2 3n 1 .
2 2 2 2 2
   
vn  un vn 1  2 cos 3 cos 32 cos 33 ....cos 3n 1 .
2 2 2 2
sin 2
M t khác cos   nên ta có.
2sin 
 
 
sin sin 3 sin 2 n3 2
3 . 1 
un  2 2 .... 2  n  2 sin cot 3n 1 .
   2 3 2
2sin 3 2sin 32 22 sin 2 3n 1
2 2 2
 
 
sin sin 3 sin n3 2 sin
3 . 2 ....... 2 1 3 .
vn  2  n2
   2 
2sin 3 2sin 32 2sin 3n 1 sin 3n 1
2 2 2 2
D đ .

  
 
  3 
 1     cot n 1 
lim un  lim  n  2 sin cot 3n 1   2sin lim  n21 
n  n  2
 3 2  3 n  2 
   
 
 
.

 3 
2sin 2sin
 3 lim 2n 1  3 3 3
 n    
3 tan 3n 1 3
2

, đ t Qn  x     x  i 2  .
n
Bài 14. Với mỗi n  *

i 0

a) Chứ g i h đ thức Qn  x  có duy nh t 1 nghi m thực xn thuộc  0;1 .


b) Chứng minh tồn tại giới hạn của dãy  xn  .

Hướng dẫn giải


a) Ta có Qn  0   Qn 1  Qn  2 2
  ...  Q  n   0 .
n
2

 
nên trong mỗi kho ng  0;1 , 1; 4  ,...,  n  1 ; n 2 có 1 nghi m củ ph
2
g trì h Qn  x   0 .

M t khác, ta có det Qn  x   n đ thức Qn  x  có duy nh t 1 nghi m xn thuộc kho ng  0;1 . .
1 1 1 
b) Ta có Qn  x   Qn  x     ...  .
 x x 1 x  n2 
2

Do Qn  x  có nghi m không là nghi m của Qn  x  nên nghi m củ ph g trì h Qn  x   0 là nghi m
củ ph g trì h:.
1 1 1
fn  x     ...  0.
x x 1 2
x  n2
1 1 1
Ta có: f n  x    2   ...   0.
x  x  1  x  n2 
2 2

Nên f n  x  nghịch bi n trên  0;1 .


1 1 1
Lại có: f n  xn     ...  0.
xn xn  12
xn  n 2
1 1 1 1 1
    ...   0.
xn   n  1 xn xn  1 xn  n xn   n  1
2 2 2 2

 f n 1  xn   0  f n  xn   f n 1  xn 1   xn  xn 1 .
D đ  xn  là dãy gi m.
Lại có xn   0;1 . Vậy dãy  xn  có giới hạn.

Bài 15. Cho x1  a, x2  b  a, b   và n.xn  2  (n  1).xn 1  xn  0 , n  1, 2,... Tìm lim xn .


n 

Hướng dẫn giải


xn 1  xn
Ta có xn  2  xn1   .
n

(1) n (1) n
 xn  2  xn 1  ( x2  x1 )   .b  a  .
n! n!
n
(1)k n
(1) k
 xn  2  x1   .  b  a   x1   a  b    .b  a  .
k 1 k! k 0 k!
1 1
 lim xn  x1  a  b   2a  b  .
e e

Bài 16. Cho dãy  un  xá định bởi: u1  2; un 1  un2  un  1, n  *


. Tìm M nh nh t th a mãn
1 1 1
  ...   M , n  *
.
u1 u2 un

Hướng dẫn giải


Ta có u1  2  1 và un 1  (un  1) 2  un . Chứ g i h ằ g qu ạp t đ un  2, n  , n  2 (*).

T ại : ui 1  ui2  ui  1  ui 1  1  ui (ui  1) .

1 1 1 1 1 1
      .
ui 1  1 ui  1 ui ui ui  1 ui 1  1
n
1 1 1 1 (*)
D đ : 
i 1 ui
 
u1  1 un 1  1
 1 
un 1  1
 1, n  *
.

Suy ra M  1 .
M t há , hứ g i h ằ g qu ạp t đ (un ) tă g. D đ u giới hạ hữu hạ L thì
L  2 . Vì ph g trì h L  L  L  1
2
u h t ghi L  1 , ởi vậ (un ) h g giới hạ
 n 1 
hữu hạ . Su r lim un    lim     1 (**).
 i 1 ui 

 n 1  n0
1
Với ọi a  1 thì từ lim     1 su r tồ tại n0 sao cho u  a . D đ M  1  M  1.
 i 1 ui  i 1 i

Bài 17. Cho 4028 số thực: a1 , a2 ,..., a2014 , b1 , b2 ,..., b2014 . Xét dãy số  xn  xá đị h h s u:.
2014
xn    ai .n  bi ,  n  1, 2,3,... .
i 1

2014
Bi t số ập th h ột p số ộ g, hứ g i h rằ g a i số gu với  a  phầ gu ủ
i 1

số thự a – số gu ớ h t h gv t quá a ).
Hướng dẫn giải
2014 2014
Đ t A   ai , B   bi . ọi gs i ủ p số ộ g  xn  , thì: n.d  xn 1  x1 .
i 1 i 1

Với ọi n  *
ta luôn có: ai .n  bi  1   ai .n  bi   ai .n  bi , i  1, 2,..., 2014 .

Cộ g v với v ủ 2014 t đẳ g thứ ù g hiều, t đ :.


A.n  B  2014  xn  A.n  B .

Thay n ởi n  1 và thay n ởi 1 , có:.

A  n  1  B  2014  xn 1  A  n  1  B .

A  B  2014  x1  A  B   A  B   x1   A  B  2014 .

Cộ g v với v ủ 2 t đẳ g thứ ù g hiều i tr thu đ :.


A.n  2014  xn 1  x1  A.n  2014 .

 A.n  2014  n.d  A.n  2014 .

 d .n  A.n  2014 .
2014
 dA  .
n
2014
Vì lim  0 nên suy ra d  A . M t há  xn  gồ t số gu gs i d ũ g số
n
gu . Vậ A gu . đp .

 1
 x1 
2
Bài 18. Cho dãy số  xn  th a mãn:  2
. Chứng minh dãy số trên có giới hạn.
 x  x  n ; n  1
x
 n 1 n
n2

Hướng dẫn giải


n  n  1
* T hứ g i h xn  n 2  với ọi n  1 (1).
2
Thật vậ : n  1 đú g.
k  k  1
i s 1 đú g với n  k  1 : xk  k 2  .
2

xk2
 xk 1   k  1  xk    k  1 .
2 2
2
k

2  k
x  k 2    k  1 .
xk 2
=
k

 k  1  k  k  1
  1   k  1 .
2

k 2  2

3  k  1 k  k  1
2

  .
2 2

k  1  3  k  1   k  1 k  2 
  k  đp .
2  2  2

* T hứ g i h  xn  giới hạ .

NX:  xn  tă g v xn  0 với ọi n .

1 1 1 2
Ta có    .
xn xn 1 xn  n 2
n  n  1

1 1  1
   2 1    2 .
x1 xn  n
1
 xn  với ọi n  1 .
2 2

Vậ  xn  giới hạ .
Bài 19. Cho dãy số  an  tă g, an  0n  1, 2,3,.... và   0 . Xét dãy số  xn  xá định bởi
n
ai 1  ai
xn   . Chứng minh rằng tồn tại lim xn .
i 1 ai 1ai n 

Hướng dẫn giải


Dễ g th rằ g  xn  tă g g t.

Tr ờ g h p 1. N u   1 .
ai 1  ai 1 1 1 1 1

    1
     xn   .
ai 1ai ai ai 1ai ai ai 1 a1

vậ  xn  ị h tr đ tồ tại lim xn
n  .
Tr ờ g h p 2. N u 0    1 .

ai 1  ai 1  1 1 

      * thật vậ *   ai11  ai 1  ai   ai1  ai .
ai 1ai   ai ai 1 

ai1  ai
  ai 1 ** .
 1

ai 1  ai

T hứ g i h ** .

th số f  x   x Tr đ ạ  ai ; ai 1  .
H số th điều i ủ đị h í L gră g tồ tại số c   ai ; ai 1  th
ai1  ai a  a a  a
f c    c 1  i 1 i   ai11  i 1 i đp .
ai 1  ai ai 1  ai ai 1  ai

1
Từ đ t  xn   dãy  xn  ị h tr đ tồ tại lim xn
 a1 n  .
n
a1a2 ...an 1
Bài 20. Cho dãy số xá định bởi a0  1; a1  1; an 1   1 n  1, 2,3,... . Đ t Sn   .
a n  k 1 ak 1a k 
2  
  2

Chứng minh tồn tại lim S n


n 
tr gđ  x là phần nguyên của x ).

Hướng dẫn giải


1 1 a 1 1 1
Ta có   k 1   .
ak 1a  k  a a1a2 ...ak a1a2 ...ak 1 a1a2 ...ak a1a2 ...ak 1
k 1
2
  ak 1  1
n
 1 1  1 1
Suy ra Sn       .
k 1  a1a2 ...ak a1a2 ...ak 1  a1 a1a2 ..an 1

Chứ g i h lim  a1a2 ...an 1    .


n 

Ta có : an  1 n  2 .
n
 2   n  an 1  an  1 su r đ h tă g.

Nh vậ an  an 1  1  ...  a1  n  1 .

1
Vậ lim  a1a2 ...an 1    , suy ra lim Sn  .
n  n  a1

u1  3, v1  2

Bài 21. Cho dãy số  un  ;  vn  đ xá đị h h s u un 1  un2  2vn2  n  N 
v  2u v
 n 1 n n
.

Tì á giới hạ s u: lim 2 vn và lim 2 u1.u2 ...un


n n

x  x 
.
Hướng dẫn giải

 
2
Ta có: n  N : un 1  2.vn 1  un2  2vn2  2 2.un vn  un  2.vn (1).

 
2
Áp ụ g 1 t su r : un  2.vn  un 1  2.vn 1 .

     
2n1 2n1 2n
The qu ạp t : un  2.vn  u1  2.v1  3 2 2  2 1 (2).

 
2n
Lập uậ t g tự t ũ g : un  2.vn  2 1 (3).

 1
   
2n 2n 
u 
 n 2 2  1  2  1 
  
Từ 2 v 3 t su r :  .
v  1  2  1 2  2  1 2 
   
n n

 n 2 2  

1
    
 
2n 2n 2n
Lại : un  2 1  2 1  2 1 , từ đ su r : 2n un  2  1 .
2  

    
2n 2n
2 1 2 1
1 
   
n n
2 2 n
: vn  2 1 2 1   2 vn 
2
T g tự t n
.
2 2  
 8 8

M t há t : vn  un . D đ t t đẳ g thứ s u:.

 
1 2n
2 1
 
 1 2 2 n n
2 1     2 vn  2 un  2  1 .
n n

8 8

Nh vậ the đị h í ẹp t su r lim 2 un  lim 2 vn  2  1 .


n n

n  n 

vn 1
H ữ the đề it : vn 1  2un vn  un  .
2vn

v2 v3 vn 1 vn 1 vn 1
Suy ra: u1.u2 ...un  . ...   .
2v1 2v2 2vn 2n v1 2n 1
vn 1 1
Vậ lim 2n u1.u2 ...un  lim 2n n 1
 lim 2 vn1 .lim 2n n1 .
n

n  n  2 n  n  2

1 1
 lim 2 2un vn .lim 2n  lim 2 2.lim 2 un .lim 2 vn .lim 2n n 1 .
n n n n

n  n  n 1    
2 n n n n 2

 1.  2 1 .  
2  1 .1  3  2 2 .

T ại t : lim 2 vn  2  1 và lim 2 u1.u2 ...un  3  2 2 .


n n

n  n 

n
Bài 22. Cho dãy số  an  xá định bởi 0  a1  1 và an 1  an  , n  1 . Chứng minh rằng
an
lim  an  n   0 .
n 

Hướng dẫn giải


1
Áp ụ g t đẳ g thứ AM-GM ta có a2  a1   2 (do a1  1 ).
a1

Nhậ x t: an  n, n  2 .

T sẽ hứ g i h hậ x t ằ g ph g pháp qu p.
Thật vậ .
Với n  2 ta có a2  2 đú g .

i s ak  k .

k
Ta có ak 1  ak   k  1  ak2  k   k  1 ak .
ak

 ak2   k  1 ak  k  0 .

  ak  1 ak  k   0 đú g .

Suy ra ak 1  k  1 .

Nh vậy an  n, n  2 điều ph i chứng minh).

n n
M t há , an 1   n  1  an    n  1  an  n   1 .
an an

an2   n  1 an  n  an  n  an  1 (1).
 
an an

Áp ụ g 1 t .
  a2  2  a2  1
a3  3 
 a2
  a  3 a3  1
a4  4  3
 a3 .
 ...

  an  n  an  1
an 1   n  1 
 an

Suy ra  a3  3 a4  4  ...  an 1   n  1  


 a2  2  a2  1 a3  3 a3  1 ...  an  n  an  1 .
a2 a3 ...an

 an1   n  1 
 a2  2  a2  1 a3  1 ...  an  1 .
a2 a3 ...an

 1  1  1
 an 1   n  1   a2  2  1   1   ... 1   .
 a2  a3   an 
n
 1
 an1   n  1   a2  2   1   (2).
i 2  ai 

n
an  1
1 a 1 an an n
T ại 1  n 1   (do an  n   1 ).
an 1 an 1 an 1 an 1 an
n
 1 a1 a2 an1 a1
Suy ra  1  a   a . ...  .
i 2  i  2 a3 an an

a1 a
Từ 2  an 1   n  1   a2  2  .   a2  2  . 1 (vì an  n ).
an n

a1
 0  an 1   n  1   a2  2  . .
n
a1 a
Mà lim  0  lim  a2  2  1  0 .
n  n n  n

D đ lim  an 1   n  1   0 hay lim  an  n   0 .


n  n 


1 
Bài 23. Cho tr ớc số thự g  và xét dãy số g  xn  th a mãn xn1     1   1 với
xn
mọi n  . Chứng minh rằng dãy  xn  hội tụ và tìm giới hạn của nó.
*

Hướng dẫn giải


1
th số f ( x)  x  , x  0 .
x

1  x 1  1
1

Ta có f ( x)   x  1
 2 ; f ( x )  0  x  x0    1
.
x x2
T g i thi ủ h f  x  :.

x 0 x0 +∞

f'(x) 0 +
+∞ +∞
f(x)

f(x0)
.
 1 
 
Suy ra f ( x)  f  x0     1
   1  (  1)  1 .

1  1
D đ xn 1     1 
  1
 xn1  .
xn xn 1

Suy ra xn 1  xn hay  xn  gi . K t h p với xn  0 với ọi n ta suy ra dãy  xn  hội tụ.



1 

Đ t lim xn    0 . Chu ể qu giới hạ t đ    (  1)  1
   x0 .

1

Vậ lim xn    1
.

u1 , u2  (0;1)

Bài 24. Cho dãy số thực  un  th a mãn  1 3 43 . Chứng minh rằng dãy (un ) có
u n  2  u n 1  u n , n  1
5 5
giới hạn hữu hạn, tìm giới hạ đ .

Hướng dẫn giải

 x1  min u1 , u2 

Xét dãy ( xn ) :  1 3 4 .
 xn 1  xn  3 xn
 5 5
T th xn  (0;1) .

1 3 43 x3  3 xn  3 xn  3 xn  3 xn 5 133
Ta có xn 1  xn  xn  n  xn  xn .
5 5 5
Vậ  xn  tă g, ị h tr hội tụ, lim xn  a (0  a  1) .

1 4
Chu ể qu giới hạ t đ : a  a3  3 a  a  1 .
3 5
T sẽ hứ g i h xn  u2 n 1 ; u2 n  1 * ằ g qu ạp the .

Ta có x1  u1 ; u2  1 . i s xn  u2 n 1 ; u2 n  1 .

1 3 43 1 4
Suy ra xn 1  xn  xn  u23n  3 u2 n 1  u2 n 1  1 .
5 5 5 5
1 3 43 1 4 1 4
xn 1  xn  xn  xn31  3 xn  u23n 1  3 u2 n  u2 n  2  1 .
5 5 5 5 5 5
Vậ * đú g với ọi gu g. Từ đ su r lim un  1 .

 x1  2007

Bài 25. Cho dãy số thực  xn  xá định bởi:  xn . Chứng minh dãy số ( xn ) có

xn 1  3  n  1
 xn2 1
giới hạn và tìm giới hạ đ .

Hướng dẫn giải

Dễ g qu ạp xn  3 .

xn 1
Ta có: xn 1  3  = 3  1  3  2 n  1 .
xn2  1 x 1
2
n

Vậ xn  2007 với ọi n ị h .

x 1 1
Xét f  x   3   f  x    f  x  khi x  3 .
x 1 x  1
2 3
2 2 2

Ta có:.

x x2
f  x  x  x  3   ( x  3)  2 2

x2  1 x 1 .

 ( x 2  3x) 2  2( x 2  3x)  3  0

 x 2  3 x  1 ( L)

 x  3 x  3
2
.
3  15
x a
2
Áp ụ g đị h ý L gr g :.
n
1 1 
xn1  a  f ( xn )  f (a)  f '( n ) xn  a  xn  a  ...    x1  a 
n 
0 D đ
2 2 2 2
3  15
lim xn  a  .
2
u1  e un21
Bài 26. Cho dãy số  un  xá định bởi:  . Tìm lim .
un1  un  2, n 
2 * n  u 2 .u 2 ...u 2
1 2 n

Hướng dẫn giải


1
Vì u1  e  2 đ t u1  a  , a > 1.
a
2
 1 1
Ta có u2  u  2   a    2  a 2  2 .
2
1
 a a

1
un 1  a 2  , n 
n
Bằ g qu ạp, t thể hứ g i hđ n .
a2
Xét.
1 1
n
 i1
n
1   1   1  n  2i1 1   1   2n 1 

i 1
ui    a 2  2i1
i 1  a

 
 
a   
a  
a     a  2i1     a    a  2n 
a  i 1  a   a  a 
2
 1  2n 1 
 a   a  2n  2 2
.
   
2 2
 2 n21 2  
u a  a   lim u 1 1
n 1
  a     a    4  e2  4
 a  a
2 n  u 2 .u 2 ...u 2
u1 .u2 ...un  2n 1 
 2n 
1 2 n
a
 a 

B i 1. Ch số  xn  xá đị h ởi.

 x1  a

 xn2  7 .
 n 1 2  x  3 , n  1, 2,3,...
x 
 n

Chứ g i h rằ g số giới hạ hữu hạ . Tí h giới hạ đ .


Hướng dẫn giải
Theo Côsy thì.

1
xn   xn  3 
16   x  1 xn  7   0 .
 6   1; xn1  xn   n
2 xn  3  2  xn  3

gi , ị h ởi 1, vậ giới hạ .
Từ lim xn  a  a  1 .

 x1  1

Bài 27. Cho dãy số  xn  , xá định bởi:  2014 . Chứng minh rằng dãy số  xn  có
 xn 1  1  1  x , n  1, 2,3...
 n

giới hạn hữu hạn và tìm giới hạ đ .

Hướng dẫn giải


2014
t h số f ( x)  1  trên  0;   . T th f ( x) i tụ v ghị h i tr  0;   (Vì
1 x
2014
f '( x)   0 . D đ 1  f ( x)  2015 .
1  x 
2

2014
Ta có xn 1  1   f ( xn ) với ọi  dãy  xn  ị h .
1  xn

M t há , t x1  x3  f ( x1 )  f ( x3 )  x2  x4  f ( x2 )  f ( x4 )  x3  x5  ... .Suy ra dãy  x2 n 1 


đ đi u tă g v ị h , ò  x2n  đ đi u gi v ị h , á  x2 n1  ,
 x2n  giới hạ hữu hạ .

i s lim x2 n 1  a và lim x2 n  b , ( a, b  1 ).

Từ x2 n 1  f ( x2 n )  lim x2 n 1  lim f ( x2 n )  b  f (a ) .
x2 n  2  f ( x2 n 1 )  lim x2 n  2  lim f ( x2 n 1 )  a  f (b) .

 2014
b  1  1  a
Vậ t h   a  b  2015 .
a  1  2014
 1 b

Vậ i xn = 2015 .

 x1  2,1

Bài 28. Cho dãy số  xn  đ xá định bởi  xn  2  xn2  8 xn  4 với mỗi số
 xn 1  * , n  1, 2,...
 2
n
1
gu g , đ t yn   2 . Tìm lim yn .
i 1 xi  4

Hướng dẫn giải


T t qu s u: với số thự a  2 t ì, t .

a  2  a 2  8a  4 a  2  a 2  4a  4 a  2   a  2 
  a.
2 2 2
D đ 2,1  x1  x2  ... Suy ra dãy  xn  tă g, gi s ị h tr tứ giới hạ lim xn  L  2 .

Chu ể qu giới hạ điều i * t ph g trì h.

x  2  x2  8x  4
x  x 2  4   x  3 x  2  .
2
ph g trì h h g ghi hữu hạ ớ h 2.

Suy ra dãy  xn  tă g v h g ị h tr lim xn   .

xn  2  xn2  8 xn  4
Ta có xn1   2 xn1  xn  2  xn2  8 xn  4 .
2

  2 xn 1  xn  2   xn2  8 xn  4  xn2 2  4   xn  3 xn  2  .


2

1 x  3 xn  2  1 1 1
  2n  2   2 .
xn  2 xn 1  4 xn 1  4 xn 1  2 xn 1  4

1 1 1
   .
x  4 xn  2 xn 1  2
2
n 1

n
1 1 1 1
Suy ra yn      10  .
i 1 x  4 x1  2 xn 1  2
2
i xn 1  2

Vậ lim yn  10 .

 x0  a

Bài 29. Dãy số thực  xn  n   đ
xá định bởi:   n   . Tìm t t c các giá trị
 xn 1  2 xn  1

2

củ để xn  0 với mọi số tự nhiên n.


Hướng dẫn giải
i s xn  0 với n  .

2
Từ xn  2  2 xn21  1  0 có   xn 1  0 .
2

2 2  2 2 1
Lại từ   2 xn2  1  0 có   xn   1  xn   , n  .
2 2 2 4
1 3 1
Suy ra xn   và xn   1, n  .
2 4 2

1 1 1 1 1 3 1
Từ đ xn 1   2 xn2  1   2 xn2   2 xn  . xn   xn  , n  .
2 2 4 2 2 2 2
Áp ụ g i ti p t đẳ g thứ này, ta có:.
2 n n
1 1 2 1 2 1 2 1 2
a   x0   x1     x2   ...    xn     , n  .
2 2 3 2 3 2 3 2 3
n
2 1 1
Mà lim    0 ph i a 0a  .
 
n 3 2 2

1 1
Th ại với a   thì xn    0, n .
2 2
1
Vậ a   giá trị u h t ầ tì .
2

 x1  2014
Bài 30. Cho dãy số thực (xn xá định bởi:  .
 xn1  3 6 xn  6sin xn , n 
*

Hướng dẫn giải

x3
S ụ g t đẳ g thứ x   sin x  x, x  0 .
6
th số f  x   3 6 x  6sin x , x  0 .

6 1  cos x 
Ta có: f '  x    0, x  0  f x u đồ g i với ọi x > 0.
 6 x  6sin x 
2
33

D đ : f  x   f  0   0 x  0 . mà x2  f  x1   0 vì x1  2014  0. .

Vậ t xn 1  f  xn   0, n  N * .

6 xn  6sin xn  xn3
M t há : xn 1  xn  3 6 xn  6sin xn  xn  .
 6 xn  6sin xn 
2
3  xn 6 xn  6sin xn 
3 xn2
x3
Vì x   sin x  x, x  0  6 x  x3 – 6sinx  0, x  0 .
6
 6 xn – 6 sinxn  xn3  0 do xn  0  xn 1 – xn  0 .

  xn  gi v ị h ới ởi 0 tồ tại giới hạ hữu hạ .

i s limxn  x( x  0) , t ph g trì h:.

x  3 6 x  6sin x  x3  6 x  6sin x  0 .
th số g  x   x3  6 x  6sin x .

g '  x   3x 2 – 6  6cosx .

g’’  x   6 x – 6sinx  0x  0 .

 g’  x   g’  0   0 . D đ g  x  u đồ g i v i tụ với ọi x  0 .

 ph g trì h g  x   0 ghi u h t x 0.

Vậ limxn  0 .

 1  an 1
an  bn  1  a
Bài 31. Cho hai dãy số g  an n0 ,  bn n0 xá định bởi: a0  3, b0  2 và  n 1 .
a 2  1  b 2
 n n

Với mọi n  0,1, 2,... . Chứng minh rằng hai dãy trên hội tụ và tìm giới hạn của chúng.

Hướng dẫn giải


 1
T hứ g i h ằ g qu ạp an  tan , bn  , n  0,1, 2,... (*) . Thật vậ .
3.2 n

cos
3.2n
  1
Với n  0 , ta có a0  3  tan  tan 0
, b0  2 

, vậ  * đú g.
3 3.2 cos
3.20
1   2 1
Với n  1 , ta có a1   tan  tan 1 , b1   , vậ  * đú g.
3 6 3.2 3 cos 
3.21
 1
i s hẳ g đị h đú g đ n  k , k  1 , tứ an  tan , bn  .
3.2 n

cos n
3.2
 1
T hứ g i h an 1  tan , bn 1  . Thật vậ . Từ 1 ta có.
3.2 n 1

cos
3.2n 1
    
sin n  1 2sin n 1 cos n 1  sin 2  cos 2
1  an 1 3.2 3.2 3.2 3.2 n 1
3.2 n 1 
 
1  an 1   
cos n cos 2 n 1
 sin 2
3.2 3.2 3.2n 1
  
2
   
 sin n 1  cos n 1  sin n 1  cos n 1 tan n 1  1
 3.2 3.2 

     
3.2

3.2 

3.2

Khi đ từ  2 ,

 cos n 1  sin n 1   cos n 1  sin  cos 3.2n 1  sin 3.2n 1 1  tan 3.2n 1
 3.2 3.2   3.2 3.2n 1 

a n 1  tan
3.2n 1
 1 1
suy ra bn21  an21  1  tan 2 1   bn 1  .
3.2 n 1
2  
cos cos
3.2n 1 3.2n 1
 1
Nh vậ the gu ý qu ạp thì an  tan , bn  , n  0,1, 2,... .
3.2 n

cos
3.2n
 1 1
D đ lim an  lim tan  tan 0  0; lim bn  lim  1.
n  n  3.2 n
n  n   cos 0
cos
3.2n
K t uậ : lim an  0; lim bn  1 .■.
n  n 

u1  2014
Bài 32. Cho dãy số (un ) xá đị h h s u:.  . Tì điều ki n của
un 1  un  (1  2a)un  a ; n  1, 2,...
2 2

a để dãy số (un ) có giới hạn hữu hạn khi n   và tính giới hạ đ .

Hướng dẫn giải


Ta có: un 1  un  (un  a ) 2  0  un 1  un ; n  1, 2,3,... .

* Su r số (un ) tă g ; từ đ số (un ) giới hạ hữu hạ hi v hỉ hi ị h tr .

i s lim un  L ( L  ) , thì hu ể qu giới hạ h thứ un 1  un2  (1  2a)un  a 2 ta có:


n 

L  L2  (1  2a) L  a 2  L  a .

-N u hỉ số k  *
mà uk  a thì un  a; n  k trái với t qu lim un  L  a .
n 

D đ : uk  a với ọi k  1, 2,... hay un2  (1  2a)un  a 2  a, n  1, 2,3,... .

 a  1  u1  a  a  1  2014  a .

*Đ ại: N u a  1  2014  a  a  1  u1  a .

 (u1  a  1)(u1  a )  0  u12  (1  2a)u1  a 2  a  0  u2  a .

và u1  u2  a  1  u2  a .

Bằ g qu ạp t hứ g i hđ a  1  un  a, n  1, 2,3,... .
Nh vậ (un ) tă g , ị h tr ới a , đ số (un ) giới hạ hữu hạ .

Kết luận: Với điều i a  1  2014  a thì số (un ) giới hạ hữu hạ hi n   và lim un  a .
n 

u1  1

Bài 33. Cho dãy số (un ) xá định bởi công thức truy hồi  1 . Chứng minh
un 1  un  u  2, n 
*

 n

rằng dãy (un ) có giới hạn hữu hạn và tính giới hạ đ .

Hướng dẫn giải


1 1 1
Đ t f ( x)  x   2; g ( x)  f ( f ( x))  x    2 2 . Khi đ .
x x x 1  2
x

 2 2
2  x    x  1
2
g '( x)    1 1
2
 0  g ( x)  g ( )  0  f ( f ( x))  x, x  ( ;1) (*). .
4 1  2 2
x x  2
 x 
1
M t há f '( x)  0, x  ( ;1) nên.
2
1 1 1 1 1
f ( x)  f ( )  f ( f ( x))  f ( )  , x  ( ;1) (**). .
2 2 2 2 2
1 1
Từ * v ** su r :  f ( f ( x))  x, x  ( ;1). .
2 2
1 1
Vậ : 1  u1  u3   1  u1  u3  u5  ,... D đ (u2 n 1 ) đ đi u gi v ị h ới tồ
2 2
1
tại lim u2 n 1  ..
n  2

Vì f ( x) i tụ tr
 1 
 2 ;1 nên u2 n  f (u2 n 1 )  lim
  n 

u2 n  f lim u2 n 1 
n 

1
2
..

Vậ (un ) đ phâ tí h th h h i hội tụ tới ù g ột giới hạ . D đ (un ) giới hạ


1
ằ g ..
2

u1  2
 n
uk
Bài 34. Cho dãy số  un  xá định  . Tính lim  .
un 1  un  2014  un  un  , n  1
1
k 1 uk 1  1
2 n 

Hướng dẫn giải


un  un  1
The gi thi t t : un 1   un mà u1  2 suy ra.
2014
2  u1  u2  u3  ....... đ  un  tă g.
i s  un  ị h tr su r lim un  L với  L  2  hi đ .
n 

un2  2013un L2  2012 L L  0


lim un 1  lim L  .
n  2014 2014 L  1

Vô lý do L  2 . Suy ra dãy  un  h g ị h tr đ .

1
lim un    lim 0.
n  n  u
n

Ta có.

un2  2013un
un 1   un  un  1  2014  un 1  un 
2014
.
un  1 1 
  2014   
un 1  1  un  1 un 1  1 

 1 1 
 Sn  2014     lim Sn  2014 .
 u1  1 un 1  1  x
 x1  2014
Bài 35. Cho dãy số thực  xn  xá định bởi:  . Tính lim xn ? .
 xn1  3 6 xn  6sin xn , n 
*

Hướng dẫn giải

x3
S ụ g t đẳ g thứ x   sin x  x, x  0 .
6
th số f  x   3 6 x  6sin x , x  0 .

6 1  cos x 
Ta có: f '  x    0, x  0  f x u đồ g i với ọi x > 0.
 6 x  6sin x 
2
33

D đ : f  x   f  0   0x  0 . mà x2  f  x1   0 vì x1  2014  0. .

Vậ t xn 1  f  xn   0, n  N *. .

6 xn  6sin xn  xn3
M t há : xn 1  xn  3 6 xn  6sin xn  xn  .
 6 xn  6sin xn 
2
3  xn 3 6 xn  6sin xn  xn2

x3
Vì x   sin x  x, x  0  6 x  x3 – 6si x  0 . x  0 .
6
 6 xn – 6si xn  xn3  0 do xn  0  xn 1 – xn  0 .

  xn  gi v ị h ới ởi 0 tồ tại giới hạ hữu hạ .

i s limxn  x( x  0) , t ph g trì h:.

x  3 6 x  6sin x  x3  6 x  6sin x  0 .
th số g  x   x3  6 x  6sin x .

g '  x   3x 2 – 6  6cosx .

g   x   6 x – 6si x  0," x  0 .

 g  x   g  0  0 . D đ g  x u đồ g i v i tụ với ọi x  0  ph g trì h g  x   0
ghi u h t x 0.
Vậ limxn  0 .

You might also like